You are on page 1of 116

Archer USMLE Step 3 Questions

1. A 15 year old boy presents to emergency room with severe lower abdominal pain that awoke him from
sleep about 5 hours ago. The pain is sharp and radiating to the left thigh. While in the emergency room,
the patient has one episode of vomiting. He denied any fever, dysuria or chills. Physical examination
reveals normal vitals with blood pressure 100/60 and temperature of 98.6F. Abdominal examination is
relatively benign. Scrotal examination reveals an elevated left testis that is diffusely tender to palpation.
Cremasteric reflex is present on the right but absent on the left. Most important next step in managing this
patient :
a. CT scan Abdomen and Pelvis
b. Testicular Ultrasound
c. Surgical Exploration
d. Intravenous Antibiotics
e. Plain X-Ray KUB

Ans. C

Testicular ultrasound is a very close distracter in this Question but one should know when to do a
testicular ultrasound as opposed to surgical exploration in a suspected testicular torsion case.

Recognize that clinical probability of testicular ultrasound can be estimated by history and physical
examination ( see the predictive clinical score below). Ultrasound should only be done if the clinical
diagnosis is uncertain and if the performance of imaging does not significantly delay the treatment. Doing
color doppler in this patient is not necessary since the diagnosis is clinically clear and imaging will further
delay surgical intervention ( it is already 5 hrs since onset of pain)

Rapid diagnosis is important in order to salvage a viable testis with prompt surgery. The testicular salvage
rate is more than 80% if surgery is performed within 6 hours, but the rate decreases to approximately 20%
if surgery is done after 12 hours after the onset of symptoms.

Testicular Torsion: Clinical features include acute onset pain, absence of cremasteric reflex, negative
prehns sign, tender testicle on palpation and a an elevated or horizontal lie of testis ( changed position of
testis). Absent cremasteric reflex is the most sensitive physical finding for diagnosing testicular torsion.
Three features in the history can serve as predictors of pre-test clinical probability of Testicular Torsion:

1. Onset of pain less than six hours


2. Absence of Cremasteric reflex
3. Diffuse Testicular Tenderness.
Presence of all the three features ( score:3) is assocaited with 87% probability (high probability) of having
Testicular Torsion as per a large study. These patients should undergo direct surgical exploration.

A score of 1 or 2 indicates moderate to low clinical probabilty and should first undergo diagnostic
ultrasound. A score of 0 favors an alternative diagnosis for acute scrotum rather than Testicular Torsion.

Key Concept : Recognize Testicular Torsion clinical score and determine the next step as follows :
1. Onset of pain less than six hours
2. Absence of Cremasteric reflex
3. Diffuse Testicular Tenderness.
Presence of all the three features ( score:3) is assocaited high probability of having Testicular Torsion as
per a large study > Next step, direct surgical exploration. A score of 1 or 2 indicates moderate to low
clinical probabilty > next step, diagnostic ultrasound. A score of 0 favors an alternative diagnosis for
acute scrotum rather than Testicular Torsion.

2. A 75 y/o man with history of Hypertension, Pagets disease and peripheral vascular disease presents to
you with complaints of chronic hearing impairment on right side. He is otherwise physically very active.
Physical examination benign except for right sided sensorineural hearing deficit. A skull x-ray was
obtained to evaluate his pagets disease.Lab studies reveal hemoglobin of 9.7, platelets 310, wbc 10k with
normal differential, BUN 38, creatinine 1.4, calcium of 11.2. The patient is currently on alendronate for
Pagets disease.

Which of the lab measure or imaging would usually be expected to be abnormal in the condition that is
responsible for his skull x-ray findings and hypercalcemia?
A. Alkaline phosphatase
B. Serum Immunoelectrophoresis
C. Bone Scan
D. Carcino Embryonic Antigen
E. CA-15-3
Ans. B

Pagets disease history is a distractor in this case. Pagets disease does not cause hypercalcemia unless the
patient is immobilized because of poor control. The hypercalcemia in pagets is secondary to prolonged
immobilization not because of Pagets disease. This patient is physically very active.

The question is asking what is the usual abnormality seen in patients with MM. Because only MM
explains his renal insufficiency, punched out lesions in the skull and hypercalcemia. The question also
presents a skull radiograph that shows Osteolytic lesions without any concomitant Osteoblastic
process. Such pure osteolytic lesions are hallmark of MM which is why Alkaline phosphatase and Bone
scans are often normal in MM. ( Recognize that Osteoblasts are the ones that are responsible for positive
bone scan and increased alkaline phosphatase and hence, these are normal in MM which has no
osteoblastic activity in skeletal lesions).

Serum and Urine electrophoresis with immunofixation may reveal a monoclonal spike and useful for
confirmation of the diagnosis. Patient also has hypercalcemia, anemia and renal insufficiency which are
associated features in multiple myeloma.

Pagets disease, on the other hand, is characterized by mixed osteolytic and osteoblastic phases. Alkaline
phosphatase is typically elevated in patients with Paget's. The skull radiograph will show a cotton wool
appearance caused by irregular areas of sclerosis ( mixed lytic areas with blastic areas).

3. A 75 y/o woman with past medical history of CVA, HTN, DM type II is sent to the Emergency Room
from Nursing home for evaluation of fever and altered mental status. Vitals reveal a temperature of 101F,
BP 100/60, RR 22, HR 110. Physical examination reveals an elderly woman not responding to verbal
stimuli but moans in response to deep pain. Echymoses are seen on lower extremities. A Foley catheter is
present draining cloudy urine. Lab studies show Hgb of 8.6, WBC 12K, Platelets 15k, BUN 48 and
Creatinine 3.2. Prothrombin time is 14.8 and Partial thromboplastin time is 58. LDH level is elevated at
600. A peripheral blood smear is shown.

Which of the following features would most likely help in identifying the etiology of this patients
thrombocytopenia?
A. Fragmented Red Blood Cells ( Schistocytes)
B. Decreased Fibrinogen and Increased D-dimer
C. Elevated LDH
D. Decreased Reticulocyte Count

Ans. B.

The patient presents with fever and possible sepsis. Cloudy urine indicates that UTI is the possible source
of sepsis. Recognize that severe sepsis can lead to multi-organ dysfunction such as hypotension,
encephalopathy. Disseminated intravascular coagulation and renal insufficiency. The latter three are seen
in this patient.

Recognize that the important difference between DIC and TTP is that DIC is a consumption coagulopathy
i.e; it consumes the entire coagulation factors along with platelets. Hence, PT and PTT are elevated and
fibrinogen is decreased in DIC but not in TTP. The intravascular thrombi in DIC are fibrin thrombi the
lysis of these lead to increased D-Dimer and Fibrin Split products. TTP is a consumption
thrombocytopenia and is composed of platelet thrombi not fibrin so, D-dimer is usually normal in TTP.

Increased LDH suggests hemolysis here and is non-specific. MAHA is associated with increased
reticulocyte count not decreased retic. The distractors in the question are typical TTP like pentad and
scistocytes on the smear. However, realize that severe sepsis can have all these features ( Fever,
thrombocytopenia, DIC leading to MAHA, altered mental status and renal failure). So, the entire clinical
scenario should be put together in arriving at the diagnosis.

The peripheral blood smear shows Schistocytes. Recognize that schistocytes are not specific for TTP.
Schistocytes can occur in any condition that is associated with Microangiopathic Hemolysis (MAHA).
MAHA can occur in conditions where intravascular thrombi rub against RBC in tiny capillaries leading to
RBC fragmentation and hemolysis eg: MAHA can be seen in TTP, HUS, DIC, HELLP Syndrome and
Malignant Hypertension.

Key Concepts:
1. DIC is a consumptive coagulopathy-thrombocytopenia and occurs secondary to several causes.
2. TTP is non-immune consumptive thrombocytopenia. PT and PTT are usually normal.
3. Severe sepsis can resemble TTP. Full clinical picture should be considered in decision making. Source
of sepsis should be sought and ruled out in suspected cases before making a diagnosis of TTP
4. MAHA is not specific for TTP. Recognize other causes of MAHA are DIC, HUS, HELLP and
Malignant Hypertension.

4. A 72 y/o man with hx of chronic alcoholism and smoking presents to your office with extreme fatigue.
Denies any fever or weightloss or nightsweats. Vital signs were normal and physical examination is
benign. Laboratory studies reveal CBC with hgb 9.5, wbc 10k with 35% neutrophils, 55% lymphocytes
and 9% monocytes, platelets 90k. Basic metabolic panel is within normal limits. A direct coombs test is
negative. A peripheral blood smear is shown below.
Most likely cause of this patients Anemia :
A) Vitamin B12 deficiency
B) Chronic Myeloid Leukemia
C) Chronic lymphocytic leukemia
D) Hypersplenism
E) Autoimmune Hemolysis

Ans. C

The smear shows classic smudge cells. Smudge cells are fragile lumphocytes that get damaged and
smudged during the smear preparation. Presence of Smudge cells in a patient like above with absolute
lymphocytosis > 5000, is suggestive of CLL. A peripheral blood flow cytometry should be obtained to
confirm the diagnosis.

CLL is staged based on lymphadenopathy, splenomegaly, anemia and thrombocytopenia. Presence of


anemia classifies CLL as stage IV where as thrombocytopenia makes it a Stage V. However, this anemia
and thrombocytopenia can occur in a CLL patient because of autoimmune mechanisms and may not be
directly secondary to CLL. A direct coombs test must always be obtained to rule out this possibility and
hence, to avoid mis-staging of CLL. If direct coombs is positive, a Bone marrow biopsy should be done
to check if there is a CLL infiltration of marrow additionally contributing to Anemia. If there is CLL
infiltration of marrow possibly causing anemia, that suggests Stage IV CLL.

Some of the criteria for therapy in CLL are a) Anemia b) Thrombocytopenia c) Presence of bulky disease
d) severe fatigue e) lymphocyte doubling time less than 6 months.

CLL need not be treated unless the above criteria for treatment are present. This patients CLL belongs to
Rai Stage V in that he has thrombocytopenia. This patient requires treatment.

Choice B is not correct since the CBC is consistent with a lymphoproliferative disorder like CLL not a
myeloproliferative disorder like CML. In CML, you expect to see proliferation of myeloid lineage with an
increase in immature granulocytes, neutrophils and especially, an absolute increase in basophils and
eosinophils.

Choice A and D are incorrect as they do not explain the lymphocytosis or smudge cells.
5. A 30 y/o pregnant woman has a one week history of a slowly enlarging red lesion on her right thigh.
She reports having gone on a camping trip about 3 weeks ago and now recalls that she removed a tick
from the site of the lesion. An ELISA test is negative for Lymes. Upon further questioning, she also
reports contact with poison Ivy like bushes during the same camping trip :

What is the next step?


A. Re-assurance
B. Ampicillin
C. Doxycycline
D. Western blot testing
E. Topical Corticosteroid

Ans. B

Ampicillin is the most appropriate antibiotic for this pregnant woman with early stage Lymes disease. The
patient has erythema chronicum migrans which is pathognomonic of early lymes disease. ELISA may be
negative in early stage LYME disease and diagnosis must be based on clinical history. No further testing
is necessary in the presence of such strong clinical history.

Ans. A is inappropriate and not treating lyme disease can be lethal to the patient.

Ans. C is an appropriate first choice for non pregnant patients with lyme disease. Doxycycline is
classified as pregnancy category D. Tetracycline exposure during the second or third trimester can cause
permanent discoloration of the teeth.

Ans. D is incorrect. Western blot is only used to confirm a positive ELISA test because ELISA is
associated with a high rate of false positive results. In the absence of strong clinical suspicion, both
ELISA and Western blot must be positive in order to diagnose lyme disease. Such testing is especially
useful when someone is suspected to have late lyme disease manifestations. ELISA in this patient is
negative. It is not required in this case to diagnose early lyme disease given that she has strong clinical
features to support the diagnosis.
6. A 24 y/o college freshman who lives in a dormitory brought to the ER with complaints of fever,
headache and neck stiffness. Lumbar puncture revealed gram negative diplococci. He was started on
ceftriaxone and vancomycin. However, over the next two hours he develops complicated disease with
renal failure and purpura. He becomes comatose and was intubated by the anesthetist for airway
protection. After knowing that the patient likely has a meningococcal disease, the ER staff, the anesthetist
and the residents who initially cared for the patient are very concerned and requests chemoprophylaxis.
What is the most appropriate course of action?
A. Give Rifampin to the resident who collected blood from the patient
B. Give Rifampin to the anesthetist
C. Give Ceftriaxone to the Nurse who took care of the patient
D. Give Rifampin to the ER physician who initially evaluated the patient
E. Give Rifampin to all those who took care of the patient.

B is the answer

Household contacts of patients with meningococcal disease have an attack rate of 4 cases per 1000, which
is 500 to 800 times greater than the general population. Antimicrobial chemoprophylaxis is recommended
for close contacts (household members, day care center contacts, and anyone directly exposed to the
patients oral secretions). Antimicrobial chemo-prophylaxis ideally should be started within the first 24
hours after the initial case is diagnosed, with prophylaxis starting greater than 14 days after contact
offering little benefit.

1) Household contacts includes recent visitors who have stayed overnight in the 7 days preceding the
cases illness; those who share the same dormitory, military barracks, hostel bunkroom. Should receive
clearance antibiotics and vaccination.

2) Sexual contacts sexual partner(s) of case + intimate kissing partners. Should be treated as household
contacts and receive clearance antibiotics and vaccination.

3) Travel contacts seated immediately adjacent to a case on flight > 8 hours in duration. Should receive
clearance antibiotics.

4) Childcare contacts children and staff in same room group at a child-care facility attended by index
case for one period of 4 hours in 7 days preceding onset of cases illness require clearance antibiotics.

5) School and University contacts depending of the nature of the contact and whether it occurred in
relation to a specific outbreak (eg. related to a particular class) then clearance antibiotics may be
appropriate.

6) Health care worker contacts only medical personnel who are directly exposed to a cases
nasopharyngeal secretions (ie: person who intubated case if facemask)

Rifampin 600 mg taken orally twice a day for two days is the recommended antibiotic for
chemoprophylaxis, but should not be used in pregnant women due to its teratogenic effects. Other
alternatives include ciprofloxacin 500 mg orally, or ceftriaxone 250 mg intramuscularly.
7. 25 y/o woman with hx of endometriosis, has had 2 year history of migraines, however, they bother only
once or twice a month. Only one of these attacks a month makes her really disabled. She has been started
on propranolol 6 months ago and has been headache free for about three months. She says she recently
started oral contraceptive pills 3 months ago and her headaches have been out of control. She is getting
about 3 to 4 episodes of migraines per month now but no aura. Physical exam is normal. What is the most
appropriate next step in management ?
A) Discontinue Oral Contraceptive pills
B) Switch to OC Pills with low dose estrogen.
C) Start prophylaxis with Topiramate
D) Switch to OC Pills with high dose Estrogen
E) Obtain MRI Brain with gadolinium

Ans. B

Oral contraceptive pills can worsen migraine but have also been shown to improve certain types of
migraines such as Menstrual migraines. Hence, patients may report variable response in the intensity of
migraines after starting OC Pills.

In our case above, OC pills are clearly responsible for worsening of her Migraine. However, they were
started in her case for a reason she has Endometriosis and also, desires contraception! Hence, benefit vs.
risk should always be weighed in making treatment decisions like this. Often, estrogen component of the
OCP is responsible for worsening headaches. So, switching to low dose estrogens should be the first step
and it often improves the situation.

The question also tests the concept of risk factors for stroke. Migraines with aura is a known risk factor
for stroke but migraines without aura are benign. OC pills are an independent risk factor for stroke.
Hence, OC pills should not be started in any patient with increased risk of stroke ( eg: severe
hypertension, smoking, migraine with aura etc).

Key Concept : Migraines in the above patient are without aura. So, OC pills need not be discontinued.
Reducing the dose of estrogen component should be the next step in management.

8. A 72 y/o man with hx of chronic alcoholism and smoking presents to your office with extreme fatigue.
Denies any fever or weightloss or nightsweats. Vital signs were normal and physical examination reveals
generalized small lymphadenopathy and mild splenomegaly. Laboratory studies reveal CBC with hgb 9.5,
wbc 10k with 25% neutrophils, 65% mature lymphocytes and 9% monocytes, platelets 90k. LDH is
increased at 600 and reticulocyte count of 8.0% . Haptoglobin level is 22mg% ( N 27 to 160) and urinary
hemosiderin level is within normal limits. Basic metabolic panel, Vitmain B12 and Folic acid levels are
within normal limits. Peripheral smear is shown below and reveals many Smudge cells.

Most likely etiology of this patients Anemia is :


A. Microangiopathic Hemolysis
B. Bone marrow infiltration with Chronic Lymphocytic Leukemia
C. Acute Lymphoblastic Leukemia
D. Autoimmune Hemolysis
E. Hypersplenism

Ans. D

To recognize when 99% of bone marrow is occupied vs. when it is not. When you have increase retic like
this , CLL is likely involving very less of bone marrow and therefore, need not be treated based on
anemia alone because the cause of this anemia is autoimmune hemolysis associated with CLL not CLL
associated myelophthisis .

Increase reticulocyte count and increase LDH go with hemolysis. The peripheral smear does not show
Schistocytes which rules out MAHA. All it shows is smudge cells consistent with CLL. Absence of
hemosiderin rules out intravascular hemolysis. Only thing left is autoimmune hemolysis and this is
autoimmune hemolysis by exclusion even though question does not comment on direct coomb test.

Question is asked because in CLL, anemia is an indication for treating CLL and anemia can be of two
causes:

a) Bone marrow infiltration with CLL

b) Autoimmune hemolysis

We should know the difference between the two because, if anemia is present in CLL due to BM
infiltration, CLL has to be treated with chemotherapy. But, if anemia in CLL is because of autoimmune
hemolysis, steroids are the treatment not chemotherapy.

9. A 65 y/o man with history of chronic smoking and COPD presents for follow up visit in your office
after being discharged from the hospital about three weeks ago. The patient was admitted and treated in
the hospital for community acquired pneumonia and COPD exacerbation. During his hospital stay he was
noted to have microscopic hematuria on routine urinalysis. The patient denies any symptoms now. His
COPD is well controlled on tiotropium inhaler. His allergies include Isoniazid and Penicillin. Past
medical history is significant for a positive PPD test ( latent tuberculosis) for which he has been on
treatment with Rifampin for past three months. Physical examination is benign. Laboratory investigations
reveal a normal CBC and serum creatinine. Dipstick is positive for blood. A repeat urinalysis during this
visit reveals persistent microscopic hematuria with 3 RBCs/HPF. A urine cytology has been ordered. The
next appropriate step in evaluating this patients hematuria is:
A) Repeat urinalysis in 3 months
B) Non-Contrast CT scan
C) Intravenos pyelogram
D) CT urogram + Cystoscopy
E) Stop Rifampin

Ans. D

This patient has significant microhematuria defines as 3 or more RBCs/HPF established on two
occasions. He also has high risk factors for having a bladder cancer or urological malignancy. So, both
upper tract imaging in the form of CT urogram as well as bladder visualization in the form of cystoscopy
are warranted in this patient.

A. is incorrect because the patient already had >3RBCs/HPF on two occasions already establishing the
diagnosis of significant microhematuria.

B. is incorrect because this patients has no symptoms or lab findings suggesting UTI.

C. is incorrect because this patient is a high risk patient and requires both upper tract imaging as well as
cystoscopy as an initial protocol. IVP is good for upper urinary tract imaging but does not adequately
visualize the bladder. Moreover, recent recommendations favor CT urogram over IVP for upper tract
imaging.

E. is incorrect. Rifampin causes red colored urine but does not cause positive dipstick or hematuria

10. A 55 y/o man presents with shortness of breath on exertion. Laboratory studies reveal iron deficiency
anemia. Patient was started on Iron pills orally. The patient consumes a lot of red meat and was surprised
to know that he was iron deficient. Fecal occult blood testing revealed a positive stool guaic. A
colonoscopy and EGD were subsequently performed which were absolutely normal. A repeat Guaic was
performed and was found to be negative. A further investigation for the cause of iron deficiency in this
patient revealed celiac disease leading to iron malabsorption. What is the most likely cause of positive
Guaic in this patient?
A. Obscure GI Bleeding
B. Celiac disease
C. False positive from red meat
D. False positive from Iron pills
E. Colon cancer
Ans. C

Red meat can lead to false positive guaic as it may oxidize the guaic reagent ( Vitamin C on the other
hand, anti-oxidizes it and causes false ve guaic.)

Obscure bleeding ( Ans. A) is possible but in this patient the most likely cause for iron deficiency was
already established as celiac disease making intermittent obscure bleeding less likely.

Celiac disease ( Ans. B) does not cause False +ve Guaic.

Iron pills ( Ans. D) can cause black discoloration of the stool but not a false positive guaic.

The fact that the colonoscopy was normal makes bleeding from colon cancer ( Ans. E)unlikely here.

11. A 7-year-old boy is brought to the emergency department by his mother because of tea-colored
urine for the last several days. He has also had some nausea and vomiting, and his eyes appear swollen
when he wakes up in the morning. The eye swelling tends to resolve over the course of the day. He is
generally very healthy and there is no family history of any chronic diseases. His temperature is 36.7 C
(98.0 F), blood pressure is 130/90 mm Hg, pulse is 96/min, and respiratory rate is 16/min. Physical
examination is unremarkable. A urinalysis shows red cell casts. At this time the most appropriate study to
confirm your diagnosis is
A. antinuclear antibody
B. antistreptolysin O antibody
C. renal biopsy
D. renal ultrasound
E. urine culture

Ans B.

Poststreptococcal glomerulonephritis is the most common cause of acute glomerulonephritis in children.


It usually follows a streptococcal pharyngitis by 1-2 weeks and a streptococcal skin infection by 2-3
weeks. It most commonly occurs in school-age children and has a male to female predominance of 2:1. It
is most commonly characterized by hematuria (microscopic or gross) with red cell casts, proteinuria,
hypertension (from fluid overload secondary to decreased glomerular filtration rate), and edema (from
retention of salt and water). Laboratory values are usually significant for markedly decreased complement
levels (C3 and C4), hypo- or hypernatremia, and a blood urea nitrogen elevated disproportionately to the
creatinine. In order to diagnose poststreptococcal glomerulonephritis with certainty, there needs to be
evidence of a preceding streptococcal infection such as an elevated ASO or streptozyme.
12. A 12 y/o boy is brought to you by his mother for skin rash and complaints of intermittent abdominal
pain, joint pains for past 2 days. He did have an upper respiratory infection about 2 days ago. On physical
exam, his vitals are normal. Abdomen is benign without any tenderness or rigidity. However, you notice
patchy purple discolorations on his extremities and the back. Lab studies are obtained that revealed
WBC: 6.6 , HGB: 15.3 , MCV: 88 , Platelets: 290,000 ( normal 180k to 400k) BUN: 11 , Creatinine : 0.6
( normal) , Anti streptolysin O titer : negative, Streptozyme : negative ,Urine dipstick : normal without
any blood Urinalysis : normal/ no rbcs/ no protein. The mother is very anxious and asks about the long
term prognosis of her son. Your response :
A) Reassure the mother that boys disorder is self limiting and does not require any follow up
B) Tell her the boy needs to be admitted and treated vigorously to prevent renal failure
C) Tell her that renal failure develops 100% of such cases and hence needs very cautious follow up
D) Tell her that 50% of such cases progress to end stage renal disease.
E) Tell her that the boy requires follow up monthly urinalysis for at least 3 months in order to make sure
there is no hematuria/ renal dysfunction.

13. If the boy presented with Renal failure in the above case, the most likely underlying pathology would
be :
A) IgA mediated vasculitis
B) Post streptococcal glomerulonephritis
C) Anti GBM disease
D) Acute tubular necrosis
E) Interstitial Nephritis.

Answers :
12. E
13. A

12. E) Tell her that the boy requires follow up monthly urinalysis for at least 3 months in order to make
sure there is no hematuria / renal dysfunction is the correct answer.

The patient has non thrombocytopenic purpura associated with abdominal pain and arthralgias. These
features are consistent with the diagnosis of Henoch Schonlein Purpura ( HSP). The initial episode of
HSP usually resolves within one month. Renal impairment may be seen at the first presentation.
However, some children may manifest with late renal disease. In children with no renal impairment at
presentation, Urinalysis and blood pressure monitoring should be followed monthly for three months after
presentation. In children presenting with renal involvement at the first presentation, should be followed
more carefully i.e; urinalysis every week for first 2 months and then every month for one year. Patients
with persistent proteinuria, hypertension, or renal insufficiency should be referred to nephrology for
further evaluation and treatment.

Choice A is incorrect because though the disease is self limiting, it needs to be followed for possible
development of renal impairment.

Choice B is incorrect no admission required because the boy has an initial mild attack of HSP without
any renal impairment at this time.

Choice C is incorrect because renal impairment may be seen only in about 20 to 50% cases of HSP. Renal
involvement is typically seen within one month of onset of systemic symptoms.

Choice D is incorrect because only 2 to 5% patients progress to End Stage Renal Disease.
13. A

HSP is IGA mediated vasculitis. IgA nephropathy and Henoch-Schoenlein purpura are two different
entities that can lead to renal disease. The mechanism of renal disease in HSP is a vasculitis that is
mediated by IgA. Extrarenal manifestations are seen most often in patients with Henoch-Schoenlein
purpura where as they are uncommon in IgA nephropathy. HSP associated renal disease tends to be self-
limiting in majority of cases where as IgA nephropathy is often associated with persistent or recurrent
hematuria and/ or proteinuria.

14. A 46 y/o woman who is a school bus driver by occupation presents to your office for regular follow
up. She has a history of ADPKD. Her blood pressure is well controlled at 120/70 on enalapril. She has no
other problems. She denies any headache. There is no family history of intracranial or subarachnoid
hemorrhage. However, she is concerned that her head might explode because her sister who also has
ADPKD was recently diagnosed of having a berry aneurysm. She wants to be screened for berry
aneurysm as soon as possible. Her physical examination is benign and does not reveal any focal
neurological deficits. Which of the following suggests the necessity for screening in her case?
A. Family history of berry aneurysm
B. Polycystic kidneys
C. School bus driving
D. Cysts in the liver
E. No screening necessary in her case

Ans. C

High risk jobs ( pilot, bus driver etc) is one of the indications to screen for berry aneurysm in
asymptomatic ADPKD patients.

Family hx of berry aneurysm (Ans. A) alone does not warrant screening for berry aneurysm in
asymptomatic ADPKD patients. Asymptomatic ADPKD patients must be screened if there is a family
history of Ruptured berry aneurysm ( history of SAH in the family etc)

E. is not the answer because this patient is a school bus driver by occupation and needs to be screened.

Key Concepts : Screening for Berry Aneurysms:


- MRA of head recommended screening test to detect berry aneurysms.
- Screen only if
- family history of subarachnoid hemorrhage ( Family hx of a ruptured berry aneurysm) not just a family
history of berry aneurysm.
- Patients with high risk jobs (pilots/ bus-drivers) an event during such a job is a risk to others safety as
well.
- Patients with symptoms suggestive of a berry aneurysm
( severe headache, focal neurological deficits)
15. A 25 y/o male comes to your office with complaints of dark red colored urine and pain in the legs that
started this morning. He has been working out at the local gym excessively for the past three days. He
does consume alcohol on weekends but reports having involved in a binge drinking episode that included
10 beers yesterday. On physical examination, he weighs 70kg and he has some tenderness in his calf
muscles which he attributes to the excessive squats he performed yesterday. Urine dipstick reveals large
blood. If this patient develops acute renal failure , the most likely mechanism would be:
A) Interstitial nephritis due to pigment
B) Glomerulonephritis
C) Acute Tubular necrosis due to pigment deposition
D) Acute Tubular Necrosis due to Ischemia
E) Alcohol related direct toxic injury

16. Lab studies revealed normal electrolytes and normal creatinine but a CPK of 50,000. His Urine output
has been at 70 ml/hr for the past 6 hours. Your first step in the management to prevent development of
patients Acute Renal Faliure :
A) Intravenos Fluids
B) Furosemide
C) Calcium Gluconate
D) No treatment because serum creatinine is normal
D) Sodium Bicarbonate

17. The above patient has been adequately treated but his repeat CPK after 2 days is still elevated at
48,000. He complains of increasing pain in his left leg and some tingling and pricking sensations. On
examination his left leg was mildly swollen and there was pain on passive stretching of the leg muscles.
Dorsalis pedis and posterior tibial pulses are intact. The most likely diagnosis at this time:
A) Deep Vein Thrombosis
B) Cellulitis
C) Compartment Syndrome
D) Edema due to renal failure
E) Congestive Heart Failure

18. The immediate course of treatment in this condition would be :


A) Anticoagulation with Heparin
B) Antibiotics
C) Emergency Fasciotomy
D) Loop diuretics
E) Elevation of the leg

Answers :
15. C
16. A
17. C
18. C

Increased CPK and leg pain indicate rhabdomyolysis precipitated by alcohol and heavy exercise. Presence
of blood on the Dipstick without RBC or RBC casts indicate that this is a pigment such as myoglobin.
The mechanism of renal failure in rhabdomyolysis is pigment deposition in tubules ( Toxic ATN)
Though creatinine is normal, rhabdomyolysis can lead to rapid renal failure if not adequately treated.
Aggressive IV hydration is helpful in preventing renal failure. There is no role for diuretics or sodium
bicarbonate.

Rhabdomyolysis leads to myoglobin release in to the tissues leading to accumulation of fluid in the
compartments. Increased compartment pressure may lead to compartment syndrome and this can be limb
threatening if it is not treated with Emergent Fasciotomy.

19. A 76 year old debilitated man with history of advanced dementia lives in an extended care facility. He
has a history of recurrent Urinary tract infections which develop every six months with mild fever,
frequency of micturition and urinary incontinence. In the past, his urine cultures revealed E.coli >
100,000 colonies on several occasions. He has no indwelling Foley catheter and he is only placed on
diapers. His vital signs are temperature of 98.6, HR 88, RR 18 and a BP of 130/84. A nurse practioner
has routinely ordered a urinalysis which revealed positive leucoesterase and nitrite. This was followed by
a urine culture that revealed 100,000 colonies of E.coli. The nurse is concerned about this finding and
calls you for appropriate management.
What is the appropriate treatment?
A. Cystoscopy and Intra-venos pyelogram
B. Continuous low dose antibiotics for prevention of recurrent UTIs
C. Catheterize and irrigate the Bladder daily
D. Intravenos antibiotics with broad spectrum coverage
E. No need of treatment as this is colonization

Ans. E

Key Concepts : Recognize the definition of Recurrent UTI . Recognize the difference between
colonization and Active Infection

Answer. E No need of treatment is the correct choice because this old man has no signs of active infection
at this time. He has no fever or tachycardia or any signs of sepsis or infection.

Ans. A is incorrect. Cystoscopy and IVP can be performed to evaluate the urinary tract if the patient had
severe UTI or recurrent UTI. It is not the appropriate step at this time.

Ans. B would be appropriate as a prophylactic therapy if the patient has Recurrent UTI. Understand that
the Recurrent UTI is defined as 2 or more episodes in past 6 months or 3 or more episodes in past one
year. This patient does not meet the criteria of recurrent UTI since he suffers only one episode every 6
months.

Ans. C is not helpful in preventing UTI and is also, not appropriate step at this time. Indwelling catheters
may in fact, increase the risk of UTIs.

Ans. D would be appropriate if the patient had severe UTI. He has no signs of acute infection at this time.
20. A 26 year old woman presents to the ER with generalized weakness associated with perioral
numbness. She is moderately built and looks slightly depressed. On physical exam, she has mild pallor.
She denies use of any medications. BP 120/88 mmHg and physical exam is normal. Lab data: Cr
1.2mg/dL, BUN 15mg/dLNa 136 , K 2.8 , Cl 88 , HCO3 38. Urine Na 45 meq/L, Urine K 35 meq/L,
Urine Cl 8 meq/L, Urine specific gravity 1.010, Urine pH 7.
Most likely diagnosis is :
A)Laxative Abuse
B)Surreptious vomiting
C)Licorice abuse
D)Malabsorption Syndrome
E)Hyporeninemic Hypoaldosteronism

21. Most appropriate next step in the management:


A)IV normal saline
B)Spronolactone
C)Amiloride
D)Psychiatry consult
E)Reassurance because this is self limiting

Ans. B
Ans. A

Key concepts : Recognize the etiologies of metabolic alkalosis. Understand the concept of urinary
chloride level in identifying the etiology of metabolic alkalosis. If urinary chloride is less than 10meq/L, it
indicates Saline responsive alkalosis. A higher than 10 value indicates Saline resistant alkalosis.

Ans. B is the correct choice because the patient has hypokalemic, hypocholremic metabolic alkalosis.
Urinary chloride less than 10 indicate that this is a saline responsive metabolic alkalosis and hence,
should be treated with IV normal saline. NS can correct this metabolic alkalosis.

Ans. A incorrect because diarrhea due to laxative abuse should cause non anion gap metabolic acidosis.

Ans. C is incorrect. The active component in licorice is glycyrrhizic acid which inhibits 11B-HSD2, the
enzyme that inactivates cortisol to cortisone in the collecting duct. Inhibition or deficiency of 11B-HSD2
causes cortisol to remain active and like aldosterone, cortisol binds to aldosterone receptors causing
hypokalemia, metabolic alkalosis and low-renin and low-aldosterone hypertension. This is not licorice
abuse because this patient does not have hypertension. Also, urinary chloride will be higher than 10 in
licorice induced metabolic alkalosis

Ans. D is incorrect because it should cause chronic diarrhea and non-gap metabolic alkalosis.

Ans. E is incorrect because hyporeninemic hypoaldosteronism causes hyperkalemia and metabolic


acidosis ( Type IV RTA).
22. A 65 y-old male undergoes a screening colonoscopy which reveals a 2cm polyp. The histopathology
reveals an adenomatous polyp with no atypical cells. The most appropriate follow up for this patient is :
A) Colonoscopy at 10 yrs
B) Colonoscopy at 5 yrs and then every 5 yrs
C) Colonoscopy at 3 yrs and then every 10 yrs
D) Colonoscopy at 3 yrs and then every 5 years
E) CEA every 3 months

Ans: D

Colon polyp screening guidelines:

a) Low Risk: 1 to 2 adenomatous polyps, both small 1cm or any adenoma with villous features or high
grade dysplasia In this group, repeat Colonoscopy in 3 years. If this is normal or shows only small polyps,
repeat colonoscopy every 5 years ( remember 3, 5)

b) Intermediate Risk : 3 to 10 small adenomatous polyps or any one adenomatous polyp >1cm or any
adenoma with villous features or high grade dysplasia. In this group, repeat Colonoscopy in 3 years. If
this is normal or shows only small polyps, repeat colonoscopy every 5 years ( remember 3, 5)

c) High risk:
- Greater than or equal to 10 adenomatous polyps, colonoscopy at 1 year.
- Sessile adenomatous polyps that are removed in pieces > colonoscopy in 3 to 6 months to ensure that it
is completely removed
- Polyp removed in total and has carcinoma insitu with clear margins > colonoscopy at 1 year and then at
3 years and then, every 5 years ( Remember 1,3 and 5)

23. A 67-year-old woman has suffered a massive subarachnoid hemorrhage. The patient was well until
three days prior when she had a sudden loss of consciousness. After emergent transport to the local
hospital where her trachea was intubated and a CT scan disclosed a large grade 4 subarachnoid bleed, she
was transferred to the intensive care unit. Over the past 72 hours her condition is unchanged. She in
nonresponsive to deep painful stimulus, there is no dolls eye or gag reflex, and corneal blink reflex is
absent. She has had no narcotics, analgesics, or paralytics. There is discord present within the family
about whether to withdraw care. The husband desires care to be withdrawn whereas the children want to
continue maximal supportive care. You explain that the most important determining factor in helping to
direct future care is
A. the decision of a court appointed legal guardian
B. the decision of the medical and nursing team taking care of the patient
C. the husbands wishes given that he is the health care proxy
D. the opinion of the hospital ethics committee
E. the patients previously expressed wishes regarding life support is known
Ans E.

In these difficult situations where the patient is not able to make their own decisions and when there is
disparate views among family members, the most important directing factor should be what the patient
would have decided where they able to speak for themselves. Therefore, previously expressed views on
what the patient would want if faced with a similar situation could prove invaluable in helping to direct
care in the direction that the patient would have wanted.

Court appointed legal guardians (choice A) are utilized when a patient has no family or has family
members who are not competent to help make medical decisions on behalf of the incapacitated patient. If
the patient has a clearly expressed view, this view would supersede any sentiments by the medical team
(choice B), the husband (choice C), or a hospital ethics committee (choice D).

24. A 17-year-old girl comes to the office for a complete physical examination before going off to
college. You have been the physician for her and her brother since birth, and for both her parents for the
past 20 years. The patient has been very healthy and has had all of her immunizations. Her mother is
always in the waiting room as you obtain the history and perform the physical examination. The patient
admits to you that she has had her first sexual experience after the prom a few weeks ago. After she tells
you this she becomes very worried that you will tell her mother. At this time you should
A. advise her to tell her mother herself so you will not have to tell her
B. explain that you will not bring up the issue but that you cannot lie if the mother specifically asks if her
daughter is sexually active
C. perform a pelvic examination and obtain a Pap smear specimen
D. tell the patient that your conversations with her will remain confidential
E. try to convince her to talk about these issues with her mother

Ans. D

Regarding minors std, OCP, pregnancy are confidential until unless its threat to partner then encourage
her to tell the partner. Again no need for parents consent for this and also physician should educate her on
STD and need for contraception and safe sex techniques. At this point establishing a good patient rapport
is essential and letting her know she can be honest and safe in a conversation with you is key .

In establishing and maintaining a good physician - adolescent relationship, the physician should tell the
patient that their conversations will remain confidential. This way the adolescent will be able to trust the
physician and will be willing to discuss issues, such as sexuality, that they do not want their parents to
know about, but that may affect their health.

An exception to the confidentiality rule is the duty to warn and protect a third party from violence or
infectious conditions, such as tuberculosis, gonorrhea, HIV and syphilis.
25. A 4-year-old boy has just had his second generalized tonic-clonic seizure in a 4 month period. You
have recommended starting an anticonvulsant. The parents have concerns about the recommended
medication and would prefer to wait and see if their son has more seizures. Your most appropriate next
step would be:
A) Comply with parents wishes
B) Start anticonvulsants
C) refer to ethics committee
D) seek legal advice from hospital attorney
E) Seek court support to start the patient on medications

Ans. A

The parents have the authority to make a choice of this sort. In general, courts have been reluctant to
overrule against parental wishes in most situations where that decision does not place the child at
considerable risk. Though failure to start an anti-convulsant may increase the risk of further seizures, this
does not pose a substantial enough risk to the child to justify overriding the parents' wishes, especially
given the potential risks associated with the medication. Though you may not agree with their decision,
the decision is a reasonable one that does not place their child at substantial risk of increased harm.

26. A 4-year-old with an obviously broken forearm is brought to the emergency department by her baby-
sitter. Both the baby-sitter and emergency room staff have attempted to reach her parents without success.
Most appropriate next step:
A) Await for the parents to call back
B) Keep trying to reach the parents
C) Refer to ethics committee
D) Take a consent from patients baby sitter before treatment
E) Get an x-ray of the forearm and treat the fracture appropriately

Ans. E

Your first duty is to the health and welfare of the child. Having attempted to reach her parents for consent
without success, you should proceed with x-rays and treatment of her fractured forearm. Rapid treatment
of the child's pain and fracture are clearly in her best interest. When optimal treatment requires immediate
intervention, treatment should not be delayed even if consent has not been obtained.

Sub case:

An ill-appearing 2-year-old with a fever and stiff neck appears to have meningitis. His parents refuse a
lumbar puncture on the grounds that they have heard spinal taps are extremely dangerous and painful.

What are your obligations in this case? How should you proceed?
A lumbar puncture is the only way to diagnose meningitis and a delay in treatment could cause significant
harm to the child. Complications from the procedure are very rare, and the benefit in this case is likely to
be substantial. There is not time to obtain an ethics consult or court order. The physician should attempt
to address the parents' misconceptions about lumbar punctures and to reassure them about the safety of
the procedure and perhaps offer to use appropriate pain control methods. A second opinion from another
physician may prove helpful.

Should these efforts not result in parental permission, the physician is justified in proceeding with the
procedure and treatment of the child. While parental authority to make medical decisions for their
children is broad, it does not include choices that may seriously harm their children. As long as the
physician has used reasonable clinical judgment in determining the need for the lumbar puncture, legal
liability should be minimal.

27. A 9 year old girl is brought to the ER for a foot infection which looks serious. She needs IV
antibiotics and debridement, or you know that her foot is in danger of amputation. The mother refuses
consent for antibiotics and debridement. You discuss the need for immediate treatment and the risks in the
presence of a witness. But mother still refuses the treatment. Your next step:

1) order the antibiotic and the debridement, overruling the mother.


2) Get an emergency court order
3) get an emergency ethics consult
4) agree with the mother and not give the treatment

Ans. A

Order the antibiotic and the debridement, overruling the mother. This is life threatening situation. Over-
rule the mother denial. In this case child is going to lose her limb. We need to overrule the parents refusal.
There is no need to take court order in this emergency case.

28. You are called to obtain surgical consent from an 84-year-old man who is hospitalized after suffering
a right femoral neck fracture, for repair of that fracture. The patient has a long history of multi-infarct
dementia and major depression. He lives with his daughter and her husband. They report to you that he
lost his footing while walking and suffered a fall. There was no loss of consciousness and no evidence by
history that the fall was syncopal in nature. You read the psychiatric evaluation in his chart that notes that
the patient, despite some dementia, confusion, and odd mannerisms, is competent to make his own
medical decisions and fully understands the nature of his condition. When you enter the room to obtain
consent from the patient for surgery, he responds with situation-inappropriate responses, fails to make eye
contact with you, and is not oriented to place or time. The patient has no power of attorney or health care
proxy listed in his medical record. The most appropriate course of action is to
A. call a family member to consent for the patient

B. consent the patient for surgery, he has been cleared by psychiatry

C. consent the patient for surgery with a note explaining his condition in the chart

D. do not consent the patient for surgery

E. perform your own competency test on this patient

Ans. D

Situations such as this frequently arise in in-patient settings. The concept of informed consent has at its
core, the requirement that the patient is clear of mind so that they may understand the conversation being
had. Although many people argue that most patients fail to understand complex medical issues involved
in such decisions, a patient must be at least of a mental status that allows their independent intelligence
and thinking ability to be the limiting factor.

Calling a family member to consent for the patient (choice A) is only appropriate if that family member
has been appointed as a designated health care proxy or has legal power of attorney for the patient.
Family members cannot be used as a convenience tool when a patient has altered mental status.

Consent the patient for surgery, he has been cleared by psychiatry (choice B) is incorrect because
irrespective of what another physician's opinion is, the consenting physician must feel that the patient is
able to understand the conversation that they will be having.

Consent the patient for surgery with a note explaining his condition in the chart (choice C) presupposes
that the surgery being considered is emergent. In such cases, when the patient is an adult and no advance
directive exists, the surgery may be performed regardless of mental status. In the case of this patient, since
the procedure is non-emergent, consent must be obtained.

Performing your own competency test on this patient (choice E) is not an option since competency is a
legal term. Judges determine competence.

29. A 10 year old girl who is a Jehovahs witness is brought to the ER after a car accident. She needs IV
fluids and transfusions. The mother refuses consent for fluids and transfusions, saying it's against her
religious beliefs. Your next step:
A. Order the fluids and transfusions, overruling the mother.
B. Get an emergency court order
C. Get an emergency ethics consult
D. Agree with the mother and not give the treatment
Ans. A

No need of a court order in a life threatening situation involving a childs life. Always remember in USA,
children are given priority at the E.R. no1 gives a F*** in regards to what the parents say. Imagine, you
have a pregnant woman who is a jehovas witness refusing blood transfusion. Treat her with NS until the
babys out of the vagina! Once, the baby is out all the Jehovas go unwitnessed and your priority is the
kid!

If there is a clear and imminent threat to life, and a delay in treatment is likely to lead to the death of a
child. Healthcare professionals can overrule parents without judicial review. (eg; in this case of a blood
transfusion for a Jehovah's witness child with life threatening bleeding after a MVA). When differences
of opinion arise between healthcare providers and parents and if there is time to obtain judicial review.
Practitioner must first try to explain and convince the parents. If this fails, practitioners should approach
courts for guidance and authority.

30. One of your 27 y/o female patient who is recently divorced has been making approaches towards you.
She is one of the most beautiful women among your patients. She requests that you go for a date with her
the following weekend. The most appropriate response in such a situation:
A) Tell her that you will transfer her to your friends care so that you can get involved with her.
B) Tell her it's unethical for a physician to get involved with his own patients so you will not entertain
such a thing
C) Tell her you could go for a dinner this weekend but you will not get sexually involved as that part is
unethical
D) Tell her it's not appropriate for her to make such advances towards her physician
E) Tell her you could definitely get involved with her provided she signs a paper releasing you from any
liability

Ans. B

Tell her it's unethical for a physician to get involved with his own patients so you will not entertain such a
thing. It's unethical for a doctor to become sexually involved with a current patient, even if the patient
initiate it.

31. Lisa was one of your patients 2 years ago. While Lisa was in NJ she has been your regular patient for
her depression. She trusts and respects you a lot. She even told you earlier that she would be very lucky if
she could date a person like you at least once in her lifetime. She is a beautiful 25 year old whom any man
would like at a first sight. You have lost contact with Lisa and she has not been your patient for past 2
years because she moved to Florida. She happens to meet you incidentally at a mall and requests that you
go for a date with her the following weekend. Your response should be:
A) Tell her that since she no longer your patient you can get involved with her.
B) Refuse her proposal because it's not ethical for you to do it
C) Tell her you could go for a dinner this weekend but you will not get sexually involved as that part is
unethical
D) Tell her it's not appropriate for her to make such advances towards her past physician
E) Tell her you would definitely get involved with her provided she signs a paper releasing you from any
liability

Ans. B

Code of Ethics of the American Medical Association (AMA) expressly prohibits sexual contact with
patients that occurs concurrent with the physician-patient relationship. Similarly, the American College
of Physicians forbids sexual relationships with patients and former patients, arguing that the intense trust
often established between physician and patient may amplify the patients vulnerability in a subsequent
sexual relationship. Often, the standards set by professional organizations carry sanctions for
professionals who violate acceptable behavior. Thus, for example, the AMAs Council on Ethical and
Judicial Affairs has authority to censure, suspend or expel an active constituent member from the
American Medical Association for an infraction of the Constitution or these Bylaws, for a violation of the
Principles of Medical Ethics, or for unethical or illegal conduct

A sexual relationship with a former patient is unethical is the doctor "uses or exploits the trust,
knowledge, emotions or influence derived from the previous professional relationship"

32. You are a busy pediatrician in the city. 6 year old John has been your patient for past 3 years. Johns
mother Mary brings him to your office for a recent upper respiratory infection. You treat John
appropriately but he gets to be admitted to the hospital for serious pneumonia. You continue to treat John
at the hospital. One evening Marissa calls you and requests that you go out for dinner with her because
she is feeling so alone and has been getting quite attached to you. Which of the following is most
appropriate statement?
A) You can go out with Mary because she is not your patient and you have never treated her in the past
B) You may want to avoid going out with her because she might cling to you later on
C) You should avoid relationship with her because there is potential for adverse effects on professional
judgment and family member behavior concerning the patients health.
D) You should consult your colleague to take his advise on this issue

Ans. C

"the clinical judgment of physician who became intimately involved with the patient or family member
may become clouded and they may breach their professional responsibilities"

33. A 33-year-old man with advanced AIDS is brought by ambulance to the hospital after collapsing on
the street. Paramedics resuscitated and intubated the patient for ventilatory support and transported him to
the hospital while in an incoherent, agitated state. Physical examination and laboratory studies confirm an
extensive pneumonia involving the entire left lung. Shortly afterwards, his designee demands that the
patient be taken off the ventilator. The designee produces a copy of the patients living will indicating the
patient wishes no life support and that his designee is to make all medical decisions if he becomes
incapacitated. The patients previous hospital record contains a copy of the same living will. Which of the
following is the best course of action?
( A ) Maintain the patient on the ventilator
( B ) Contact the patients parents
( C ) Contact the hospitals attorney
( D ) Remove sedation and wean the patient
( E ) Provide sedation and pain medication and extubate the patient

Ans. E

Unless there is evidence that a patient was not mentally competent when executing a living will, had a
change of mind, or named another healthcare proxy to make medical decision for him or her, a designee
has the right to make medical decisions on behalf of a patient, including termination of ventilatory
support.

The patient has the right to make decisions about his own health care (principle of autonomy)

ethically, there is no difference between withholding or withdrawing medical care.

Although discontinuing mechanical ventilation may carry emotional repercussions, it is ethically no


different than making the initial decision not to ventilate the patient. Bringing the patient's parents into
decision making is contrary to the patient's wishes. Although the parents are not prohibited from visiting
the patient (unless he directed otherwise), the legal right to make decisions has been assigned to the
designee and is not bound by a family relationship. If the patient executed the will properly, there are no
grounds for hospital attorney to take the matter to court. It is unlikely a court will consider the case.

The patient directed that no life support should be utilized and delaying removal from the ventilator in an
attempt to avoid resulting death does not comply with the patient's wishes . If the patient is in pain or
agitated and comfort medication is removed, the physician likely is acting in an unethical and in some
legal jurisdictions, an illegal manner.

34. A 10 y/o boy is brought by his mother for extensive rash on his lower extremities that started one day
ago and has been worsening. There is no history of fever . The family just returned four days ago from a
camping trip and the mother does not recall any exposure to ticks except that the boy stepped in to a bush
while walking downhill. On examination, there is extensive erythema along with vesicles and bullae on
the front and the back of bilateral lower extremities up until the level of the knees. The upper portion of
the lower extremities is unaffected. The rest of the physical examination is normal. The best treatment for
the management of this childs condition is :
A. Topical triamcinolone
B. Prednisone orally
C. Ceftriaxone intra-muscular
D. Diphenhydramine
E. Observation
Ans B.

Oral prednisone is the treatment of choice here, since the boy has greater than 10% involvement of total
body surface area and also, extensive bullae.

The lesions are too extensive for topical steroids use. Diphenhydramine may provide symptomatic relief
but will not address the underlying pathology.

35. A 32 year old man present to your office with complaints of a lesion on his penis that developed 3
days ago. He denies any fever or chills or pain. On further questioning, he reports an unprotected sexual
intercourse with a stranger that he met in a night club 2 weeks ago. Physical examination reveals a
normal vitals and no lymhadenopathy. The genitalia examination reveals the following findings as
depicted in the picture

Most appropriate next Step in confirming the Diagnosis :


A. VDRL
B. RPR
C. FTABS
D. Cultures
E. Dark field microscopy of the scraping

Ans. E

FTABS and VDRL might be falsely negative in primary syphilis. Dark field microscopy of the
scraping will confirm the diagnosis.
36. A 25 year old woman presents to your office with history of genital lesions. The patient says that she
has had read about genital warts on the internet and is now very concerned that she might be suffering
from the same. She is sexually active with her boyfriend and does not want him to get infected with her
condition. Physical examination reveals lesions that are flesh-colored, soft pearly papules found on the
inner aspects of labia minora which are symmetrically distributed on either side of the vulva and are
easily separable from each other.

Next Step in the Management:


A) Topical Imiquimod
B) Trichloroacetic Acid
C) Treat both patient and her partner with Podophyllin
D) Re-assurance
E) Oral Acyclovir

Ans. D

Vestibular Papillae are common benign lesions that are often mistaken as genital warts and cause
unnecessary concern to the patients. So, questions like these are often high yield on the USMLE.
Vestibular papillomatosis is a normal vulvar anatomical condition. It is a female counterpart of male
pearly penile papules. A correct diagnosis is important and prevents unnecessary stress to the patient.
Vestibular papillae are flesh-colored, soft pearly papules found on the inner aspects of labia minora. They
are usually symmetrically distributed on either side of the vulva and can be easily separated from each
other on examination.

On the other hand, genital warts are not confined to the vestibule. The cauliflower or filiform projections
of genital warts tend to fuse at the base and cannot be separated easily. Acetic acid test : When 5% acetic
acid is applied to condyloma acuminatum, a whitening occurs. There is usually no whitening with
Vestibular papillae.

37. A 34 year old man with no significant past medical history is seen in your office for a painful blister
that appeared on his lower lip yesterday. He complains of burning and itching in the area. He never had
similar lesions in the past. He denies any fever or chills. Vital signs reveal a temperature of 98.4, HR 88,
RR18 and BP of 120/76. On physical examination a lesion is seen on his lower lip as shown in the
picture. rest of the physical examination is benign.
The most appropriate next step in management is:
A) Oral Acyclovir
B) Re-assurance
C) Topical Penciclovir
D) Cold compresses
E) Oral Cephalosporin

Ans. C

Topical penciclovir. The patient has Herpes Labialis: Caused by Herpes simplex Virus Type I. Most
people are asymptomatic but only few people have recurrent outbreaks. Triggers for outbreak are cold
weather, stress, trauma.

Treatment: Topical Penciclovir as your first choice. Recurrent severe cases can be treated with oral
Acyclovir.
Ans. A is incorrect. Oral acyclovir is the treatment option for severe or recurrent herpes labialis.
Ans. B is incorrect. This patient requires topical penciclovir.
Ans. D is sub-optimal management.
Ans. E is incorrect. This is not suggestive of a bacterial infection.

38. A 45-year-old woman is very concerned about an eruption on her face. She has developed lesions on
the cheeks and forehead over the last few months. They are not associated with itching. The eruptions are
worsened with by prolonged exposure to sun, excessive stress and hot drinks. She denies any history of
alcoholism. Physical examination reveals a papular eruptions with associated erythema, telangiectasia and
pustules. There are no lesions in any other areas except on her face.
The Most Likely Diagnosis :
A. Nodulo cystic acne
B. Rosacea
C. Porphyria Cutanea Tarda
D. Seborrheic Dermatitis
E. Cutaneous Lupus

39. The most appropriate next step in management :


A) Topical Corticosteroid
B) Topical Benzoyl Peroxide
C) Oral Isotretinoin
D) Topical Metronidazole
E) Oral Doxycycline

Ans. 38- B, 39- D

Oral antibiotics, such as tetracycline, doxycycline (Vibramycin), and metronidazole (Flagyl) effectively
treat papulopustular rosacea. Topical metronidazole (cream [MetroCream] or gel [MetroGel])
administered twice daily is as effective as oral tetracycline and is considered the agent of choice for
pustular and papular rosacea,

However, some patients experience burning and stinging with the use of topical metronidazole. Some
studies suggest that topical metronidazole also reduces erythema and telangiectasis in some patients.

Topical clindamycin (Cleocin) is an effective alternative in patients who are pregnant; the use of oral
tetracycline or metronidazole is undesirable during pregnancy. The antibiotic gel or cream should be
applied across the entire face, rather than spot treating the lesions.

Rosacea is a chronic, relapsing disorder, and long-term treatment is generally required. Control of
symptoms can be successfully maintained by long-term use of metronidazole gel.

SECOND-LINE THERAPIES
In recalcitrant cases where antibiotics have failed or were partially successful, oral isotretinoin (Accutane)
or topical tretinoin (RetinA) therapy may be effective. Retinoid treatment (especially in the topical form)
may help recalcitrant papular and pustular forms of rosacea but may worsen erythema and telangiectasis.
Some authorities question the role of retinoids in rosacea treatment.

Other second-line therapies include: trimethoprim-sulfamethoxazole (Bactrim, Septra), methotrexate,


dapsone, primaquine, chloroquine (Aralen), and oral prednisone; however, no studies have evaluated the
comparative efficacy or optimal dosing regimens of these agents.

Topical corticosteroids must be avoided on the face. The use of fluorinated topical steroids on the face
frequently produces a rosacea-like syndrome, and even low-potency, non-fluorinated steroids may worsen
pre-existing rosacea and delay the resolution of steroid-induced flare-ups by months.
40. A 46-year-old fisherman and Vietnam veteran presented with a recurrent rash on his arms and legs
and a painful, swollen area on his left leg of several days duration. The rash had been a problem for
about two years and was treated with several courses of antibiotics for cellulitis. The patient reported that
for the past two years his skin had been prone to blister and tear with minor trauma and that at times his
urine appeared to be dark reddish in color. On examination, he had a slight fever and an area of cellulitis
on his left leg. His face was erythematous. On his hands, arms, and legs were vesicles and small bullae,
some crusted lesions, and hypopigmented and hyperpigmented macules. What is the most important next
step in diagnosis?

A. ANA
B. Rheumatoid factor
C. Skin biopsy
D. Hepatitis C serology
E. Hepatitis B serology

Ans. D

Porphyria cutanea tarda: PCT is due to a defective enzyme (uroporphyrinogen decarboxylase) in liver .
( the enzyme is involved in heme synthesis). PCT begins in mid-adult life especially after exposure to
substances that increase the production of porphyrins (precursors of heme) in the liver such as alcohol,
estrogen e.g. oral contraceptive, hormone replacement or liver disease.

Clinical features include Sores (erosions) following relatively minor injuries, Fluid filled blisters (vesicles
and bullae) and Increased sensitivity to the sun. Characteristically, the urine is darker than usual, with a
reddish or tea-coloured hue.

If asked on the exam, consider the diagnosis of Hepatitis C infection ( important association).
DX - Elevated urinary porphyrins, woods light on urine gives marked fluorescence
RX - Avoid alcohol, Use tanning creams in sun and avoid sun in acute flare. Discontinue estrogens.
Therapeutic phlebotomy to reduce iron stores (this improves heme synthesis disturbed by ferroinhibition
of UROD. ) In patients in whom phlebotomy is not convenient or is contraindicated and in those who
have relatively mild iron overload ; use oral chloroquine phosphate (or ) hydroxychloroquine sulfate.
41. A 17 year old boy is seen in your office for a recurrent rash on his back. He noticed that the rash
became more obvious in summer as it shows up more prominently in his tanned skin. He is concerned
about his appearance and has stopped going to the beach in summer. He denies any itching or pain. He is
sexually active and occasionally uses condoms. He denies alcohol or drug use. Physical examination
reveals macular rash on his back as shown below.

The Most likely diagnosis:


A) Secondary Syphilis
B) Vitiligo
C) Pityriasis Rosea
D) Tinea Versicolor
E) Tinea Capitis

42. The most appropriate treatment for this patients condition:


A) Oral Prednisone
B) Topical Selenium Sulfide
C) Oral Flucnozole
D) Reassurance and observation
E) Topical Corticosteroid
F) Benzathine Penicillin Intramuscular

Ans. D and B

Tinea versicolor is caused by a yeast called Malassezia furfur that lives in the skin of most adults. This
exists in two forms, one of which causes visible spots. Factors that can cause the fungus to become more
visible include high humidity and immune or hormone abnormalities. However, almost all people with
this very common condition are perfectly healthy.

Because the tinea versicolor fungus is part of the normal adult skin, this condition is not contagious. It
often recurs after treatment, but usually not right away, so that treatment needs to be repeated only every
year or two.

Tinea versicolor patches that are brown or reddish-brown go right away after treatment. When this
condition produces spots that are lighter than the surrounding skin, it may take several months for overall
color to even out. It always eventually does. Tinea versicolor does not leave permanent skin discoloration.
42. A 65 y/o man with history of chronic smoking and COPD presents for follow up visit in your
office after being discharged from the hospital about three weeks ago. The patient was admitted
and treated in the hospital for community acquired pneumonia and COPD exacerbation. During
his hospital stay he was noted to have microscopic hematuria on routine urinalysis. The patient
denies any symptoms now. His COPD is well controlled on tiotropium inhaler. His allergies
include Isoniazid and Penicillin. Past medical history is significant for a positive PPD test ( latent
tuberculosis) for which he has been on treatment with Rifampin for past three months. Physical
examination is benign. Labarotory investigations reveal a normal CBC and serum creatinine.
Dipstick is positive for blood. A repeat urinalysis during this visit reveals persistent microscopic
hematuria with 3 RBCs/HPF. A urine cytology has been sent. The next appropriate step in
evaluating this patients hematuria is:
A) Repeat urinalysis in 3 months
B) Urine cultures
C) Intravenos pyelogram
D) CT urogram and Cystoscopy
E) Stop Rifampin

Ans. D

This patient has significant microhematuria defines as 3 or more RBCs/HPF established on two
occasions. He also has high risk factors for having a bladder cancer or urological malignancy.
So, both upper tract imaging in the form of CT urogram as well as bladder visualization in the
form of cystoscopy are warranted in this patient.

A. is incorrect because the patient already had >3RBCs/HPF on two occasions already
establishing the diagnosis of significant microhematuria.
B. is incorrect because this patients has no symptoms or lab findings suggesting UTI.
C. is incorrect because this patient is a high risk patient and requires both upper tract imaging as
well as cystoscopy as an initial protocol. IVP is good for upper urinary tract imaging but does not
adequately visualize the bladder. Moreover, recent recommendations favor CT urogram over
IVP for upper tract imaging.
E. is incorrect. Rifampin causes red colored urine but does not cause positive dipstick or
hematuria

43. A 55 y/o woman with history of well controlled DM Type II presents for her regular follow-
up visit. She has no new complaints. She has been well controlled on Metformin alone with a
hemoglobin A1c of 6.5. The patient is afebrile with a B.P 96/72, HR 88, RR 16. Physical
examination is benign except for decreased sensation in her bilateral lower extremities consistent
with diabetic neuropathy and bilateral lower extremity edema. Her last urinary microalbumin
about one year ago was negative. A repeat dipstick test now is positive for protein and blood but
negative for leucoesterase and nitrite. Subsequent urine microscopy reveals 4 dysmorphic
RBCs/HPF and red cell casts. Labs reveal elevated serum creatinine at 1.4 as opposed to her
baseline creatinine of 0.8 6 months ago. The next important step in approaching this patients
Renal Insufficiency is:
A) Obtain CPK level
B) 24 hour urine for microalbumin
C) Start ACE inhibitor
D) Repeat urinalysis in 3 months
E) Referral to Nephrologist and Renal biopsy
F) Start emperic antibiotic therapy for UTI

Ans. E.

The presence of red cell casts indicate glomerular origin of this patients hematuria. Etiologies
include various glomerulonephritis and hence, a renal biopsy is warranted. A nephrotic syndrome
that is expected with Diabetic nephropathy should not have dysmorphic red cells or red cell casts
in urine. Presence of these indicates Nephritis but not nephrotic syndrome.

A. is not the answer because here a positive dipstick is also followed by a positive urinalysis
indicating true hematuria. A myoglobinuria will have positive dipstick but no RBCs on urine
microscopy.
B. is not the answer because it does not add anything to elucidate the cause of this patients
hematuria. In view of concomitant presence of RBC casts, this patients acute onset protein in
the urine may be secondary to glomerulonephritis rather than DM nephropathy.
C. Presence of RBC casts indicate glomerual cause of hematuria. So, the patient should be
referred to a nephrologist rather than a urologist
D. Repeating urinalysis in 3 months is appropriate for a new microscopic hematuria without any
features suggesting kidney involvement. Here hematuria is clearly glomerular in origin and
requires further work up as soon as possible.
F. Is incorrect because this patient has no evidence of UTI. The patients clinical features as well
as urinalysis findings do not suggest a UTI. The patient has no fever or dysuria. Dipstick is
negative for leucoesterase or nitrite. Urinalysis has no WBCs or WBC casts. Absence of all these
make UTI an unlikely etiology of her hematuria.

44. A 24 y/o athlete presents to your office with complaints of reddish discoloration of urine. He
claims that he has been exercising and running vigorously for the past two days. He is very
determined to lose the extra weight that he has put up in the recent months and has been fasting
in the nights for the past one week. His past medical history is significant for two abdominal
surgeries which included laparotomy and appendicectomy in the past for intermittent severe
abdominal pain. The patient does not smoke but does occasional consumes alcohol in binges. He
did involve in one such alcohol binge last night. Physical examination is benign except for
decreased power and reflexes in bilateral lower extremities. There is no rash. His urine specimen
was grossly red in color. Urine dipstick was negative for protein, blood, leucoesterase and nitrite.
Urine microscopy did not reveal any RBCs, WBCs or Casts. Serum creatinine and complete
blood count are within normal limits. A Creatinine Phosphokinase ( CPK) level has been ordered
but is not yet available. The most likely cause of this patients grossly red urine is :
A) Rhabdomyolysis
B) Paroxysmal Nocturnal Hemoglobinuria
C) Acute Intermittent Porphyria
D) Await CPK level for correct diagnosis
E) Glomerulonephritis

Ans. C

Reddish discoloration of urine with a negative dipstick for blood suggests that this red color is
not from either a pigment globin ( hemoglobin or myoglobin) or a Red blood cell ( Hematuria).
Such red colored urine with negative dipstick can be seen with drugs such as Rifampin, foods
such as beets and substances like porpyrins in urine.

This patient also has sensory as well as motor neuropathy in his lower extremities, a typical
manifestation of Acute intermittent porpyria attacks. The presence of peripheral neuropathy in
patients with history of recurrent abdominal pains should raise the suspicion of Acute
Intermittent Porphyria ( AIP). This patient had several severe abdominal pain episodes which
were misdiagnosed as appendicitis and he even underwent a futile laparotomy. Patients are pain
free between the attacks. Fasting and drugs like phenobarbital, alcohol can precipitate AIP
attacks. Unlike other porphyrias, rash is not typically seen in AIP.

A. is not the answer because dipstick would be positive for blood in rhabdomyolyisis.
B. is not the answer because dipstick would be positive in hemoglobinuria.
D. is not the answer since the diagnosis of reddish urine here is not in favor of myoglobinuria.
E. a negative dipstick and negative microscopic urinalysis rules out gross hematuria as a cause of
this red urine.
F. Negative dipstick for blood, negative urine microscopy and absence of RBC casts rule out
glomerulonephritis as a cause of this patients red urine.

45. A 45 y/o woman presents to the Emergency room with complaints of severe flank pain and
nausea. Patients past medical history reveals chronic smoking and occupational exposure to
aniline dyes. Physical examination reveals mild right costo-vertebral angle tenderness. The
patient is afebrile. Laboratory investigations reveal urine dipstick positive for blood but negative
for leucoesterase and nitrite. Urinalysis reveals numerous RBCs per HPF. There are no RBC
casts or WBC casts. Urine HCG is negative. The next step in managing this patients
condition is:
A) Start intravenous antibiotic therapy
B) Non Contrast CT scan
C) Contrast enhanced CT Scan
D) CT urogram and Cystoscopy
E) Ultrasonography

Ans. B

This patients clinical features as well as hematuria on urinalysis suggest renal colic from
possible urolithiasis. Non Contrast CT is the best and first imaging test of choice in evaluating
renal calculi ( do not choose plain x-rays or ultrasound. Ultrasound is optimal only in pregnant
patients). The patient is at risk for bladder cancer however, her current symptoms of acute
painful hematuria suggest a renal colic not a bladder malignancy.

A. is not correct as this patient has no evidence of pyelonephritis or UTI ( absent wbc casts,
absent wbcs or fever, no leucoesterase or nitrite on dipstick).
C. is incorrect because CT urogram involves administration of contrast after a initial non contrast
study. A non contrast CT is sufficient in most cases to evaluate the presence of stones ( except
Indinavir stones in HIV positive patients on HAART where a contrast CT is preferred).
Cystoscopy is not needed as acute flank pain with hematuria in this patient favors renal calculus
more than a bladder cancer.
D. IVP is incorrect since non contrast CT is the best test to visualize the stones.
E. is incorrect because Non contrast CT is better than ultrasound in visualizing the renal calculi

46. A 55-year-old man presented for a regular follow-up to your office 2 weeks ago at which
time a palpable nodule of 1.7 cm was noted in the left thyroid lobe. He denies a history of head
and neck irradiation, hoarseness, pain, dysphagia, or hemoptysis. His physical exam is otherwise
normal, with no lab abnormalities. Most appropriate next step in management?
A. Ultrasound of thyroid
B. TSH level
C. Fine Needle Aspiration ( FNAC)
D. Observation
E. Suppressive therapy with levothyroxine

47. The patient in Q46. subsequently, underwent an FNAC which revealed Papillary Carcinoma
of thyroid. Staging work-up revealed no evidence of distant metastases and a neck CT scan does
not reveal any lymphadenopathy. The most appropriate management of his thyroid cancer
involves:
A. Radio iodine therapy ( RAI)
B. Partial thyroidectomy
C. Total thyroidectomy
D. Lifelong levothyroxine + Total Thyroidectomy + RAI therapy
E. Total Thyroidectomy + Lifelong levothyroxine
F. Partial thyroidectomy + lifelong levothyroxine

Q46. Ans. B

TSH level is the first step for palpable thyroid nodule. A high or normal TSH indicates a cold
nodule and this requires further evaluation with FNAC. If the TSH is low, next step would be
RAIU scan.

Choice A is incorrect. Ultrasound helps to detect non palpable nodules and also, gives some
useful information on whether the nodule is benign vs. malignant. However, when a thyroid
nodule is palpable, TSH should be performed first and further approach should be defined based
on TSH results.
Choice C is incorrect. FNAC should be subsequent step after TSH . If TSH is high or normal
suggesting a cold nodule, proceed with FNAC
Choice D is incorrect because the nodule is greater than 1.5 cm and requires further evaluation as
per AACE guidelines.
Choice E is incorrect. This is an appropriate option if the work-up reveals a hot nodule.

Q47. Ans. D

Papillary carcinoma of thyroid requires Total thyroidectomy and adjuvant Radio-Iodine therapy
to destroy any remaining thyroid tissue. After surgery and RAIU therapy, patients are inevitably
hypothyroid. Hence, they require lifelong levothyroxine. Lifelong levothyroxine is also
important to keep TSH low in order to prevent recurrence of thyroid cancer. Thyroglobulin level
must be carefully monitored every 6 months in patients with history of thyroid cancer to detect
recurrence or persistent or metastatic disease.
Choices A, B, C, E and F are sub-optimal management and does not completely eradicate thyroid
cancer.

48. A 55 year old man was recently found to have a 2.0 cm thyroid nodule on palpation during
his annual physical. An ultrasound revealed no suspicious features of malignancy. TSH and free
T4 levels were normal. Patient denies any history of neck irradiation, pain, dysphagia or
hemoptysis. There is no history of cancer in his family. The next best step in evaluation of the
nodule :
A. Suppressive therapy with levothyroxine
B. FNAC
C. Lobectomy with isthmectomy
D. Observation
E. Radio iodine therapy
Ans. B.

Many students have questions about how to approach a Thyroid Nodule. The questions on
thyroid nodule are often very high yield on Endocrinology portion of USMLE Step 3. Our
experience showed that most students usually got them wrong and have wrongly interpreted the
guidelines. Many books give different recommendations and students find it confusing. So, we
made an attempt to briefly summarize the guidelines here. These recommendations are taken
from most popular updated sources in medicine and American College of Endocrinology
guidelines.

A. Approach Palpable Thyroid Nodule

Cold nodules are more likely to be malignant when compared to hot nodules ( hot functioning
nodule virtually rules out malignancy)

a. If thyroid nodule palpable => Get TSH First.


b. If High TSH suggests cold nodule/ Hashimatos => Get FNAC

(source: NEJM)(AACE recommends ultrasound as the next step here because hashimatos may
have benign nodularity that regress with therapy and ultrasound will help to see if there are
suspicious features. If U/S suspicious, then FNAC is recommended. This may be optimal
approach because hurthle cells of hashimatos may cause false positives on cytology if the FNAC
is obtained from such benign nodule so, we would recommend that you choose ultrasound as
your next step if that is there in your MCQ choices. If the choices have no ultrasound, choose
FNAC as answer). Further approach will depend on FNAC results. For hypothyroidism issue
Treat with levothyroxine if overt hypothyroidism or if subclinical hypothyroidism that warrants
treatment.

a) If TSH normal suggests cold nodule next step, get FNAC.


b) If TSH low suggests Hot nodule ( toxic adenoma) but not confirmative (What if there is
GRAVES in the surrounding tissue and this is a cold nodule?) so, next step get RAIU scan. If
RAIU scan shows Hot nodule treat with I131 ( if there is overt hyperthyroidism from this toxic
adenoma) or just observation. If RAIU shows COLD nodule, get FNAC.

Further Approach depends on FNAC results :


a) If FNAC is benign Suppressive therapy with LT4 in some cases if cosmetically warranted
b) If FNAC is malignant/ suspicious SURGERY
c) If FNAC is non-diagnostic repeat FNAC. If repeat FNAC is again non-diagnosotic, surgery

B) Approach Thyroid Incidentalomas

Thyroid Incidentalomas These are those nodules ( not the palpable ones) detected on
ultrasound such as when ultrasound was done for other purposes such as for other palpable
thyroid abnormalities or during carotid artery imaging or ultrasound done for
hyperparathyroidism).
The next step in such nodules discovered on the ultrasound depends upon the features of the
nodule.
FNAC is indicated in such incidentally discovered thyroid nodules if :
- Nodule > 10 mm in diameter
- On ultrasound, if nodule has suspicious features of malignancy hypoechoic,
microcalcifications, irregular shape, blurred margin or increased vascularity
- If there are risk factors for thyroid cancer ( family history, childhood neck irradiation)

49. A 45 year old woman presents to your clinic with history of intermittent epistaxis. Of late,
this has become more frequent. The patient has a history of atrial fibrillation for which she has
been taking aspirin as recommended by her cardiologist. She has no other past medical history.
The patient also takes over the counter medications such as Vitamin b-complex which she thinks
keeps her from getting tired. She says she also takes about 4 grams of Omega 3 fatty acids to
keep her heart healthy and Ginkgo biloba to slow the ageing of her brain. Her laboratory
tests reveal normal complete blood count. Prothrombin time and partial thromboplastin time are
within normal limits. Which of the following is your next step in managing this patients
recurrent epistaxis?
A. Advise her to discontinue omega 3 fatty acids.
B. Discontinue Aspirin
C. Advise her to stop both Omega 3 fatty acids and Ginkgo biloba
D. Advise her to stop Ginkgo biloba
E. Advise her to stop Vitamin B-complex

Ans. C

Advise to discontinue both omega 3 fatty acids and ginkgo biloba. Ginkgo biloba potentiates
aspirin by increasing the bleeding time. Omega 3 fatty acids in large doses can also prolong the
bleeding time by decreasing the platelet aggregation. Since this patient is having recurrent
epistaxis, it is advisable to discontinue both of them at this time and observe.

Ans. A is sub-optimal management.


Ans. B is incorrect. The patient has lone atrial fibrillation and she requires aspirin to reduce her
stroke risk.
Ans. D is sub-optimal management.
Ans. E is incorrect. B-complex has no effect on bleeding time or platelet aggregation.

You must understand the relative importance of each drug presented in the question. You must
weigh the risk versus benefit of stopping a drug. You must also assess the severity of the
symptoms. In this case, she has minor bleeding in the form of epistaxis. If this were a major
bleeding, it is justified to stop all the anti platelet drugs. It is not just the ASA but it is the
interaction between Ginkgo, ASA and Omega that is increasing her risk of bleeding. Omega fatty
acids and Ginkgo biloba are adjuncts that the patient is using at her own discretion. So, Ginkgo
and omega must be stopped first. ASA is used in this patient for a reason it is being used for
stroke prevention because she has lone atrial fibrillation. Stopping ASA would increase her
stroke risk and it is an unacceptable step. If the patient were to suffer a stroke, medico-legally,
your license could be shot for failing to balance risk versus benefit of your intervention.
Similar language is used so that you can understand the concept better no pun intended!

50. A 50 year old woman with history of coronary artery disease and hypertension presents to
your office for regular health check up. The patient is compliant with her medications which
include aspirin, atorvastatin and enalapril. Her blood pressure is 110/70. Laboratory
investigations including CBC and comprehensive metabolic panel are within normal limits. A
fasting lipid panel that was performed one week ago reveal an LDL cholesterol of 65, HDL of 50
and Triglycerides of 150. You discuss the results with her and you inform her that the goals of
therapy are being adequately met. She seemed happy to know about the results but tells you that
lately, her mood has been slightly low. There are good days but she tends to have frequent bad
days as well. She enjoys surfing as she used to before. She has no weight loss and her appetite is
good. She denies any suicidal ideations. She asks you if there is any medication that would
benefit her heart and also, help her mood. The next best step in managing this patient is :
A. Start escitalopram
B. Start clozapine
C. Refer to psychiatrist
D. Start Omega 3 Fatty Acids
E. Start St.Johns Wort

Ans. D

Omega 3 fatty acids are useful in improving the risk of coronary artery disease. Omega 3 fatty
acids are also effective in reducing serum triglycerides and improves mood in mild to moderate
depression. This is the medication that will help address her heart issues as well as her mood.
Choice.

A is incorrect because the patient does not have significant depression that warrants
pharmacotherapy at this time. Escitalopram also does not have any beneficial effect on
cardiovascular risk.
Choice. B is incorrect because the patient does not have pychotic depression that warrants
clozapine at this time. Clozapine also increases cardiovascular risk by causing obesity, metabolic
syndrome and worsening hyperlipidemia.
Choice. C is incorrect because the patient does not have significant depression that warrants
psychiatry referral at this time.
Choice E is incorrect. St.Johns wort is useful in treating mild to moderate depression. However,
St.Johns wort has no effect in improving the patients cardiovascular risk.

51. A 75 year old woman was diagnosed with Stage II breast cancer one year ago. The patient
received chemotherapy, underwent modified radical mastectomy and radiation. . The cancer was
ER+, PR+ and Her2-neu negative. The patient has been receiving Tamoxifen for the past few
months. She reports that she has been experiencing vaginal spotting and intermittent mild vaginal
bleeding over the past few months. She has also been experiencing intermittent hot flashes after
starting Tamoxifen therapy. On physical examination, her vitals are within normal limits. Pelvic
examination does not reveal any gross pathology. Next step in managing this patient:
A. Stop Tamoxifen
B. Start oral progesterone
C. Obtain endometrial biopsy
D. Do a hysterosalpingogram
E. Recommend Hysterectomy with bilateral oophorectomy

Ans. C

Tamoxifen is a selective Estrogen Receptor Modulator. It acts as an antagonist on the breast but
acts like an agonist on the endometrium. Hence, Tamoxifen is used in breast cancer therapy but it
also, increases the risk of endometrial hyperplasia and endometrial cancer. Vaginal bleeding is a
less common adverse effect with Tamoxifen and is usually, benign. However, when a post
menopausal patient on Tamoxifen develops vaginal bleeding, endometrial biopsy must be
performed at least initially to exclude malignancy.

Stopping Tamoxifen ( Ans. A) may stop bleeding if it is tamoxifen related adverse effect but it
will not clarify whether the cause of the bleeding in benign or malignant.
Oral progesterone ( Ans. B) is the treatment for benign endometrial hyperplasia and will help
prevent vaginal bleeding from benign hyperplasia. It is not yet clear if this patients vaginal bleed
is secondary to benign hyperplasia until a biopsy is performed.
Hysterosalpingogram (Ans. D) may reveal the pathology such uterine masses but will not
provide tissue diagnosis.
Hysterectomy with bilateral salpingo-oophorectomy ( Ans. E) is too aggressive measure at this
time. It can be recommended if the patient has biopsy proven early stage endometrial cancer.
52. Sam is a 35 y/o alcoholic who is brought to the ER in a comatose state. Sams wife tells you
that she had an argument in the evening about 5 hrs ago over Sams alcohol habits. Sam
apparently got mad over the discussion, drove his car and returned an hour ago in a very
intoxicated state. Wife called the EMS and rushed him to the ER. On examination Sam is
disoriented and hallucinating , Pulse 120 Tm 99, RR 26 BP 126/76. The rest of the physical
exam is normal except for stupor's state and alcohol smell. Lab studies revealed Na 130 k 3.4 cl-
95 Hco3 16, Glucose 90 Creatinine 1.6 BUN 45. Blood Ethylalcohol level was 180. Serum
osmolarity was 360mg. ABGs revealed pH 7.28, Pco2 28, Po2 76 Sao2 93. The next best step in
management ?
A) Endotracheal intubation in view of severe acidosis
B) Hemodialysis because this is an acute renal failure causing acidosis
C) Fomepizole because of suspicion of ethylene glycol intoxication
D) Supportive treatment for now because this is an ethylalcohol induced lactic acidosis
E) Bicarbonate drip to reverse the acidosis because this is renal tubular acidosis

Ans. C

Anion Gap: 130+3.4-95-16 = 22.4 and serum osmolarity 360 (high)


Fomepizole is the inhibitor of alcohol dehydrogenate and is the antidote for suspected ethylene
glycol poisoning in this case can be diagnosed by the prolong osmolar gap which cannot be
explained by high ethyl alcohol alone.

The pH can help in ruling out the co-ingestion of methanol and ethylene glycol, because
significant acidemia is associated with those ingestions. However, reports in the literature have
documented that the co-ingestion of ethanol and methanol does not cause significant acidosis.

53. 4-month-old girl is brought by her concerned mother because the child has been
inconsolably crying for 6 hours. The child is breast fed and has been doing well. There is no
vomiting, diarrhea, constipation, or increased gas. The mother has not changed her diet and the
infant is on no medications. There has been no fever. On physical examination the child is active
and screaming. The physical examination is normal except that there is redness and swelling
affecting the left third toe with indentation proximal to the redness. It is tender to touch. There
was no history of similar problems. The picture is shown below:
The most likely working diagnosis:
A. Trauma
B. Herpetic whitlow
C. Hairy tourniquet syndrome
D. Acute paronychia
E. Ingrown toe nail
F. Acute Gout

54. The most appropriate next step in management :


A. Joint aspiration
B. X-ray of the foot
C. Inspect the toe for hair strangulation
D. Incision and drainage
E. Topical acyclovir

Ans. C and C

Postpartum hair loss can not only make a mess of a mothers appearance and bathroom counters,
but can cause the loss of her childs extremities if shes not careful.

Hair tourniquet syndrome occurs when a hair becomes wrapped around a childs appendage so
tightly as to cut off circulation. Toes, fingers or genitals can be affected. The syndrome happens
most often in the postnatal period when mothers experience increased hair loss. Occasionally a
thread or fibre is the culprit.

A typical presentation is a screaming baby with no obvious cause for complaint. Upon closer
inspection, redness, tenderness and swelling of the affected area most commonly the middle
finger or toe may be noted. Often the hair itself is no longer visible due to the swollen tissues
around it, making it difficult to unwind or cut.

Untreated, hair tourniquet syndrome can cause progressive swelling, ischemia (lack of oxygen)
and tissue necrosis (death), and can cause auto-amputation of the strangulated structure.
Maybe theres a legitimate reason for a mom haircut after all. Removal is indicated in all cases of
hair or thread tourniquet syndrome and should be implemented as early as possible.
Adequately visualized constricting bands with little or no tissue edema are good candidates for
the unwrapping technique. Cases associated with mild to moderate edema are candidates for the
blunt probe cutting technique.

In cases where other techniques have failed, when the swelling is so severe that the constricting
band is not visible, or when epithelialization has occurred, the incisional approach should be
implemented. The use of depilatory creams is probably better suited for mild to moderate cases
of hair tourniquet syndrome.

55. A 55-year-old man presented for a regular follow-up to your office 6 months ago at which
time a palpable nodule of 1.7 cm was noted in the left thyroid lobe. A TSH level was normal.
Subsequently, he underwent an FNAC which revealed Papillary Carcinoma of thyroid. Staging
work-up revealed no evidence of distant metastases and a neck CT scan did not reveal any
lymphadenopathy. He underwent total thyroidectomy combined with RAIU therapy and was
started on levothyroxine. The patient arrives for follow up visit at 6 months. His TSH level at 6
month is less than 0.1. He denies any symptoms such as palpitations or chest pain or heat
intolerance. His weight is unchanged. Most appropriate next step in management?
a) Reduce levothyroxine dose
b) Discontinue levothyroxine
c) Obtain serum thyroglobulin level
d) Obtain anti-thyroglobulin antibodies and serum thyroglobulin levels.
e) Radio-iodine scan

Ans. D

Low TSH levels in the bloodstream reduce tumoral growth rates and reduce recurrence rates of
well-differentiated thyroid carcinomas. The extent to which TSH should be suppressed is
controversial. Most authors recommend reducing TSH levels to 0.1 mU/L. This level provides
adequate thyroid suppression while avoiding deleterious cardiac and bone effects of profound
thyroid suppression.

Perform a postoperative scintiscan of the neck 4-6 weeks after thyroid hormone replacement
therapy is completed. At this time, a scan of the neck can help demonstrate if thyroid tissue is
still present. If thyroid tissue is present, a dose of radioactive iodine is administered to destroy
residual tissue. The patient is then placed back on thyroid hormone replacement (levothyroxine)
therapy.

Serum antithyroglobulin antibodies are measured in addition to thyroglobulin because their


presence invalidates the assay. Thyroglobulin antibody levels should be obtained with each
thyroglobulin measurement. Rising thyroglobulin level after thyroid ablation suggests
recurrence. Because we do not know the status of thyroglobulin and anti thyroglobulin antibodies
before treatment.
Evaluate thyroglobulin serum levels every 6-12 months for at least 5 years. Consider a level
greater than 20 ng/mL, after TSH suppression, to be abnormal. A recurrence of thyroid cancer
can be detected if a rise in the thyroglobulin level occurs. A study by Brassard et al found that
thyroglobulin measurements allow prediction of long-term recurrence with excellent specificity.
TSH stimulation may be avoided when thyroglobulin levels measured 3 months after ablation are
less than 0.27 ng/mL during levothyroxine treatment.

56. A 42 year old presents with a pain & swelling in the left knee. The symptoms started 3days
ago and have been progressively worsening. Upon further questioning, the patient tells you that
he has experienced intermittent pain and swelling in the toes, wrists and ankles for more than 10
years. On examination the left knee is swollen and warm to touch. There is also soft tissue
swelling of the wrists, ankles and right foot. He has a 10 year history of intermittent pain &
swelling in the toes, wrists & ankles. On examination the right knee is warm to touch with an
effusion, & there is soft tissue swelling of the wrists, right ankle & right foot. X-ray of the wrist
is shown below. Laboratory studies reveal increased ESR at 90mm/hr WBC are elevated at 12k.

The most likely diagnosis :


A. Early erosive osteoarthritis
B. Reactive Arthritis
C. Psoriatic Arthritis
D. Pseudo-Gout
E. Rheumatoid Arthritis

57. The most important next step in management :


A) Obtain Rheumatoid Factor level
B) Obtain Anti citrullin peptide level
C) Start prednisone
D) Start ibuprofen and Methotrexate
E) Arthrocentesis of the Left knee

56. Ans. E
The patients clinical presentation is consistent with acute flare of Rheumatoid arthritis. The x-
ray reveals extra-articular erosion of ulnar styloid process. The involvement of proximal joints
with accompanying soft tissue swelling, symmetrical involvement, history of chronic arthritis
and extra-articular erosions on the x-ray are consistent with the diagnosis of Rheumatoid
Arthritis. American Rheumatism Association (ARA) has developed 6 criteria for the diagnosis of
RA. 4 out of this 6 criteria are necessary to diagnose RA.

These include morning stiffness more than one hour, arthritis of 3 or more joint areas, arthritis of
hand joints (wrist, MCP or PIP joint), Symmetric arthritis, Rheumatoid nodules, Serum
rheumatoid factor and Radiographic changes typical of rheumatoid arthritis on posteroanterior
hand or wrist radiographs, which must include erosions within the involved joint or adjacent to
the involved joints. This patient already has 4 out of six criteria for RA.

Ans. A is incorrect. Early onset erosive OA is an important differential diagnosis of RA but the
patients radiographic features, symmetricity, proximal joint involvement are more typical of
RA.
Ans. B is incorrect. Reiters syndrome is characterized by asymmetric pattern of joint
involvement, symptoms or signs of enthesopathy ( inflammation at the site of tendon insertion)
and radiographic evidence of sacro-ileitis. This patient has symmetrical arthritis which readily
eliminates the possibility of Reiters syndrome.
Ans. C is incorrect. Psoriatic arthritis can be either symmetric or asymmetric oligoarthritis or
polyarthritis. Symmetric polyarthritis presentation of psoriatic arthritis is sometimes difficult to
differentiate from RA.
In addition to inflammatory arthritis, the diagnosis of psoriatic arthritis requires three of the
following features : dactylitis, psoriatic skin lesions, typical nail lesions ( pitting or onycholysis),
negative rheumatoid factor and juxta-articular new bone formation seen on the x-rays. In the
presence of typical psoriatic skin lesions, only one additional feature of the above is sufficient for
diagnosis. The patient in the question does not meet this criteria.
Ans. D is incorrect. Pseud-gout is usually acute mono-arthritis or oligoarthritis. It is very
uncommon for pseudo-gout to present with polyarthritis. Pseudo-gout is also characterized by
chondro-calcinosis which is deposition of calcium pyrophosphate crystals in the articular
cartilage. Chondro-calcinosis can be visualized on the X-rays. The x-ray presented in the above
case does not reveal chondrocalcinosis.

57. Ans. E

The patient has clinical criteria sufficient to diagnose Rheumatoid Arthritis. The current
presentation of acute left knee swelling and pain could be related to a flare of RA. However,
septic arthritis should be included in the differential diagnosis of any acute large joint swelling.
Septic arthritis is also more common in RA patients when compared to general population.
Hence, a joint aspiration must be performed to exclude septic arthritis before attributing such an
acute large joint presentation to a RA flare.

Ans. A is incorrect. The patient satisfies clinical criteria for RA diagnosis. A rheumatoid factor is
adjunctive and will not help in confirming the etiology of her left knee swelling.
Ans. B is incorrect. The patient satisfies clinical criteria for RA diagnosis. A anti-citrullinated
peptide is adjunctive and will not help in confirming the etiology of her left knee swelling. Anti-
CCP positivity also carries an unfavorable prognosis in RA.
Ans. C is incorrect. It is inappropriate to start steroid without excluding septic arthritis.
Ans. D is incorrect. It is inappropriate to start treatment of RA without excluding septic arthritis
in this patients left knee. Treatment for RA can be initiated after arthrocentesis. Ibuprofen can
be used to relieve her symptoms and a DMARD such as Methotrexate should be started due to
presence of erosions on the radiographs. In the absence of erosions, a milder DMARD such as
hydroxychloroquine should be started.

58. A 30 year old man comes to your office with complaints of pain in both the wrists and in the
hands for the past two months. The pain is particularly worse in the nights and awakens him
from sleep. It radiates to the forearms from the wrists. Occassionally, the pain is associated with
abnormal sensations in both hands. He denies any pain or paresthesiae at this time. Upon further
questioning, he also reports some mild stiffness & very mild pain in the neck in the past two
weeks. Physical Examination is normal. The most appropriate initial investigation is:
A. Nerve conduction studies
B. Plain X-ray of the wrist
C. Plain X-ray of the cervical spine
D. Rheumatoid factor
E. MRI cervical spine

59. The most likely underlying etiology of this patients clinical features is :
A. Spinal cord compression
B. Brachial plexopathy
C. Rheumatoid arthritis
D. Hypothyroidism
E. Cervical Spondylosis

Ans. A and C

A. Clinical features are consistent with Carpal tunnel syndrome (CTS)


Carpal tunnel syndrome bilateral in up to 60% of cases as compression of the median nerve in
the carpal tunnel. It is characteristic by dull pain which worse in the nights (important buzz).
Pain is often accompanied by Paresthesia and motor weakness in the hand. In some patients, pain
may radiate into forearms from the wrists. Direction of radiation of pain is important and can
help in excluding other causes (eg: brachial neuralgia). Carpal tunnel: pain radiates upwards
from wrist into the forearm. Brachial neuralgia: pain radiates down the shoulder into the arm.
Nerve conduction studies and electromyography are helpful in establishing the diagnosis and to
rule out other causes of similar pain.
C. CTS can occur with RA. Stiffness in the neck will serve as a clue for possible early RA.
Though Hypothyroidism can cause CTS, the patient in the question does not have any clinical
history suggestive of Hypothyroidism.

Note: Cervical Spondylitis is a degenerative disease of cervical spine in older people which
eventually, causes cervical canal stenosis. It may present with cervical radiculopathy where the
pain radiates down the shoulder into the arms. Paresthesia in the involved roots can occur. There
is no increase in pain during the nights in cervical rediculopathy unlike the pain in CTS. Sleep is
the provocative factor for pain in the CTS.

60. A middle aged man presents with a history of foot pain on walking for a few weeks. The
patient has a history of long standing diabetes mellitus for more than 15 years. He has been on
Metformin and Glyburide combination. His HGBA1C that was obtained 3 months ago revealed
inadequate control at 9%. He reports pain and swelling in his right foot for about one month.
On examination the foot is swollen & tender to touch. Neurological examination reveals loss of
vibratory sensation and position sensation in bilateral feet. There is also impaired light touch
until the level of knees bilaterally. Joint motion at the level of ankle is within normal limits.
Complete blood count , creatinine and ESR are within normal limits. Most likely working
diagnosis for his right foot pain is :
a) Chronic Gout
b) Septic arthritis
c) Peripheral Neuropathy
d) Charcot arthropathy
e) Reflex Sympathetic Dystrophy

Ans. D

Charcot arthropathy occurs as a complication of diabetes, syphilis, chronic alcoholism, leprosy,


meningomyelocele, spinal cord injury, syringomyelia, renal dialysis, and congenital insensitivity
to pain. Diabetes is considered to be the most common cause of Charcot arthropathy.

The autonomic neuropathy leads to abnormal bone formation, and the sensory neuropathy leads
to an insensate joint that is susceptible to trauma. The development of abnormal bone with no
ability to protect the joint results in gradual bone fracture and in the subluxation of the joint.
Acute Charcot arthropathy almost always presents with signs of inflammation. Profound
unilateral swelling, an increase in local skin temperature (generally, an increase of 3-7 above
the nonaffected foots skin temperature), erythema, joint effusion, and bone resorption in an
insensate foot are present. These characteristics, in the presence of intact skin and a loss of
protective sensation, are often pathognomonic of acute Charcot arthropathy.
Pain can occur in more than 75% of patients; however, the pains severity is significantly less
than would be expected based on the severity of the clinical and/or radiographic findings.
Instability and loss of joint function also may be present. Passive movement of the joint may
reveal a loose bag of bones. Approximately 40% of patients with acute Charcot arthropathy
have concomitant ulceration, which complicates the diagnosis and raises concerns that
osteomyelitis is present.

61. 24 year old woman presents to the Emergency Room with complaints of left sided weakness
and slurred speech. The patient has history of Systemic Lupus Erythematosus. Her SLE was
diagnosed 2 years ago when she had malar rash and abnormal blood counts. She reports that she
has not received any treatment for it since her physician felt this was not needed at that time. She
denies any history of kidney involvement. She denies any joint pains or rash. Physical
examination revealed left hemiparesis consistent with cerebrovascular accident. A CT of the
head reveals right parietal infarct without any bleed. An EKG is obtained and is normal. Anti-
cardiolipin antibody is +ve which was also positive 1 year ago as per her old records . The
patient is started on adequate therapy and follows up in your office 1 month later. Which of the
following interventions is most appropriate to prevent recurrent stroke in her case?
a) Aspirin for life
b) Warfarin for 1 year
c) Warfarin for 6 months
d) Warfarin for life
e) Hydroxychloroquine

Ans. D

In patients with SLE, consider hydroxychloroquine, which may have intrinsic antithrombotic
properties. The antithrombotic properties of hydroxychloroquine have long been recognized and
may be considered in the prophylactic treatment of a patient with SLE and a positive APL
antibody test result.

Low-dose aspirin is used widely in this setting; however, the effectiveness of low-dose aspirin as
primary prevention for APS remains unproven. Clopidogrel has anecdotally been reported to be
helpful in persons with APS and may be useful in patients allergic to aspirin

If Thrombosis like this case:


Perform full anticoagulation with intravenous or subcutaneous heparin followed by warfarin
therapy. Based on the most recent evidence, a reasonable target for the international normalized
ratio (INR) is 2.0-3.0 for venous thrombosis and 3.0 for arterial thrombosis. Patients with
recurrent thrombotic events, while well maintained on the above regimens, may require an INR
of 3.0-4.0. For severe or refractory cases, a combination of warfarin and aspirin may be used.
Treatment for significant thrombotic events in patients with APS is generally lifelong.
The patient has anti-phospholipid antibody syndrome by definition and needs anticoagulation for
life. She is now presenting with arterial thrombotic event. APLA should be repeated to confirm
the positivity.

62. A 49 y/o woman with past medical history of rheumatoid arthritis presents to the emergency
room with a 2 day history of a severely painful, warm, swollen Right knee. The patient has been
on treatment with prednisone and methotrexate . Her symptoms were well controlled until
recently. She denies any pain or swelling in any other joints at this time. Physical examination
reveals swollen and tender right knee. She is afebrile. The most appropriate next step in
management is:
A. Add infliximab
B. Increase dose of Methotrexate
C. Obtain x-ray of the Knee
D. Arthrocentesis
E. Intra-articular glucocorticoid injection

Ans. D

The patient has clinical criteria sufficient to diagnose Rheumatoid Arthritis. The current
presentation of acute right knee swelling and pain could be related to a flare of RA. However,
septic arthritis should be included in the differential diagnosis of any acute large joint swelling.
Septic arthritis is also more common in RA patients when compared to general population.
Hence, a joint aspiration must be performed to exclude septic arthritis before attributing such an
acute large joint presentation to a RA flare.

63. A 44 year old man presents to your office with pain and swelling of the small joints in his
hands and wrists. The symptoms have been progressing over the past 4 months. He denies any
fever or weight loss. He reports stiffness in his both hands that occurs every morning and lasts
for 2 hours. On examination, he has symmetrical involvement of both wrists and proximal
interphalangeal joints. The involved joints are swollen and tender. Laboratory tests shows high
ESR, negative rheumatoid factor, and a positive anti-CCP antibody. The most appropriate next
step in management :
A. Start etanercept
B. Start Ibuprofen and follow-up in one month.
C. Start Ibuprofen and Methotrexate
D. Obtain plain x-rays of hands and wrists
E. Obtain anti-nuclear antibodies
Ans. D

The patient has the criteria for the diagnosis of Rheumatoid Arthritis. Once Rheumatoid arthritis
is diagnosed, some baseline imaging study to evaluate the hands and the wrists must be
performed. This will help to assess the severity of the disease at the time of initiation of therapy
and also, for comparison with post treatment radiographs to evaluate response to the therapy .
Plain radiographs of the hands and wrists are appropriate. Further managements depends upon
the baseline severity of the disease at the time of diagnosis.

Mild RA : less than 6 inflamed joints, no extra-articular manifestations, no bony erosions or joint
cartilage loss.
Moderate RA : Elevated ESR, Positive RF or positive Anti CCP antibodies, radiographic
changes such as osteopenia or small peripheral erosions and minimal joint space narrowing, no
extra-articular disease.
Severe RA : imaging studies showing bony erosions and loss of cartilage, high titers of RF or
Anti-CCP, presence of extra-articular manifestations, Anemia of chronic disease,
Hypoalbuminemia.

Ans. A is incorrect. It is inappropriate to start therapy without assessing baseline bone disease
with an imaging study.
Ans. B is incorrect. It is inappropriate to start therapy without assessing baseline bone disease
with an imaging study. This option is appropriate if the patient is determined to have early, mild
RA. Imaging must be performed to assess if this is mild or severe RA.
Ans. C is incorrect. It is inappropriate to start therapy without assessing baseline bone disease
with an imaging study. This option is correct if the patient is determined to have early, severe
RA. Imaging must be performed to assess if this is mild or severe RA.
Ans. E is incorrect. The clinical criteria and positive anti-ccp suggest rheumatoid arthritis in this
patient. There is no reason to obtain ANA.

64. A 44 year old man presents to your office with pain and swelling of the small joints in his
hands and wrists. The symptoms have been progressing over the past 4 months. He denies any
fever or weight loss. He reports stiffness in his both hands that occurs every morning and lasts
for 2 hours. On examination, he has symmetrical involvement of both wrists and proximal
interphalangeal joints. The involved joints are swollen and tender. Laboratory tests shows high
ESR, negative rheumatoid factor, and a positive anti-CCP antibody. X-ray of the wrist and hands
reveal erosions in the proximal phalanges and ulnar styloid process. The most appropriate next
step in management :
A. Prednisone
B. Start NSAID and follow-up in one month.
C. Start NSAID and Methotrexate
D. Start Infliximab
E. Start NSAID and Hydroxychloroquine
Ans. C

This patient has severe rheumatoid arthritis. He has all the criteria for the diagnosis of RA (
morning stiffness > 1hr, symmetricity, hand joint involvement, more than 3 joints, +ve CCP and
bony erosions). His disease can be classified as severe because he has high ESR, high CCP and
most importantly, he has erosive bony disease.

Therapy depends on severity at the time of diagnosis:

Mild RA : NSAID alone can be started first. NSAID does not prevent disease progression or
erosive bone disease. If symptoms persist or progress while on NSAIDS or if no remission
occurs after six weeks on NSAID therapy, a disease modifying agent must be added ( DMARD).
For mild disease, a less toxic DMARD such as Hydroxychloroquine or Sulfasalazine must be
used initially ( not methotrexate).

Moderate RA : NSAID + less toxic DMARD such as hydroxychloroquine should be started from
the time of diagnosis.

Severe RA : An NSAID along with a more potent DMARD such as Methotrexate should be
started together as initial therapy. NSAID alone should not be used as sole therapy in severe RA
as it will not prevent disease progression. In patients who cannot be started on MTX ( for
contraindications), a TNF alpha inhibitor such as leflunomide or etanercept must be started as
initial DMARD.

Ans. A is incorrect. Prednisone is usually used for treating RA flares.


Ans. B is incorrect. This option is appropriate if the patient is determined to have early, mild RA.
Patients can be started on NSAID alone and follow up in 1 month to see if there is remission. If
disease is still present, DMARD must be added.
Ans. D is incorrect. This option is appropriate if the patient is determined to have early, severe
RA and if the patient cannot be started on DMARD such as methotrexate
Ans. E is incorrect. This option is appropriate if the patient is determined to have early, mild RA
or moderate RA. Patients can be started on NSAID alone in mild RA and follow up in 1 month to
see if there is remission. If the disease is still present, a mildly toxic DMARD must be added
such as hydroxychloroquine or sulfasalazine.

65. A 22 year old man with history of chronic backache presents to your office. The patient has a
history of progressive low back pain for 3 years. He relates his pain to a motor vehicle accident
that happened to him 3 years ago. X-rays of the spine were performed three years ago at the start
of his symptoms and were normal. He reports that his pain is worst in the morning but gradually
improves towards the end of the day. The patient has been using over the counter ibuprofen and
tylenol without any relief. He was recently started by his primary care doctor on oxycodone
which he takes 6 times daily as needed for pain. This did not relieve his pain either. The patient
is here to see you for a second opinion. The most appropriate next step in management is :
A. Plain X-ray of lumbar spine and pelvis
B. MRI of the lumbar spine
C. Prescribe long acting narcotic
D. Physical Therapy
E. HLAB27 testing

Ans. A

Patient has clinical presentation consistent with Ankylosing spondylitis. According to modified
New York Criteria, diagnosis of AS can be confirmed if it meets one radiological criterion and
one clinical criterion. Plain X-rays of the lumbar spine and pelvis will help us to know if this is
inflammatory arthritis associated with sacroileitis . Then, proceed to E

Modified New York criteria for AS. Includes Clinical and radiological parameters:

Clinical parameters:
Low back pain and stiffness for more than three months that improves with exercise, but is not
relieved by rest
Limitation of motion of the lumbar spine in both the sagittal and frontal planes
Limitation of chest expansion relative to normal values correlated for age and sex

Radiological parameters :
Sacroiliitis grade 2 bilaterally
Sacroiliitis grade 3 to 4 unilaterally

AS is confirmed if patient fulfills at least one radiological parameter plus at least one clinical
parameter.

Radiographs are the single most important imaging technique for the detection, diagnosis, and
follow-up monitoring of patients with ankylosing spondylitis. Overall bony morphology and
subtle calcifications and ossifications may be demonstrated well radiographically. The diagnosis
may be reliably made if the typical radiographic features of ankylosing spondylitis are present.
Radiographs are limited in detecting early sacroiliitis and in demonstrating subtle changes in the
posterior elements of the vertebrae.

Magnetic resonance imaging (MRI) is useful in assessing early cartilage abnormalities and bone
marrow edema. MRI is limited by its relatively poor ability to detect calcification, ossification,
and cortical bony changes. But, because cost benefits, always start with plain X ray and then
MRI.

66. A 69 year old woman presents to your clinic with history of pain in bilateral hands. Her
symptoms started about 3 years ago but lately, have been more significant. She denies any
morning stiffness or redness or swelling of the joints. On examination, there are hard growths at
distal interphalangeal joints. Clinically, you make a diagnosis of Osteoarthritis.
Radiographically, which of the following features is LEAST likely to be seen in Osteoarthritis :
A. Narrowing of the joint spaces in involved joints
B. Sclerosis near the articular surface
C. Bony erosions
D. Osteophytes
E. Sub-chondral cysts

Ans. C.

OA is degenerative joint disease ,not inflammatory . RA , Gout and Pseudogout are


inflammatory joint diseases.

Radiological finding of Osteoarthritis:


1.Joint Space Narrowing
2.Development of Osteophytes
3.Subchondral Sclerosis
4.Subchondral Cyst Formation

67. A 44 year old woman presents to your office with pain and swelling of the small joints in her
hands and wrists. The symptoms have been progressing over the past 4 months. She denies any
fever or weight loss. She reports stiffness in her both hands that occurs every morning and lasts
for 2 hours. On examination, she has symmetrical involvement of both the wrists and two of her
left proximal interphalangeal joints. The involved joints are swollen and tender. Laboratory tests
shows normal ESR, negative rheumatoid factor, and a negative anti-CCP antibody. X-ray of the
wrist and hands reveal mild joint space narrowing with only very small peripheral erosions. The
most appropriate next step in management :
A. Prednisone
B. Start NSAID and follow-up in one month.
C. Start NSAID and Methotrexate
D. Start Infliximab
E. Start NSAID and Hydroxychloroquine

68. The patient in the above question is started on appropriate therapy. She returns to your clinic
in 1 month for regular follow up and her disease is well controlled with near complete remission.
The most important next step in follow up of this patient :
A. DEXA scan in 1 year
B. PPD placement
C. Check hepatitis B serology
D. Ophthalmology referral at 3 months
E. Liver function tests

Ans. E and D
Moderate RA : NSAID + less toxic DMARD such as hydroxychloroquine should be started from
the time of diagnosis. Because it is moderate RA (small peripheral erosions, negative lab work,
normal ESR). Because you are starting on Hydroxychloroquine you want to follow up
ophthalmology.

Hydroxychloroquine side effect (Chloroquine retinopathy): One of the most serious side effects
is a toxicity in the eye (generally with chronic use). It should be noted that macular toxicity is
related to the total cumulative dose rather than the daily dose. People taking 400 mg of
hydroxychloroquine or less per day generally have a negligible risk of macular toxicity, whereas
the risk begins to go up when a person takes the medication over 5 years or has a cumulative
dose of more than 1000 grams. Regular eye screening, even in the absence of visual symptoms,
is recommended to begin when either of these risk factors occurs.

69. You are treating an 18-year-old white male college freshman for allergic rhinitis. It is
September and he tells you that he has severe symptoms every autumn, which impair his
academic performance. He has a strongly positive family history of atopic dermatitis. Which one
of the following is the most appropriate management?
a) Intranasal decongestants
b) Intranasal glucocorticoids
c) Intranasal cromolym sodium
d) Intranasal antihistamine
e) RAST testing

Ans. B

Drug of choice is intranasal steroids.


Topical intranasal glucocorticoids are currently believed to be the most efficacious medications
for the treatment of allergic rhinitis. They are far superior to oral preparations in terms of safety.

Cromolyn sodium is also an effective topical agent for allergic rhinitis; however, it is more
effective if started prior to the season of peak symptoms.
Because of the high risk of rhinitis medicamentosa with chronic use of topical decongestants,
these agents have limited usefulness in the treatment of allergic rhinitis.
Some of the newer oral antihistamines have been found to be comparable in efficacy to
intranasal steroids, but their use slightly increases the incidence of adverse effects and drug
interactions. They are not as useful for congestion as they are for sneezing, Pruritus, and
rhinorrhea. Newer agents are relatively free of sedation. Overall, they are not as effective as
topical glucocorticoids. Azelastine , an intranasal antihistamine, is effective in controlling
symptoms but can cause somnolence and has a very bitter taste.
70. A 40-year-old man presents to your office for regular follow up. He has history of
hypertension for which he has been on Hydrochlorthiazide and Lisinopril. Lately, his blood
pressure has not been well controlled and this has been documented both in the office and
outside during several visits. The patient says he has had increasing fatigue and irritability over
the last few months and has difficulty concentrating at work. He thinks his boss has never been
supportive and he attributes his irritability to this. He has smoked 2 packs of cigarettes per day
for the past 15 years. On physical examination he is a obese male with a neck circumference of
18cm. He has a ruddy complexion. His vitals reveal a HR of 90 and a BP of 152/92. Reminder
of the physical exam is normal. EKG reveal changes consistent with long standing hypertension.
A CBC, electrolyes and creatinine are normal. Which of the following is the most important
investigation that can identify the cause of his uncontrolled hypertension?
( A ) Ambulatory blood pressure monitoring
( B ) Pulmonary function studies
( C ) Polysomnography
( D ) Arterial blood gases
( E ) Urine drug screen

Ans. C

Obstructive sleep apnea (OSA) is a risk factor for systemic hypertension, but a direct etiologic
link between the two disorders has not been established definitively. Furthermore, the specific
physiological mechanisms underlying the association between OSA and systemic hypertension
have not been identified. Patient presents with chronic fatigue, decrease concentration and hx of
progressive HTN, with increase BMI and neck circumference clearly pointing out OSA. Once
you suspect this pt has OSA, you should do Polysomnogram on this patient to confirm the dx.

71. A 65 y/o African American man is brought by his daughter to you and requests a PSA test
because there is a hx of prostate ca in their family. You perform PSA and DRE. DRE does not
reveal any palpable mass. The lab test reveal : PSA : 8ng/ml, Free PSA: 1.5ng/ml. You reveal the
results to patient and his daughter. The daughter asks you if her father has a cancer. Your best
response is :
A) The PSA level increases with age and your fathers PSA is in the age-appropriate range
B) PSA level is very nonspecific and your father does not have a cancer
C) The fact that the free PSA is only 1.5ng/ml as opposed to a bound of 6.5 indicates that your
father most likely has a cancer etiology rather than benign cause
D) PSA will not help in diagnosing carcinoma prostate
E) I did this test only because you requested for it, I don't think this results mean anything.

Ans. C

The fact that the free PSA is only 1.5ng/ml as opposed to a bound of 6.5 indicates that your
father most likely has a cancer etiology rather than benign cause.
The PSA test measures the blood level of PSA, a protein that is produced by the prostate gland.
The higher a mans PSA level, the more likely it is that he has prostate cancer. However, there
are additional reasons for having an elevated PSA level, and some men who have prostate cancer
do not have elevated PSA.
The PSA test has been widely used to screen men for prostate cancer. It is also used to monitor
men who have been diagnosed with prostate cancer to see if their cancer has recurred (come
back) after initial treatment or is responding to therapy.

Digital rectal examination (DRE) has minimal effect on PSA levels, leading to transient
elevations of only 0.26 to 0.4 ng/mL, and PSA can be measured immediately after DRE.

Ejaculation can increase PSA levels by up to 0.8 ng/mL, though levels return to normal within
48 hours. We do not usually ask men to abstain from sexual activity prior to PSA measurement.
However, if an initial measurement is high enough to potentially prompt an intervention (ie,
biopsy), but close to a borderline value, it is appropriate to repeat the PSA measurement after
having the man abstain from ejaculation for at least 48 hours.

Bacterial prostatitis may elevate PSA levels, but they generally return to baseline six to eight
weeks after symptoms resolve. Asymptomatic prostatic inflammation can also elevate PSA
levels, but this diagnosis is made on biopsy and so cannot generally be used to defer screening
tests.

Prostate biopsy may elevate PSA levels by a median of 7.9 ng/mL within 4 to 24 hours
following the procedure. Levels will remain elevated for two to four weeks. Similarly, a
transurethral resection of the prostate (TURP) can elevate PSA levels by a median of 5.9 ng/mL.
Levels will remain elevated for a median time of approximately three weeks. A screening PSA
test should not be performed for at least six weeks following either of these procedures.

Acute urinary retention may elevate PSA levels, but the levels can be expected to decrease by 50
percent within one to two days following resolution. A screening PSA test should not be
performed for at least two weeks following an episode of acute urinary retention.

free-to-total PSA ratio:


complexed PSA increases in prostate cancer, decreasing the percentage of the free fraction
< 10% free PSA suggestive of cancer, >20% free suggests benign cause

PSA velocity:
change of >0.75 ng/mL/yr associated with increased risk of cancer

PSA density:
PSA divided by prostate volume as found on TRUS
>0.15 ng/mL/g associated with increased risk of cancer
72. A 55 y/o African American man with newly diagnosed Stage B prostate cancer undergoes
radical prostatectomy and is referred to you from surgical clinic for routine follow up. The
patient requests how often he should follow up with you and what tests he would need. Your best
response is:
A) You do not need any follow up because you had a local cancer that was completely resected
B) PSA need to be tested every six months for 5 years and thereafter, every year
C) Bone scan to evaluate metastasis is needed every year
D) Digital Rectal Exam every year to look for local recurrence
E) You need endocrine therapy before we proceed further

Ans. B

PSA need to be tested every six months for 5 years and thereafter, every year. The clinical course
of these patients was followed for 2-8 years.

Disease monitoring: serum PSA should fall to a low level following radiation therapy, high
intensity focused ultrasound and cryotherapy and should not rise on successive occasions. PSA
should remain undetectable following radical prostatectomy

Outcome prediction: in patients with metastatic disease receiving androgen suppression


therapy, failure to achieve a PSA nadir of <4.0 ng/mL seven months after initiation of therapy is
associated with a very poor prognosis (median survival: one yr).

73. A HIV positive man with CD4 count of 25 presented with complaints of painful swallowing
for the past 2 days. He never had similar issues in the past. The patient has been non-compliant
with his HAART medications and PCP prophylaxis. The patient also reports that three days ago,
he visited the ER for a cold sore on his lip and he was started on Acyclovir orally by the ER
physician. On physical examination he is febrile with temperature of 101F. There is no thrush or
pharyngeal erythema. You start the patient on empiric therapy with fluconozole and ask him
continue acyclovir. However, his symptoms continue to worsen after one week of empiric
therapy. A decision is made to perform endoscopy for further diagnosis . Clinically, the most
likely etiology of this patients odynophagia is :
A) Resistant Esophageal Candidiasis
B) HSV esophagitis
C) Pill induced Esophagitis
D) Gastro esophageal Reflux disease
E) Esophageal cancer
F) CMV esophagitis

Ans. F.

Esophagitis in HIV may be due to:


Candida (the most common cause is Candida): The patient with severe odynophagia, without
dysphagia or thrush, is more likely to have ulcerative esophagitis, such as herpes simplex virus,
rather than Candida. If pt has oral thrush, treat empirically with Fluconazole for 2 weeks, if
symptoms does not improve then endoscopy.

Herpes simplex virus: Patients usually present with odynophagia and dysphagia. Fever,
epigastric pain, nausea, vomiting, and heartburn are less frequent. Endoscopic Findings for
Herpes simplex => deep, small multiple lesion. Rx-Acyclovir.

Cytomegalovirus: CMV esophagitis presents with fever, odynophagia, and nausea, and is
occasionally accompanied by substernal burning pain. Endoscopy will show extensive, large,
shallow ulcers or erosions in the distal esophagus. Treatment is with gancyclovir.

or aphthous ulcers:

74. A 75 year-old man with history of hypertension presents to the emergency room with
complaints of shortness of breath and palpitations. His vital reveal a heart rate 142/min, blood
pressure 130/86, temperature 98.6 and oxygen saturation of 89% on room air. On auscultation,
there are no rhonchii or crepitations, the heart rate was irregular and rapid without any murmurs.
The patient is placed on oxygen by nasal cannula. An urgent EKG is obtained which reveals
rapid atrial fibrillation with no evidence of significant ST-T changes. The patient is started on
diltiazem. Chest x-ray is normal and a brain natriuretic peptide is 80ng/L. Electrolytes, TSH and
complete blood count are within normal limits. Cardiac enzymes are drawn. Arterial blood gases
reveal a pH of 7.48, po2 of 58, pco2 of 20 on room air ( Fio2 of 21%). The next step in
evaluating the etiology of his atrial fibrillation :
A) Cardiac catheterization
B) Spiral CT scan of the chest
C) Venous doppler of lower extremities
D) 2D Echocardiogram
E) D-Dimer

75. What is the most likely etiology of atrial fibrillation in Case 1?


A) Acute ST elevation MI
B) Acute pulmonary embolism
C) Pneumothorax
D) COPD exacerbation
E) Congestive heart failure

Ans. D and B

An Echocardiogram would be able to detect atrial fibrillation and the cardiologist reading it
should comment on the rhythm in the report. EKG is better for evaluating for atrial fibrillation
though. I know you already had an EKG and this would have detected it if you were in atrial
fibrillation at that time. Keep in mind though that EKG and Echocardiogram give a glance of the
heart rhythm for only a short period of time. Some people with atrial fibrillation can go in and
out of it, therefore not detecting it on EKG does not eliminate the possibility of atrial fibrillation
at other times. The best way to evaluate for that is a 24 to 48 heart monitor. Atrial fibrillation can
also be detected simply by feeling the pulse and it is classically associated with an "irregularly
irregular" rhythm.

D-dimer testing is most reliable for excluding pulmonary embolism in younger patients who
have no associated comorbidity or history of venous thromboembolism and whose symptoms are
of short duration. However, it is of questionable value in patients who are older than 80 years,
who are hospitalized, who have cancer, or who are pregnant, because nonspecific elevation of D-
dimer concentrations is common in such patients. D-dimer testing should not be used when the
clinical probability of pulmonary embolism is high, because the test has low negative predictive
value in such cases.

Chest radiographs are abnormal in most cases of pulmonary embolism, but the findings are
nonspecific. Common radiographic abnormalities include atelectasis, pleural effusion,
parenchymal opacities, and elevation of a hemidiaphragm. The classic radiographic findings of
pulmonary infarction include a wedge-shaped, pleura-based triangular opacity with an apex
pointing toward the hilus (Hampton hump) or decreased vascularity (Westermark sign). These
findings are suggestive of pulmonary embolism but are infrequently observed.

V/Q scanning of the lungs is an important modality for establishing the diagnosis of pulmonary
embolism. However, V/Q scanning should be used only when CT scanning is not available or if
the patient has a contraindication to CT scanning or intravenous contrast material.

A negative ultrasonographic scan does not rule out DVT, because many DVTs occur in areas that
are inaccessible to ultrasonographic examination. Before an ultrasonographic scan can be
considered negative, the entire deep venous system must be interrogated using centimeter-by-
centimeter compression testing of every vessel. In two thirds of patients with pulmonary
embolism, the site of DVT cannot be visualized with ultrasonography, so a negative duplex
ultrasonographic scan does not markedly reduce the likelihood of pulmonary embolism.

76. A 65 year-old man with history of recently diagnosed metastatic colon cancer being treated
with chemotherapy is admitted to the hospital with constipation and vomiting. His colon cancer
was diagnosed by colonoscopy 2 months ago when he presented with massive GI bleeding. At
this admission, patient is diagnosed with bowel obstruction secondary to descending colon
cancer and underwent a palliative left hemicolectomy to provide symptomatic relief. He has no
occult or gross GI bleeding at this time. On the sixth post-operative day, you are called by the
nurse because the patients blood pressure is 80/40. His heart rate is 82, respiratory rate 24 and
temperature of 100.6. The patient is given Normal saline bolus. A CXR is normal. EKG reveals a
prominent S wave in lead I, a Q wave and inverted T wave in lead III. Of note, a pre-operative
EKG was completely normal. First set of cardiac enzymes are negative. A bedside 2D echo
reveals global hypokinesis of the right ventricle. A repeat blood pressure obtained after normal
saline bolus is still low at 70/40. The most likely etiology of the shock in this patient is :
A) Hypovolemia
B) Septic shock
C) Acute myocardial infarction leading to cardiogenic shock
D) Acute pulmonary embolism
E) Tension Pneumothorax

77. Most important next step in treating this patients shock?


A) Continued fluid boluses
B) Antibiotics and pressor support with dopamine
C) Intra-aortic balloon counter-pulsation followed by urgent cardiac catheterization.
D) Anticoagulation with heparin
E) Tissue plasminogen activator ( tpA)
F) Embolectomy
G) Chest tube placement.
H) Inferior vena cava filter

78. The patient was appropriately treated. The discharge recommendations should include :
A) Inferior venacava filter
B) Life-long low-molecular weight heparin
C) Life-long coumadin
D) Hypercoagulability testing
E) Compression stockings

Ans. D, F, B

The currently accepted indications for thrombolytic therapy include hemodynamic instability or
right ventricular dysfunction demonstrated on ECHO.
In patients with submassive acute pulmonary embolism, either catheter embolectomy or surgical
embolectomy may be considered if they have clinical evidence of an adverse prognosis (ie, new
hemodynamic instability, worsening respiratory failure, severe right ventricular dysfunction, or
major myocardial necrosis). These interventions are not recommended for patients with low-risk
or submassive acute pulmonary embolism who have minor right ventricular dysfunction, minor
myocardial necrosis, and no clinical worsening
The current ACCP guidelines recommend that all patients with unprovoked pulmonary
embolism should undergo a risk-to-benefit evaluation to determine if long-term therapy is
needed (grade 1C). Long-term treatment is recommended for these patients who do not have risk
factors for bleeding and in whom accurate anticoagulant monitoring is possible (grade 1A).
Patients who have pulmonary embolism and preexisting irreversible risk factors, such as
deficiency of antithrombin III, protein S and C, factor V Leiden mutation, or the presence of
antiphospholipid antibodies, should be placed on long-term anticoagulation.
Anticoagulation for prophylaxis against recurrent VTE should generally be avoided if there is
active bleeding and/or there are other contraindications to anticoagulant use (eg, recent surgery,
pre-existing bleeding diathesis, platelet count <50,000/microL, coagulopathy).

2008 ACCP Guidelines: initiate treatment with LMW heparin for the first three to six months.
Because the risk of recurrent VTE is unacceptably high in patients with active cancer who stop
anticoagulant therapy, it is suggested that subsequent treatment with either LMW heparin or
warfarin be given indefinitely or until the cancer is resolved.
For patients with malignancy, a reasonable quality of life and life expectancy, and venous
thromboembolism, we suggest initial treatment with LMW heparin over the use of oral
anticoagulants
For selected high risk general surgery patients, including those who have undergone major
cancer surgery or have previously had VTE, we suggest that continuing thromboprophylaxis
after hospitalization with LMW heparin for up to 28 days be considered.

79. A 38-year old female on birth control pills, has suddenly become extremely short of breath.
Someone has seen her collapse and called 911. She was diaphoretic and complained of severe
chest pain before she collapsed. She is now in the ER/ED and you have been asked to evaluate
her. Her old records show that she is a cocaine abuser and was admitted for subarachnoid
hemorrhage 6 weeks ago from which she completely recovered. Clinical findings revealed Vitals
: B.P 65/ palpable, R.R 45. Pulse 140, Tm: 99.2 F. Chest exam revealed decreased breath sounds
in right lower lobe and distant heart sounds. Pulse oximetry revealed 88%. EKG showed sinus
tachycardia with a q wave and T wave inversion in lead III. 2D echo showed global hypokinesis
of the Right Ventricle and pulmonary hypertension. You started her on Intravenos fluids and her
blood pressure has slightly improved to 66/30. Your next step in management ?
A) Transfer to cath lab and notify the interventional cardiologist stat
B) Intra aortic balloon counterpulsation
C) Thrombolytic therapy
D) Surgical Embolectomy and Inferior vena cava filter
E) Obtain cardiothoracic surgery consult for subxiphoid window

Ans. D.

This patient is an absolute contraindication to tpa:


History of hemorrhagic stroke
Active intracranial neoplasm
Recent (< 2 months) intracranial surgery or trauma
Active or recent internal bleeding in prior 6 months: this patient had SAH which is an internal
bleeding, 6 weeks back.
80. A 55 y/o male with history of lung cancer recently had a portable cath placed in the SVC.
However, one week later he presents to your office with increasing swelling of this face, neck
and upper extremities and increasing jugulovenos distension. You diagnose SVC syndrome and
your suspicion is confirmed by an SVC venogram. You send the patient to interventional
radiologist for SVC dilatation. In the radiology OR patient suddenly becomes unresponsive and
hypotensive. His heart rate was 140 and B.P 78/40. He responds well to IV fluids but tachycardia
persists. He is then transferred to ICU. You pay him a visit in the ICU and examine him. At the
time of your exam he suddenly becomes unresponsive again and his blood pressure drops to
80/40. You restart IV fluids. Chest is clear to auscultation. Heart sounds are audible and normal.
He has increased JVD but wife reports he has had this for past one week. The EKG is shown.

The most important test that will best help you in diagnosis:
A) 2D ECHO
B) Cardiac enzymes
C) Chest X-ray
D) Electrocardiogram
E) Blood cultures

81. Next Step in management of this patient :


A) Tube thoracostomy
B) Pericardiocentesis
C) Intraaortic balloon counterpulsation
D) Percutaneous transluminal coronary angioplasty
E) IV Antibiotics
Ans. A and B

POSSIBLE EKG FINDINGS IN CARDIAC TAMPONADE:


sinus tachycardia
low voltage
electrical alternans

ELECTRICAL ALTERNANS:
beat to beat alternation in QRS appearance
related to back & forth swinging motion of the heart in the pericardial fluid.

82. A 35 y/o woman was admitted for Thrombotic thrombocytopenic purpura and was receiving
plasmapheresis. She became agitated one evening and pulled off her right jugular catheter which
has been her plasmapheresis access. Arrangements are being made to obtain an alternate access
for plasmapheresis. On examination her B.P is 60/32., HR: 120, Tm: 102, RR 24. Physical
examination revealed a remnant of jugular catheter thats still bleeding. You remove the catheter
immediately. Chest is clear to auscultation bilaterally. The patient is now crashing and blood
pressure is no longer recordable but palpable at 60. Next immediate step in managing this patient
is :
A) Pericardiocentesis
B) Start IV fluids
C) Put patient in trendelenberg position
D) Needle thoracentesis
E) Portable chest x-ray
F) 2D echo

Ans. C

The most common problem is a drop in blood pressure, which can be experienced as faintness,
dizziness, blurred vision, coldness, sweating or abdominal cramps. A drop in blood pressure is
remedied by lowering the patients head, raising the legs and giving intravenous fluid.

83. A 35-year-old man with a 10-year history of type 1 diabetes mellitus is evaluated because of
recent onset of morning hyperglycemia. His home blood sugar logs over the last 10 days have
consistently been showing elevated sugars in the range of 220 to 300 mg/dL in the early morning
( pre-breakfast). He has also experienced nightmares recently. He has been compliant with his
diet instructions and has not changed his dinner potions recently. He takes mixed insulin
regimen : NPH/Regular insulin 70/30 mix at 30 units in the AM before breakfast and 20 units
in PM 30 minutes before dinner. Which of the following best explains this patients morning
hyperglycemia?
( A ) Diabetic nephropathy
( B ) Under treatment with insulin
( C ) Overtreatment with insulin
( D ) Insulinoma
(E) Non compliance with Insulin

84. The best diagnostic study in establishing the diagnosis in this patient :
A) C-Peptide level
B) Urine 24 hour catecholamines
C) Check pre-dinner blood sugar level
D) Check blood sugar level 30 minutes post dinner
E) Check blood sugar level between 2:00 AM and 3:00 AM

85. Next best step in managing this patients pre-breakfast hyperglycemia :


A) Increase pre-breakfast regular insulin dosage in AM
B) Increase pre-dinner regular insulin dose
C) Reduce pre-dinner NPH insulin dose
D) Decrease the carbohydrate consumption in the night
E) Discontinue Pre-dinner insulin

Ans. C,E,C

Dawn phenomenon. The dawn phenomenon is the end result of a combination of natural body
changes that occur during the sleep cycle and can be explained as follows. Between 3:00 a.m.
and 8:00 a.m., your body starts to increase the amounts of counter-regulatory hormones (growth
hormone, cortisol, and catecholamines). These hormones work against insulins action to drop
blood sugars. The increased release of these hormones, at a time when bedtime insulin is wearing
out, results in an increase in blood sugars. These combined events cause your bodys blood sugar
levels to rise in the morning.

Somogyi effect. Named after the doctor who first wrote about it, this condition is also called
rebound hyperglycemia. Although the cascade of events and end result high blood sugar
levels in the morning is the same as in the dawn phenomenon, the cause is more man-made
(a result of poor diabetes management) in the Somogyi effect. The term refers to pattern of high
morning sugars preceded by an episode of hypoglycemia (with no symptoms). Your blood sugar
may drop too low in the middle of the night, so your body counters by releasing hormones to
raise the sugar levels. This could happen if you took too much insulin earlier or if you did not
have enough of a bedtime snack.

Somogyi Effect = rebound hyperglycemia because the body was in hypoglycemia (too much
insulin).so the stress hormones epinephrine and glucagone will kick in and there u have
hyperglycemia in the morningso the PT is hypoglycemic at 3 AM and he is
hyperglycemic at 7-8 AM for example.
Mx = give NPH at bed time instead at dinner time and give LOWER dose.
Dawn Phenomenon:
- high sugar at 3 a.m. and also at 7 a.m. b/c theres increased insulin resistance between 5-8 a.m.
caused by growth hormone
Mx = INCREASE dose of NPH at dinner time.

So if we are not sure if the morning hyperglycemia is due to the Dawn or Somogyi effect, what
do we do? how do we modify the Insulin dose?
-Answer is to DECREASE the insulin at night.
Then check am glucose:
-If the glucose is increased in the am, it was Dawn Phenomenon. (not enough Insulin or not the
right type at pm)..
-If the glucose is normal or decreased in am, it was Somogyi. (rebound hyperglycemia due to the
bodys counter regulatory hormones).
(NEVER INCREASE the Insulin dose, because if the patient has Somogyi, we could end up
putting him in a coma)
Once we have determined which one it is we can modify the Insulin accordingly.
For Dawn effect we can break down the pm dose to regular (rapid acting) in the pm and NPH
(intermediate acting) right before bedtime.

86. A patient had a closed fist injury at a bar while trying to punch his friend who he later learnt
was HIV positive. The patient tells you that there was only an abrasion on his hand and all he
noted on his hand was his friends saliva. He is very concerned. What is your next step?
A. Give HIV prophylaxis with HAART
B. Clean and debride the wound and reassure that no need for prophylaxis
C. Call surgical consult
D. Close the wound with sutures
E. Check for HIV antibody

Ans. B. Clean and debride the wound and reassure that no need for prophylaxis

87. A 42 year old african-american man is admitted to hospital with acute seizures. Seizures
were appropriately controlled in the ER and the patient currently, in post-ictal confusion. He is
unable to give further history. However, a review of the old records reveal that the patient has
history significant of Chronic HIV infection. He also has a history of IV drug use. As per his
sister, the patient has been compliant with Highly active anti-retroviral therapy and prophylactic
medications for Pneumocystis jiroveci and Mycobacterium Avium Complex for the past one
year. His recent CD4 count 1 month ago was 45. On physical examination, he is afebrile with a
blood pressure of 120/60. He is confused. Reflexes are intact. Electrolytes and CBC are within
normal limits. Urine drug screen is negative. A non-contrast CT scan did not reveal any bleed. A
CT scan with IV contrast reveals a 4 cm ring enhancing lesion in left cerberal hemisphere. A
subsequent MRI brain confirmed the findings on the CT. There is no mass effect. Next step in
approaching this patient ?
A. Stereotactic Brain Biopsy
B. Start emperic Toxoplasma therapy.
C. Obtain Toxoplasma Serology ( IgM and IgG)
D. PCR for Papova Virus JC
E. Emperic therapy for CNS tuberculosis.

Ans. C

Now patient has CD4 count 45 and he is on TMP-SMX and antiretroviral medication. TMP-
SMX is also the prophylaxis for Toxoplasmosis. Although pt on TMP-SMX there are higher
chance of getting Toxoplasmosis infection in such low CD4 count. So, first do Toxoplasma
serology, if positive start therapy. If negative then do PCR for Parvo virus JC.

According to MTB:
The patient has a ring-enhancing lesion. In developed countries such as the US, the most
common causes include toxoplasma and CNS lymphoma. About 10% of general population has
toxoplasma antibody; so a positive serology is not confirmatory. So, empiric treatment with
sulfadiazine & pyrimethamine. After 10-14 days, a repeat CT/MRI is done. If the size of the
lesion is reduced, then continue initial treatment; otherwise treat for CNS lymphoma.

88. A 75 year old woman is sent from the nursing home for evaluation of fever and altered
mental status. The patients past medical history is significant for moderate Alzheimers
dementia. On examination, she is confused. Her vitals reveal Temperature of 102F, Blood
pressure 80/60 and a HR of 102/min. Chest and cardiovascular examination is benign. On
abdominal examination, the patient moans upon palpation of right upper quadrant. Cholecystitis
is suspected and ultrasound is obtained that reveals very distended gall bladder with
pericholecystic fluid, a normal caliber common bile duct and a gall stone in the cystic duct. The
patient is started on IV Normal saline and broad spectrum antibiotics. Her blood pressure despite
initial hydration is still 80/40. She is started on Norepinephrine drip. The next most important
step in managing this patient ?
A. Urgent Cholecystectomy
B. Endoscopic Retrograde Cholangiopancreatography
C. Percutaneous Cholecystostomy
D. 2D echocardiogram
E. Exploratory Laporotomy

Ans. C

Mild Sx observe if stable, early lap chole if unstable


Moderate Sx early lap chole if stable, percutaneous gb drainage if unstable
Severe Sx including organ dysfunction (cardio-hypotension, neuro-decreased consciousness,
pulm-respiratory distress, renal-Cr > 2, hepatic-PT/INR abnormal, hem-thrombocytopenia)
Intravenous fluid and IV antibiotics, percutaneous drainage, if unstable then early chole
In all cases if percutaneous drainage is done, follow up with elective lap chole.
http://www.med.upenn.edu/gastro/documents/GastroClinNAcholecystitisandcholangitis.pdf

89. A 61-year-old man is admitted because of altered mental status. On physical examination, he
is afebrile. Laboratory studies show sodium 136 mmol/L, potassium 4.4 mmol/L, chloride 108
mmol/L, CO2 30 mmol/L, glucose 78 mg/dL, urea nitrogen 49 mg/dL, calcium 13.8gm/dL,
creatinine 5 mg/dL, hemoglobin 8.9gm/dL, total protein 8.3 g/dL, albumin 3.7 g/dL, alkaline
phosphatase 116 U/L, AST 45 U/L, ALT 22 U/L, and total bilirubin 1.2 mg/dL. The patient is
started on aggressive Intravenos hydration. Which of the following may be typically seen with
this patients disease?
A) Hypercellular Bone marrow with many blasts
B) Serum protein electrophoresis with polyclonal hypergammaglobulinemia
C) An increase in all immunoglobulins ( IgA, IgG and IgM)
D) A negative anion gap
E) Increased Alkaline Phosphatase
F) A positive bone scan

Ans. D

A negative anion gap - Due to large amount of negative protein charge, since this is multiple
myeloma with hypercalcemia and renal failure.

Common causes of an elevated Anion Gap include:


Ketoacid overproduction due to fat metabolism (diabetes, alcohol, starvation)
Lactic Acid overproduction due to respiratory failure (the tissue has inadequate oxygen),
genetic defects of enzymes of carbohydrate metabolism, nutritional deficiencies that
impair the body's ability to metabolize lactic acid (B vitamins, especially vitamin B1)
Inability to excrete acids (sulfate and phosphate) due to renal disease (usually with an
elevated BUN and creatinine).
Dehydration.
Medications such as salicylates causing a metabolic block.
Toxins such as ethylene glycol, methanol, paraldehyde, propyl alcohol

The Anion Gap is decreased by free radical pathology due to overproduction of alkaloids. Other
causes that have been reported associated with a reduced anion gap are
Alkalosis for any reason
Hyperchloremic acidosis (excess chloride)
Multiple Myeloma
Hyponatremia (low blood sodium level; see appropriate lab finding webpage)
Hypoalbuminemia (see albumin; can increase the amount of free blood calcium)
Bromide Ingestion (displaces chloride)
Uncalculated blood cations (calcium, magnesium)
Lithium toxicity (can be due to effects on sodium)
Primary hypothyroidism
Kidney disease (due to the loss of the cations sodium and or potassium)
Polymyxin B

90. A 57 year old hispanic woman is admitted to the hospital with right leg cellulitis. The patient
is started on intravenous antibiotics. She is afebrile. Physical examination reveals erythema and
tenderness in right lower extremity. Venous Doppler is negative for DVT. Her past medical
history is significant for chronic alcoholism and liver cirrhosis. Laboratory investigations reveal
a WBC count of 1700/l with absolute neutrophil count of 800, Hemoglobin of 10.2gm% and
Platelet count of 52,000/l. Peripheral smear reveal reduced platelets and no evidence of any
abnormal cells. B12 and folic acid level as well as Iron studies are within normal limits. Blood
cultures are negative at Day 1. HIV serology, Hepatitis B and Hepatitis C are negative.
Antinuclear antibodies are negative. Reticulocyte count is 4% and Lactic Dehydrogenase is 170
*( normal). Haptoglobin is 220( normal). A hematology evaluation is obtained and a bone
marrow biopsy performed to evaluate her pancytopenia. BM biopsy reveals hypercellualar bone
marrow with no dysplatic features. The most important next step in evaluating this patients
pancytopenia?
A) CT scan of the abdomen
B) Parvo virus B19 Antibodies
C) CT scan of the chest to rule out Thymoma
D) Methyl malonic acid level
E) Flow cytometry for CD58 and CD59

91. Most likely cause of this patients Pancytopenia?


A) Hypersplenism
B) Parvovirus B19
C) Aplastic Anemia
D) Sub clinical Vitamin B12 deficiency
E) Acute Leukemia
F) Paroxysmal Nocturnal Hemoglobinuria

Ans. A and A

Increased reticulocyte count: hemolysis +nt


Normal haptoglobin: extravascular hemolysis
hyperspleenism causes extravascular hemolysis

It's not B/B(parvovirus related) because: The dramatic decrease or absence of measurable
reticulocytes is a hallmark laboratory finding in persons with B19 infection. A bone marrow
biopsy is usually remarkable for severe aplasia. In this case retic is 4% and BM is hypercellular.
92. 67 year old African American man presents to your office for regular check up. He has no
significant past medical history. He underwent a screening colonoscopy 5 years ago that was
normal . He underwent a PSA testing 3 months ago and it was 2.5. The patient denies any
complaints. He denies any fatigue or recent weight changes. He denies alcohol use or smoking.
He does not use any medications at home. Physical examination is normal. Routine laboratory
investigations reveal a WBC count of 4200/l, Hemoglobin of 9.6gm/dL , Mean Corpuscular
Volume of 106, Reticulocyte count of 1% and Platelet count of 152,000/l. Peripheral smear
reveal macrocytosis and hyposegmented neutrophils. B12 level 540 pmol/L, TSH 2.0 ( N = 0.3 -
4.2) miU/L and Folic acid level is within normal limits. Lactic Dehydrogenase is 170 *(
normal). Haptoglobin is 220( normal). Most likely etiology of this patients anemia?
A) Iron deficiency
B) Subclinical Vitamin B12 Deficiency
C) Subclinical Hypothyroidism
D) Hemolytic Anemia
E) Myelodysplatic Syndrome

93. The next best step in obtaining the diagnosis :


A) Serum Ferritin
B) Methylmalonic Acid Level
C) Free T4 and T3 level
D) G6PD level
E) Bone marrow biopsy

Ans. E and E

E. Myelodysplastic Syndrome
>Elderly
>Special Neutrophil with 2 lobes called Pelger -Huet Cell
>RBC->MCV
E. Bone Marrow Biopsy

heterogeneous group of malignant stem cell disorders characterized by dysplastic and ineffective
blood cell production resulting in peripheral cytopenias.

Pathophysiology
-disordered maturation - ineffective hematopoiesis despite presence of adequate numbers of
progenitor cells in bone marrow (usually hypercellular)
-intramedullary apoptosis - programmed cell death within bone marrow
-both processes lead to reduced mature cells in periphery
-30-40% develop AML
Risk Factors
elderly, post-chemotherapy, benzene or radiation exposure
occurs in 601100,000 in patients >60 yrs old
Clinical Features
insidious onset: associated with those of pancytopenia
infections and bleeding out of proportion with peripheral blood counts
Investigations
diagnosed by:
anemia thrombocytopenia neutropenia
CBC and peripheral blood film
RBC: usually macrocytic with oval shaped red cells (macro-ovalocytes), decreased
reticulocyte count
WBC: decreased granulocytes and abnormal morphology (e.g. bilobed or unsegmented
nuclei = Pelger abnormality)
platelets: thrombocytopenia, abnormalities of size and cytoplasm (e.g. giant hypogranular
platelets)
bone marrow aspirate and biopsy with cytogenetic analysis required for definitive diagnosis
bone marrow - dysplastic and often normocellular/hypercellular
cytogenetics - partial or total loss of chromosomes 5, 7, Y, or trisomy 8

Treatment
low risk of transformation to acute leukemia (IPSS low and intermediate 1)
supportive care: RBC and platelet transfusion
erythropoietin SC weekly may be effective in reducing transfusion requirements
high risk of transformation to acute leukemia (IPSS intermediate 2 and high)
supportive care
stem cell transplantation if age <65 yrs
epigenetic therapy: DNA methyltransferase inhibitors (e.g. 5-Azacytidine), histone
deacetylase inhibitors

94. A 62 year old man presents with right leg pain and swelling. The pain and swelling started 2
days ago and has been increasing. He denies any fever, chest pain or shortness of breath. He has
no significant past medical history. There is no family history of clots. His last visit to a doctors
office was 30 years ago. He denies any weight loss or dyspepsia or melena or rectal bleeding.
Physical examination reveals ankle tenderness and mild swelling of the right lower extremity up
until his mid thigh. Laboratory investigations reveal normal complete blood count with
prothrombin time and partial thromboplastin time within normal limits. A venous doppler reveals
a common femoral to popliteal DVT in his right lower extremity. The patient is started on Low
Molecular Weight heparin. The next important step in evaluating the hypercoaguilabilty in this
patient?
A) Protein C level
B) Protein S level
C) Anti thrombin III level
D) Screening Colonoscopy
E) CT scan of the abdomen and Pelvis.
F) Heparin Induced Platelet Antibodies

Ans. D

Hypercoagulable State:
1.Inherited
a. FH(+)
b. <50 yrs
c. Recurrent

2.Acquired
GI cancer induce DVT. This patient did not get a screening colonoscopy as it was supposed to
have at 60 y/o.

95. A 62 year old man with history of DM Type II and Coronary Artery Disease presents to the
Emergency room with right leg pain and swelling. The pain and swelling started 2 days ago and
has been increasing. He denies any fever, chest pain or shortness of breath. He was recently
admitted to the hospital 10 days ago for Non ST elevation Myocardial infarction. The patient was
treated at that time with medical management that included Heparin, Clopidogrel, Aspirin and
Beta blockers. The patient was discharged with instructions to continue aspirin, clopidogrel and
metoprolol. At the time of discharge. his labs were all within normal limits. He says he has an
appointment with his cardiologists office next week for further work-up. He denies any
bleeding. Physical examination reveals ankle tenderness and mild swelling of the right lower
extremity up until his mid thigh. Laboratory investigations reveal a WBC of 5100, HGB 14.2 and
a platelet count of 40k/l ( N = 160 to 400k/l. Prothrombin time and partial thromboplastin
time within normal limits. A venous doppler reveals a common femoral to popliteal DVT in his
right lower extremity. Next step in managing this patient ?
A) Start Low Molecular Weight Heparin
B) Start Warfarin
C) Place Inferior Vena Cava Filter due to bleeding risk
D) Start Lepirudin
E) Platelet Transfusion

B. Which of the following is most likely to establish the diagnosis in this patient?
A) Lupus Anticoagulant Profile
B) Anti Platelet Factor 4/ Heparin antibodies
C) Factor V leiden mutation
D) Prothrombin gene mutation
E) Peripheral Blood Smear

Ans. D and B
Heparin induced thrombocytopenia(HIT). In HIT, Thrombosis is the most common
Manifestation Venous Thrombosis 3 times more common than Arterial Thrombosis. Mx - Direct
Thrombin Inhibitors-Argatroban / Lepirudin.

Standard unfractionated heparin can cause an antibody-mediated (Type II) thrombocytopenia in


2-3% of individuals who receive this drug for longer than 7 days. When the platelet count falls
precipitously,STOP heparin. Do not start low-molecular-weight heparin because it will cross-
react with the antibody 90% of the time. If a rapidly acting drug is needed, substitute a direct
thrombin inhibitor, either lepirudin (Refludan) or agatroban.

Dosing lepirudin:
Stop unfractionated heparin
Do not substitute LMW-heparin
Hold warfarin

IV infusion (for rapid therapeutic anticoagulation).

Loading dose: 0.4 mg/kg bolus i.v..


Maintenance: 0.15 mg/kg/hr i.v. (up to 110 kg body weight
Adjust maintenance dose to maintain activated partial thromboplastin time (APTT) at 1.5 to
2.5 times the laboratory's mean normal value.

Dosing agatroban:

Stop unfractionated heparin


Do not substitute LMW-heparin
Hold warfarin

IV infusion (for rapid therapeutic anticoagulation)

Loading dose: no loading dose


Maintenance: 2 ug/kg/min
Adjust mainenance dose to maintain APTT at 1.5 to 2.5 times laboratory's mean normal value

Give lepirudin or argatrobn for at least 3 days while holding warfarin. When the platelet count
has recovered above 100,000/uL, give warfarin at 5 mg/day and adjust dose by INR.
96. A 65 year old man with history of DM type II presents to your office with complaints of
chest pain that he has been having lately. He denies any pain now. He says his chest pain is more
left sided and about 5/10 in intensity and it appears after walking about 2 blocks. The pain
disappears after resting for a while. He has been having these chest pain episodes for the past 3
months. He also reports severe cramps and bilateral leg pain upon walking a block that is also
relieved by rest. He denies any shortness of breath. He has smoked about 1 pack per day for the
past 40 years. He denies any cough. His only medications are Glyburide and Metformin. An
EKG is obtained in the office and it reveals non-specific ST segment changes. Next important
step in managing this patient:
A) CT angiogram of Chest
B) Exercise Treadmill Stress test
C) Obtain Cardiac Enzymes
D) Dipyridamole Stress test
E) Start Calcium Channel Blocker

Ans. D

A waxing and waning chest pain for many months, chest pain that appear only on exertion and
relieved by rest and chest pain is not present now; it's clearly shows that this pt is suffering with
stable angina. He is not a good candidate for typical exercise treadmill test because he has a sx of
intermittent claudication. She should go for Dipyridamole stress test. COPD is the
contraindication for Dip Stress test.

97. A 68 year old man with history of DM type II, Hypertension and severe osteoarthritis
presents to your office with complaints of chest pain on exertion for past 3 months. He denies
any pain now. He smokes about one pack cigarettes per day. His only medications are
Glyburide, Metprolol, Enalapril and Metformin. An EKG is obtained in the office and it reveals
non-specific ST segment changes. The patient is scheduled for Dipyridamole stress test . The
patient should be advised regarding which of the following:
A) Avoid Aspirin or NSAID for 24 hours prior to testing
B) Stop Metprolol 24 hours prior to testing
C) Avoid Coffee or Caffeine for 24 hours prior to testing
D) Avoid smoking for one week prior to testing
E) Start inhaled Albuterol two days prior to testing

Ans. C

Caffeine blocks Adenosine Receptors & blunt the vasodilators properties. Xanthine derivatives
such as caffeine may abolish the coronary vasodilation induced by IV Dipyridamole
administration. This could lead to a false negative result. Xanthine derivatives should be avoided
24 hours before myocardial imaging with IV Dypridamole. Consumption of coffee or xanthine is
a contraindication for doing dypridamole stress test.
98. A 68 year old man with history of DM type II, Hypertension and severe osteoarthritis
presents to your office with complaints of chest pain on exertion for past 3 months. He denies
any pain now. His only medications are Glyburide, Metprolol, Enalapril and Metformin. An
EKG is obtained in the office and it reveals non-specific ST segment changes. While undergoing
the test, the patient develops severe chest pain and headache. EKG monitor reveals > 2mm ST
depressions in the anterior leads. The technician immediately terminates the dipyridamole
infusion. After 2 minutes of cessation of infusion, the patient continues to have chest pain. His
blood pressure is 88/68 mm hg. The next step in managing this patient :
A) Order Cardiac enzymes
B) Start Heparin infusion
C) Administer Intravenos Aminophylline
D) Reassure the patient that symptoms will improve in few minutes
E) Urgent Cardiac Catheterization

Ans. C

Dipyridamole exhibits a coronary steal phenomenon where it steals the blood from abnormal/
diseased arteries and moves it to normal ones. In this process, it exacerbates the ischemia which
will appear as a defect on the nuclear component of this stress test in a patient with ishemic heart
disease. Side effects of dipyridamole are bronchoconstriction, headache, chest pain. If these side
effects occur during this stress test, First step is stop the infusion and observe. If after 2 minutes,
the patient still having adverse symptoms next step is to reverse the effects of Dipyridamole
with Aminophylline. Aminophylline is a Xanthine oxidase inhibitor and serves to reverse the
effects of dipyridamole.

A. is incorrect. The timing and the symptoms are consistent with adverse reaction to
Dipyridamole infusion. The adverse events with Dipyridamole Stress can be successfully
reversed by Aminophylline.
B. is incorrect. The timing and the symptoms are consistent with adverse reaction to
Dipyridamole infusion. The adverse events with Dipyridamole Stress can be successfully
reversed by Aminophylline. If the patient is suspected to have an Acute MI, heparin infusion
needs to be started. The EKG changes are non-specific and are not consistent with ST elevation
MI.
D. Is incorrect. The patient is having severe symptoms even after 2 minutes of cessation of
dipyridamole infusion and observation. Reassurance, stopping the infusion and observation are
the initial steps however, if symptoms do not improve after 2 minutes, an antidote has to be
given.
E. Is incorrect. The timing and the symptoms are consistent with adverse reaction to
Dipyridamole infusion. The adverse events with Dipyridamole Stress can be successfully
reversed by Aminophylline. If the patient is suspected to have an ST elevation MI, an urgent
cardiac catheterization should be performed. The EKG changes are non-specific and are not
consistent with ST elevation MI.
99. A 65 y/o man with presents to your office with complaints of exertional chest pain for the
past 4 weeks. The chest pain is usually left sided, occurs on walking about three blocks and goes
away with rest. He has developed a habit of taking rest when the chest pain comes and he did not
think it needed medical attention until his friend told him yesterday that it might be a symptom
of heart disease. He is concerned and requests your recommendation. He denies any chest pain
now. He also reports no change in quality or intensity of his chest pain. His past medical history
is significant for pacemaker insertion for a symptomatic second degree heart block,
Hypertension, and Smoking . His medications include lisinopril, atenolol and hydrochlorthiazide.
Physical examination is benign. An EKG is obtained which reveals pacemaker rhythm with
secondary ST-T changes. The next best step in establishing the diagnosis and prognosis in this
patient is :
A) 2 D -Echocadiogram
B) Exercise Stress Test ( Treadmill Stress Test)
C) Dobutamine Stress Echocardiogram
D) Persantin Stress Test
E) Cardiac Catheterization

Ans. D

pre-test probability is very high here ( Typical Angina, risk factors). The reason stress test is
recommended even when high pre-test probability is present is because stress test provides both
diagnostic and prognostic information prior to cath and it will also determine the extent of
benefit the patient may achieve from re-vascularization.
But one should realize that when pre-test probability is very high, a negative test could mean a
false positive so, a negative stress test in a case like this should be considered as false negative
and eventually, he will still need Cath.
Once you know Stress test is the first step, next step is to determine the type of stress test. The
best one for his condition is Persantin stress test.

Patient with pacemaker rhythms, LBBB and severe LVH, will have baseline EKG changes that
may make EKG component of the stress test difficult. However, in these conditions, tachycardic
tress (exercise stress test) may also produce false positive defects on nuclear imaging. So, the
solution is to use a different type of stress test such as vasodilator stress (persantin). Moreover,
this patient was also on beta blocker, which makes it difficult to achieve target heart rate during
the exercise stress test. Dobutamine echocardiogram is revered for patient with bronchospasm or
heart blocks (in condition where persantin is contraindication). Dobutamine stress echo also
looks at septal wall motion during stress. In LBBB, septal wall movements cannot be relied
upon.

If stress test revealed ischemia, cardiac catheterization should be performed.

100. A 52 year-old man presents to your office with complaints of exertional chest pain for the
past 4 weeks. The chest pain is usually left sided, occurs on walking about three blocks and goes
away with rest. He has developed a habit of taking rest when the chest pain comes and he did not
think it needed medical attention until his friend told him yesterday that it might be a symptom
of heart disease. He is concerned and requests your recommendation. He denies any chest pain or
shortness of breath now. He also reports no change in quality or intensity of his chest pain. His
past medical history is significant for Hypertension and Smoking . His medications include
lisinopril and hydrochlorthiazide. Physical examination is benign. The next best step in
establishing the diagnosis in this patient is :
A) Electrocardiogram
B) 2 D -Echocadiogram
C) Exercise EKG Stress Test
D) Persantin Stress Test
E) Cardiac Catheterization

Ans. C

Exercise EKG stress test is the preferred test in evaluating patients who can walk and are
presenting with symptoms of typical CAD. The patient give a hx of typical exertional chest pain
that improves with rest. This highly suggestive of stable angina. The pt has no chest pain now. A
resting EKG is useful to show if there are any baseline changes but it will not establish the dx.
An EKG should be obtained during the stress test to establish the dx. In patients who can walk,
exercise test is the preferred modality since one can also assess the symptoms, EKG changes and
functional capacity. In pt who cannot walk persantin (diprydamole) stress test is preferred.
Dobutamine echocardiogram is revered for patient who cannot walk and have contraindication to
diprydamole stress test.

101. A 65 y/o man with presents to your office with complaints of exertional chest pain for the
past 4 weeks. The chest pain is usually left sided, occurs on walking about one block and goes
away with rest. He denies any chest pain now. He also reports no change in quality or intensity
of his chest pain He also reports having been diagnosed with peripheral arterial disease about 2
months ago for which he was advised exercise therapy. He does experience leg pain on walking
about one block which also improves with rest. His past medical history is significant for
moderate COPD, Hypertension and a hernia repair about 3 years ago. His medications include
lisinopril, hydrochlorthiazide and tiotropium inhaler. Physical examination is benign. The next
best step in establishing the diagnosis in this patient is :
A) 2 D -Echocadiogram
B) Exercise Stress Test ( Treadmill Stress Test)
C) Dobutamine Stress Echocardiogram
D) Persantin Stress Test
E) Cardiac Catheterization

Ans. C

Patient cannot walk due to peripheral vascular disease and has COPD. Dobutamine stress echo is
preferred for this pt.
102. A 65 year old man presents to your office with increasing abdominal distension and
bilateral leg swelling. He reports his symptoms started 3 months ago and progressively
worsening. He smokes about one pack cigarettes per day and drinks one pint vodka every day.
His last drink was 1 day ago. On examination, he is afebrile and he has abdominal distension and
ascites without any tenderness on palpation. Lab studies show WBC 8k/l, Hemoglobin of 10.2
gm%, Platelets 90k/l, Total protein of 6.4, Albumin 2.2, SGOT 300, SGPT 130, Total Bilirubin
4.2 , Direct Bilirubin 3.3, Prothrombin time of 19 seconds and Creatinine 2.2. The patient
undergoes diagnostic paracentesis which reveals a total protein of 1.4, albumin of 0.6, WBC
count of 400 with polymorphonuclear neutrophils of 100cells/ml. Bacterial cultures are pending.
The most important step in managing this patient is :
A) Intravenos Ceftriaxone
B) Intravenos Corticosteroids
C) Intravenos Albumin Infusion
D) Trans-jugular Intrahepatic Porto-systemic Shunt (TIPS)
E) Arrange for Liver Tranplant
F) Furosemide and Spironolactone

103. The most important factor that should be considered in determining the etiology of this
patients Ascites:
A) Fluid WBC
B) Fluid Albumin
C) Fluid Total protein
D) Serum Ascites- Albumin Gradient
E) Serum Albumin and Prothrombin time

Ans. F and D

There is no criteria for Spontaneous Bacterial Peritonitis (SBP). If this patient is suffering with
acute alcoholic hepatitis then he should be tx with IV corticosteroids. This patient is having
alcoholic liver cirrhosis and patient is actively drinking put him not a candidate for liver
transplant. Albumin infusion only indicated in hypotension or during paracentesis in ascites.

Serum - Ascites - Albumin gradient differentiates between transudate and exudate.

104. A 65 year old man presents to your office with increasing abdominal distension and
bilateral leg swelling. He is accompanied by his daughter. He reports his symptoms started 3
months ago and progressively worsening. He smokes about one pack cigarettes per day and
drinks one pint vodka every day. His last drink was 1 day ago. On examination, he is afebrile and
he has abdominal distension and ascites without any tenderness on palpation. Lab studies show
WBC 8k/l, Hemoglobin of 10.2 gm%, Platelets 90k/l, Total protein of 6.4, Albumin 2.2,
SGOT 300, SGPT 130, Total Bilirubin 4.2 , Direct Bilirubin 3.3, Prothrombin time of 19 seconds
and Creatinine 2.2. Ultrasound reveals cirrhosis of the liver and splenomegaly. The daughter asks
you if her father can be considered for Liver transplantation. The most common indication for
liver transplantation in the United States :
A) Alcoholic Liver Disease
B) Chronic Hepatitis B
C) Acute Liver Failure
D) Hepatitis C
E) Primary Biliary Cirrhosis

Ans. D

Hepatitis C cannot be treated acutely and although everything looks normal even liver enzymes,
if it the Hepatitis C viral PCR or antibody is detected then its best to start the patient on the 3
drug regimen, Ribavirin, interferon and a protease inhibitor, although theres a monoclonal
antibody that may be added as well.

Most patients can only know the condition of the liver until the liver biopsy is done because the
labs are normal and they may be asymptomatic for a while. Remember, Hepatitis C is
transmitted more parenterally, through needles and blood transfusion than sex! If a person is in a
monogamous relationship with his wife for 20 years and he is HCV+ and she is HCV- then,
CDC does NOT recommend condom use.

Most common cause of liver transplant in USA is hepatitis C, because before the 1989 no one
really got screened for hepatitis C and approximately 170 million worldwide are infected with
Hep C. Hope this helps!

105. A 66 year old man presents to your office with complaints of productive cough and low
grade ever for past two days. He denies any sick contacts. On physical examination, his
temperature is 100F, breath sounds are reduced in left lower lobe. A chest x-ray reveals left
lower lobe infiltrate. Laboratory investigations reveal WBC count of 12,000/l with 80%
neutrophils. Sputum gram stain and cultures are sent for. Blood cultures have been obtained and
are pending. He received a Flu vaccine about one month ago but never received a pneumococcal
vaccine. The next important step in managing this patient:
A) Oral Azithromycin
B) Oral Levofloxacin
C) Admit the patient
D) Swallowing evaluation
E) PPD skin test

Ans. B

CURB 65 score is widely being used for admission in stable patients. CURB > 1 is indication for
admission. This patient should be tx as outpatient pneumonia.
Azithromycin is preferred choice in younger patients for outpatient pneumonia. Older patients
with age > 50 have higher chance of having resistant Strep. Pneumonia, so Azithromycin will not
work in older patients.
106. A 46 year old man is seen in your office for complaints of severe fatigue over the last one
week. The patient was diagnosed with chronic Hepatitis C infection 2 weeks ago for which he
was started on Interferon and Ribavirin. On examination, his vitals are stable and he is afebrile.
Conjunctivae are notable for pallor. Abdominal examination is benign except for mild
splenomegaly.
Laboratory investigations :
WBC count of 3,000/l
Hemoglobin of 5.8gm/dl
Platelet count of 60k/l.
Total protein 6.8gm/dl
Albumin 3.2gm/dl
Total Bilirubin 3.2gm/dl
Direct bilirubin 0.8gm/dl
SGPT 52 U/L
SGOT 66 U/L
Alkaline Phosphatase 110U/L
Haptoglobin < 5.8
Reticulocyte count of 6.0%
Lactic Dehydrogenase 1200 IU/L
Serum creatinine 1.0
Peripheral blood smear reveals reduced platelets, polychromasia and anisocytosis. His laboratory
tests 4 weeks ago were normal except for mild elevations in his liver enzymes. The immediate
next step in managing this patient is :
A) Parvo virus B19 antibodies
B) Endoscopy
C) Plasmapheresis
D) Stop Ribavrin
E) Intravenos Methylprednisolone

Ans. D

Pt. is hemolysing with anemia, elevated total bilirubin with major fraction coming from indirect,
increased lactic dehydrogenase, decreased haptoglobin and increased reticulocyte count. Also
thrombocytopenic and leukopenic. All of these are side effects of Ribavirin.

Ribavirin causes hemolytic Anaemia in 10 % cases


Recommendation-Check Hct pretreatment, 2 wks & 4 wks
Highly Teratogenic
Parvovirus B19 causes Aplastic Crisis in Sickle cell disease or other Hemoglobinopathies.
107. A 24-years old man with history of Hepatitis C is brought to the ER with history of
attempted suicide by cutting his veins with a knife. His family members spotted him while trying
to cut his wrist and could save him. He was immediately admitted to psychiatric ward of the
hospital. On examination, his vitals are stable. The patient appeared very depressed. The wrist
was bandaged and there is no significant bleeding. The patient has a history of depression for the
past three years which was adequately controlled on Fluoxetine. Laboratory investigations
revealed ALT: 95 IU/ml , AST: 65 IU/ml, , Hemoglobin: 15.2 gr/dl, Platelet count: 345000/ml,
WBC count: 6500/ml. Approximately 4 weeks ago, the patient was diagnosed with HCV
infection after screening due to intravenous drug addiction history. At that time, HCV RT PCR
was positive, HIV ab and Hepatitis B Surface antigen were negative. The patient was started on
Interferon and ribavirin for Hepatitis C infection. The patient denied any recent drug use. He
has been compliant with his Fluoxetine. He reports increased suicidal thoughts over the past few
days. The most important next step in controlling this patients depression :
A) Electroconvulsive therapy
B) Discontinue Fluoxetine
C) Start Cyproheptadine
D) Discontinue Interferon
E) Start Clozapine to reduce suicidal tendency

Ans. D

Discontinue interferon as it can induce or add on to depression that is why before starting
interferon the pt is asked about depression and also sometimes psychiatric consultation is
recommended.

Combination therapy with interferon- and ribavirin is considered the treatment of choice for
chronic hepatitis C. However, interferon- may induce severe depression. It has been suggested
that interferon- is able to modify cytochrome P450 (CYP) 1A2 and 2D6 activity. We therefore
decided to study the effects of the interferon--2b pegylated derivative on fluoxetine disposition
in patients receiving combination chemotherapy for chronic hepatitis C. Data suggest that
interferon- may induce, rather than inhibit, the biotransformation of fluoxetine.

108. A 34 year old man with history of schizophrenia and hypertension presents to your office
with complaints of sore throat and low grade fever. He has mild cough. He denies any sick
contacts. On examination, the patient has a temperature of 100F. Throat examination reveals
mild erythema with no exudate. There is no cervical lymphadenopathy. His medications include
Metoprolol and Clozapine. The most important step in managing this patient :
A) Influenza Rapid Antigen Test
B) Throat Lozenges
C) Obtain Complete Blood Count
D) Oral Azithromycin
E) Discontinue Clozapine
Ans. C

Do CBC first to see if the pt has agranulocytosis. Decrease WBC count then go ahead and
discontinue Clozapine.

109. A 35 year old HIV positive male patient comes to your office with complaint of anorexia,
nausea and vomiting and abdominal pain. His anti-retroviral medications include Stavudine and
Didanosine. On physical examination, his temperature is 100F. His abdomen is tender in the
epigastric area. Laboratory results reveal WBC count of 20k/l, Serum amylase 500 IU/L ( N
= 25 to 160 U/L) , Lipase 590 units/liter (Normal = 25 to 300U/L), AST 120 IU/L, ALT 200
IU/L, Total bilirubin 3.6gm% and Alkaline phosphatase 200IU/L . Ultrasound reveal gallstones
with no cystic duct obstruction and no evidence of cholecystitis and a common bile duct
diameter of 1.5cm. The most appropriate step that would address the etiology of his acute
pancreatitis is:
a) Intravenos Imipinem/ Cilastatin
b) Supportive care followed by Endoscopic Retrograde Cholangiopancreatography ( ERCP)
c) Stop Didanosine
d) Stop Stavudine
e) Exploratory Laporotomy

Ans. B

When you have a patient with pancreatitis, there can be many causes - drug, ETOH, gall stones.
It is important to indentify the cause from their labs, so we can select next suitable step. A gall
stone pancreatitis often has elevated ALT more than AST along with increase ALP and total
bilirubin. If ETOH pancreatitis, LFT are typically normal but, if there is concomitant ETOH
hepatitis you will still have AST > ALT. Remember in gall stone cystic bile duct (CBD)
obstruction, ALT tends to rise first. Then do ultrasound to confirm this and increased CBD
diameter > 1.0 cm is indicative of obstruction and therefore, should be followed by ERCP for
confirmation of diagnosis as well as to relieve CBD obstruction by stone removal and/or stent
removal.

110. A 45 year old HIV positive patient has been receiving Highly active anti-retroviral therapy.
Her medications include Lamivudine, Zidovudine, Indinavir and Ritonavir. Her most recent HIV
viral load was undetectable and Absolute CD4 count was 400/l . Eight weeks after initiation of
therapy, the patient comes to the emergency department complaining of nausea, burning
urination, frequency and severe flank pain. One week prior to this visit, the patient visited the ER
for burning urination and was treated with Trimethoprim/Sulfamethaxozole. Urine cultures from
last visit are negative. Laboratory investigations reveal a serum creatinine of 2.2. A urinalysis is
negative for protein, nitrite and leucoesterase without any bacteriuria. Urine microscopy reveals
numerous WBCs and some starburst crystals. A non-contrast abdominal CT scan reveals mild
right hydronephrosis without any evidence of stones. Patient is given adequate pain medications.
The most likely etiology of this patients renal insufficiency :
a) Acute Pyelonephritis
b) Acute Bacterial Cystitis
c) Indinavir Nephropathy
d) Allergic Interstitial Nephritis
e) HIV nephropathy

111. Most important next step in managing this patients renal failure :
A) IV hydration and Intravenos Ceftriaxone
B) IV hydration and Discontinue Indinavir for three days
C) IV hydration and Enalapril
D) Arrange for Hemodialysis
E) Discontinue Indinavir and refer for Lithotripsy

Ans. C and B

Crystallization of indinavir within the renal tubules may occur with or without urologic
symptoms. Symptomatic urinary tract disease associated with indinavir includes a range of
clinical manifestations that depends on whether stone formation has occurred. Patients with
indinavir-related nephrolithiasis typically have acute onset of severe flank pain and hematuria.
Kidney stones typically formed by indinavir are radiolucent unless calcium oxalate has been
incorporated and thus indinavir-induced kidney stones are not usually apparent on a non-contrast
computed tomographic imaging of kidney ureter bladder (CT-KUB). Patients with crystalluria in
the absence of stone formation generally have intermittent dysuria or flank pain. This
crystalluria-dysuria syndrome may mimic infectious cystitis and lead to unnecessary
antimicrobial treatment. Indinavir-induced crystal nephropathy generally does not cause
symptoms, but this disorder should be suspected if the patient develops an increased serum
creatinine level or sterile pyuria; CT-KUB scans show renal parenchymal imaging abnormalities,
including cortical atrophy or hydronephrosis. Persistent sterile pyuria appears to be an early
indicator in the development of renal insufficiency and corresponds histologically to
interstitial nephritis. Unlike HIV nephropathy, proteinuria does not appear to be a significant
component of indinavir-related nephropathy.

In order to prevent the formation of indinavir crystals within the urine, it is recommended that all
patients on indinavir maintain a fluid intake of at least 1.5 liters per day (roughly 8 glasses of
fluid per day). Routine urinalysis and monitoring of renal function should be incorporated into
the care of patients on indinavir. Since asymptomatic crystalluria is relatively common and is not
a predictor of subsequent complications, it is not an indication for the withdrawal of indinavir
therapy. This finding may, however, suggest the need for increased fluid intake. The presence of
sterile pyuria is more concerning as it seems to represent early tubular inflammation and
identifies individuals with renal insufficiency as well as those at risk for developing
renal insufficiency. In most patients, the indinavir-induced renal insufficiency reverses with
discontinuation of indinavir and increased hydration. Most indinavir stones are passed through
the urine with conservative management consisting of vigorous hydration, adequate analgesics,
and discontinuation of indinavir for 1 to 3 days. In this setting, considering indinavir is no longer
a preferred antiretroviral medication, most experts would recommend switching to an
antiretroviral regimen that does not contain indinavir. Attempts to treat indinavir nephrolithiasis
with urinary acidification have been unsuccessful. For larger stones, or stones that fail to respond
to conservative management, ureteral stenting and ureteroscopic removal may be necessary to
remove the obstruction. Due to the gelatinous nature of these stones, lithotripsy is not effective.

112. A 30-year-old HIV-infected man presents to your office for evaluation regarding starting of
Anti-retroviral therapy. The patients most recent CD4 count was 460 cells/mm3 and HIV RNA
level of 20,000 copies/ml. He has a history of Intravenous drug abuse. The patient was also
noted to have elevated liver enzymes twice the upper limit of normal. During the work-up for
his liver abnormalities, his Hepatitis C is negative but Hepatitis B surface antigen returns
positive consistent with hepatitis B active infection. He has no HIV-related symptoms and has
not had any AIDS-defining illnesses. The patient requests that he be started on Highly Active
Anti-Retroviral therapy (HAART). Which of the following is the most appropriate indication for
starting HAART in a HIV infected patient?
A) CD4 count of less than 500
B) HIV viral load greater than 50,000
C) Initiation of treatment for Hepatitis B co-infection
D) Renal Insufficiency without proteinuria
E) All Reproductive age group HIV + women

Ans. C

Guidelines for Initiating Antiretroviral Therapy in Chronically HIV-infected Patients

AIDS-defining illness or severe - Any CD4 count - Trea symptoms of HIV infection
Asymptomatic - CD4 < 350 cells/mm3 - Treat
Asymptomatic - CD4 > 350 cells/mm3 - Consider treatment to decrease risk of HIV
associated complications (e.g. TB, Kaposi's), decreased HIV transmission to others
HIV-associated nephropathy - Any CD4 count - Treat
HIV-associated thrombocytopenia - Any CD4 count - Treat
Pregnant women - Any CD4 count - Treat to prevent mother to child transmission.
Consider discontinuing treatment post-partum if not otherwise indicated
Persons co-infected with HBV or HCV - Any CD4 count - Treat both HIV and
HBV/HCV
113. A 44-year-old HIV-infected man with a CD4 count of 280cell/mm3 presents to your office
with complaints of fatigue, body aches, leg cramps and muscle pain. His viral load is
undetectable. His medications include tenofovir, lamivudine and Ritonavir. The patient was
recently seen in the office for lipodystrophy and hyperlipidemia. His LDL cholesterol was 190
during last visit and hence, he was started on Simvastatin about 2 weeks ago. On physical
examination, he is afebrile and he has diffuse muscle tenderness. Laboratory studies show a
serum creatinine of 3.2 mg/dL ( his baseline = 1.0 mg/dl), serum urea nitrogen = 55 mg/dL , total
bilirubin 0.8gm/dl, aspartate aminotransferase (AST) level of 632 U/L and alanine
aminotransferase (ALT) level of 140 U/L . Urinalysis was positive for blood on dipstick. Urine
microscopy shows no red cells or white cell casts. The most useful test in determining the
etiology of the liver enzyme elevations in this patient:
A) Ultrasound Abdomen
B) Serum Creatinine Phosphokinase
C) Gamma glutamyl transferase (GGTP)
D) CT abdomen with contrast
E) Hepatitis C antibodies

Ans. B

Statin induced myopathy can cause CPK elevation which leads to elevated LFT and renal failure.
AST = muscle origin, myopathy related LFT elevation.
Not all blood in urine is RBC, It can be myoglobin or drugs. The AST elevation here is from
muscle, it is not from liver. Know the non-hepatic causes of elevation AST and ALP, Very
important for this exam.

The most severe adverse effect of statins is myotoxicity, in the form of myopathy, myalgia,
myositis or rhabdomyolysis. Clinical trials commonly define statin toxicity as myalgia or muscle
weakness with creatine kinase (CK) levels greater than 10 times the normal upper limit.
Rhabdomyolysis is the most severe adverse effect of statins, which may result in acute renal
failure, disseminated intravascular coagulation and death.

114. A 44-year-old HIV-infected man with a CD4 count of 280cell/mm3 presents to your office
with complaints of fatigue, body aches, leg cramps and muscle pain. His viral load is
undetectable. His HIV medications include tenofovir, lamivudine and Ritonavir for the past one
year. The patient was recently seen in the office for lipodystrophy and hyperlipidemia. His LDL
cholesterol was 190 during last visit and hence, he was started on Simvastatin about 2 weeks ago.
On physical examination, he is afebrile and he has diffuse muscle tenderness. Laboratory studies
show a serum creatinine of 3.2 mg/dL ( his baseline = 1.0 mg/dl), serum urea nitrogen = 55
mg/dL , total bilirubin 0.8gm/dl, aspartate aminotransferase (AST) level of 632 U/L and alanine
aminotransferase (ALT) level of 140 U/L . Urinalysis was positive for blood on dipstick. Urine
microscopy shows no red cells or white cell casts. The most likely reason behind the etiology of
this patients renal failure :
A) Polymyositis
B) HIV associated Nephropathy
C) Tenofovir induced Nephrotoxicity
D) Interaction between Ritonavir and Simvastatin
E) HIV myopathy

Ans. D

Most PIs inhibit the metabolism of most statins and can significantly increase serum statin levels,
thus increasing the risk of toxicity, including myopathy and rhabdomyolysis. The degree to
which statin metabolism is affected by PIs varies according to the statin as well as the specific
PI. In general, the potential for inhibition of statin metabolism is as follows: simvastatin and
lovastatin > atorvastatin and rosuvastatin >> pravastatin.

The most significant adverse effects associated with statin use have consisted of skeletal muscle
injury and hepatic dysfunction. The manifestations of skeletal muscle injury may include
myalgias, acute myositis, and rhabdomyolysis. Although the risk of developing myopathy in
patients taking statins is very low, it increases substantially with concurrent use of drugs known
to inhibit CYP3A4 isozymes. Rare reports exist of life-threatening rhabdomyolysis resulting
from drug-drug interactions with PI and statins. The laboratory manifestations of myopathy
consist of elevations in one or all of the following: creatine kinase (CK), lactate dehydrogenase,
and transaminases (AST). Severe cases of drug-induced rhabdomyolysis may lead to acute renal
failure and severe electrolyte imbalances. As in this case patient is receiving statins with
Ritonavir. Delavirdine has been implicated in a case report of severe rhabdomyolysis and acute
tubular necrosis when co-administered with atorvastatin (20 mg once daily).

115. A 44-year-old HIV-infected man with a CD4 count of 280cell/mm3 presents to your office
for regular follow up. His viral load is undetectable. His HIV medications include tenofovir,
lamivudine and Ritonavir for the past one year. On examination, he has features of
lipodystrophy. A fasting lipid panel reveals Total cholesterol 270 mg/dl, LDL cholesterol
200mg/dl, Triglycerides 150mg/dl and HDL 40mg/dl. He is advised to start low fat diet and
exercise. The most important next step in controlling this patients hyperlipidemia.
A) Add Niacin
B) Add Simvastatin
C) Add Pravastatin
D) Add Lovastatin
E) Hold HAART therapy until lipids normalize
Ans. C

Pravastatin metabolism is complex, utilizing both oxidative and conjugative enzymes


for elimination, but CYP3A4 is not a significant enzyme in its metabolism. Therefore, it
appears a safe option in patients taking protease inhibitors. However, pravastatin may be less
effective at lowering lipid levels in these patients due to the induction of enzymes responsible for
the metabolism of pravastatin.

Niacin is another option for patients with high triglycerides. A small pilot study suggested
that it may be safe in HIV patients on antiretroviral therapy. However, the HIV Medical
Association of the Infectious Disease Society of America and the AIDS Clinical Trials Group
guidelines do not advocate using it as first-line therapy in patients receiving protease inhibitors
or with lipodystrophy because of the potential for insulin resistance. Additional studies
evaluating the safety of niacin among HIV patients are ongoing.

116. A 26 year old woman has dysmenorrhea that has not responded to treatment with NSAIDs.
Her past medical history is significant for migraine without aura and takes Topiramate for
prevention of migraine. Her migraines are well prevented now. She is also sexually active and
requests contraception. In view of her dysmenorrhea, OC pills have been recommended to her as
it serves to address both the issues of contraception as well as her dysmenorrhea. But she tells
you that she once read the package insert in the OC pills and also heard from her friends that she
should not use OCPs because she has migraine. Her exam does not reveal any neurological
deficits. She does not smoke and leads an active lifestyle. Her B.P is 110/70. What is your best
recommendation to her?
A. Reassure her and start OC Pills
B. Tell her to use condoms alone
C. Start minipill because OC pills may worsen her headache
D. Start OC pills but switch topiramate to valproic acid to prevent her migraines better

Ans. A

This pt has migraine without aura, no focal neurological deficits and no additional risk factors for
stroke. So, it is recommended to start OC pills as benefits outweigh risks in her case. Studies
have shown that headache occurring in association with OC pills use tend to improve despite
continue OC use.

117. A 35-year-old woman with history of smoking 1 ppd x 15 yrs, comes to you 4 months after
beginning OC pills. Shortly after starting OCs, she started experiencing headaches twice a week
lasting 12 hours. The headaches are bilateral, throbbing, and accompanied by nausea and
sensitivity to light and sound. They are heralded by a 50-minute visual disturbance consisting of
a bright, zigzag lines and then fades away as the headache begins. Upon questioning, she
reports occasional similar headaches prior to OC use but they were not this bad and never had
visual disturbances earlier. Her physical examination is normal. She is sexually active with one
partner and desires effective contraception. Her partner does not like using condoms. The next
step in management?
A. Reduce the dose of estrogen in the combination pill
B. Switch to mini pill
C. Ask her to convince her partner to use condoms
D. Reassure her and continue OC Pills
E. Stop OC pills and restart after one month.

Ans. B

Mini pills are progesterone only pill. No risk for DVT, no risk for decreasing breast milk
production and safe for pt with falcemia (sickle cell disease).

If migraine worsen or if there is new-onset migraine related to OC use, consider the following
stroke risk factors:

Patient age (age > 35 increased risk)


The type of migraine i.e, with aura is increased risk for CVA
The presence of other vascular risk factors i.e, smoking, HTN. In this woman smoking is
additional risk along with migraine with aura.
So, combination pills have to be decreased. Switched to mini pill or other forms of reasonable
contraception.
Any unusual headache with sudden onset, focal neurological deficits - immediately stop OC pills
and get a head CT.

118. A 30-year-old woman has been using oral contraceptive pillls, combination type for past 8
yrs. However, she also has a history of migraines. Lately, she has been experiencing an average
of 14 episodes of severe migraine without aura yearly. Careful evaluation of her headache
calendar reveals that most of them occur exclusively during the pill-free week of her OC
regimen. She has no history of smoking. She has never had DVT or family hx of thrombophilia.
Her physical exam is normal without any neurological deficits. Next step in management ?
A. Switch to low dose estrogen pills
B. Switch to minipill
C. Discontinue OC pills
D. Start extended duration OC pills like seasonale
Ans. D

Realize that some migraines improve with OC pills. Some patients especially gives you a history
that they get more migraines during pill free period this is related to estrogen withdrawal. This
patients will benefit from extended OC regimens.

Seasonale: 84 consecutive hormonal pills followed by 7 days of placebo . Reasons for the use of
extended duration OC regimens ( 91 days).
-Convenience.
-patient desire for fewer episodes of withdrawal bleeding ( they will have only 4 bleeding
episodes per year).
-To reduce the occurrence of headache and other estrogen-withdrawal symptoms ( Premenstrual
syndrome).

119. A 60-year-old woman presents to the Emergency Room with massive hemetemesis. The
onset is acute. She denies any alcohol use or any antecedent nausea, vomiting or retching. On
physical examination, the patient is found to be hypotensive with a blood pressure of 80/40. The
patient is started on Intravenous fluids and proton pump inhibitors. Prothrombin time and liver
function tests are within normal limits. Hemoglobin is low at 7gm/dl and the patient is now being
transfused with 2 units of packed red cells. An immediate Endoscopy is scheduled which
revealed bleeding gastric varices but no esophageal varices. Local vasocontrictor therapy and
band ligation could not restrain the bleeding. Ultrasound and CT scan of abdomen revealed
enlarged spleen, an engorged splenic artery and an intraluminal filling defect in the Splenic Vein
as shown in the picture below

The most common etiology of the condition mentioned above :


A) Polycythemia Vera
B) Inherited Thrombophilias
C) Liver Cirrhosis
D) Chronic Pancreatitis
E) Carcinoma Pancreas
120. The pathophysiology behind the development of Varices in this patient :
A) Systemic Portal Hypertension
B) Superior Mesenteric Vein Thrombosis
C) Liver Cirrhosis
D) Sinistral Portal Hypertension
E) Angiodysplasia

121. The most effective treatment for the condition described above:
A) Transjugular Intrahepatic Portosystemic Shunt
B) Thrombolysis
C) Anticoagulation with heparin
D) Splenectomy

Ans. D, D, D

Keywords: Left-sided portal hypertension, Sinistral portal hypertension, Upper gastrointestinal


haemorrhage, Gastric varices and Splenectomy.

Sinistral, or left-sided, portal hypertension is a rare cause of upper gastrointestinal haemorrhage.


There are many causes of sinistral portal hypertension. The primary pathology usually arises in
the pancreas and results in compression of the pancreatic vein. This compression causes
backpressure in the left portal venous system and subsequent gastric varices. Management is
usually surgical to treat the underlying pathology and splenectomy to decompress the left portal
venous system.

122. A 75 year old man presents to your office with complaints of severe fatigue and
constipation for the past one week. He has no significant past medical history except for benign
prostatic hypertrophy for which he takes terazosin. Physical examination reveals mild tenderness
in left mid thigh area. Rest of the exam is normal. An x-ray of the left lower extremity reveals a
lytic lesion in the shaft of the femur. A subsequent bone scan reveals multiple lytic lesions in the
ribs, right iliac bone, left and right femur shaft as well as in the left femur neck. The most
important next step in managing this patients symptoms is :
A) Serum PSA level
B) Serum protein electrophoresis
C) Start IV Bisphosphonates
D) Check serum calcium level and start IV hydration
E) Skeletal surevey
123. The patient was admitted to the hospital and his symptoms were managed appropriately.
However, during the second hospital day the patient complains of pain in his left thigh. Review
of his previous x-ray reveals lytic lesion occupying the head and neck of femur. The most
important immediate next step in managing this condition
A) MRI of the Hip
B) CT scan of the Hip
C) Orthopedic consultation for internal fixation
D) Radiation therapy
E) Chemotherapy

Ans. D and C

Management issues should be addressed first and not Multiple Myeloma. Giving chemotherapy
or radiation will not treat his thigh pain that needs to be referred to the orthopedic surgeon.
Hypercalcemia needs to be treated with IV hydration!

Doing an electrophoresis now does not change the course of management . The vignette clearly
tells us its a MM patient. Electrohporesis takes time and money. Hypercalcemia can cause
severe symptoms such as confusion , altered mental status. Clue to that : Fatigues , constipation .
thus we have to address this issue first.

If the question were to said; Whats the best next step to confirm this patients condition : that
would be Serum electrophoresis.
However question states whats the next step in managing this patients SYMPTOMS .
Orthopedic consultation is the next answer because patient complaints THIGH PAIN + LYTIC
LESION OF FEMUR HEAD possibility of femoral head osteonecrosis is high. We have to
address this issue first.

124. A 50 year old man comes for regular health checkup. His family history is significant for
colon cancer in his grandfather at the age of 70 and prostate cancer in his father at age 75. His
mother had breast cancer at age 65. The patient recently underwent colonoscopy which was
normal. During his visit, he asks you if there are any pills or medications that he could use to
prevent cancer. Which of the following have been shown to prevent cancer in a prospective
clinical trial?
A) Vitamin D
B) Selenium
C) Beta Carotene
D) Finasteride
E) Vitamin B-complex
Ans. D .

Finasteride is the only drug in the list proven to reduce prostate cancer risk in men . The PCPT (
Prostate Cancer Prevention Trial) has conclusively shown that finasteride can reduce the
incidence of prostate cancer.

Beta-Carotene is a great distracter in this question as everyone must have heard a lot about it.
However, it does not prevent cancer in fact, increases lung cancer risk, SELECT trial is still
ongoing and has not shown any cancer prevention role of selenium in preliminary data. SELECT
stands for the Selenium and Vitamin E Cancer Prevention Trial. Beta carotene is detrimental and
increases lung cancer risk. Vitamin D trials so far have not shown that supplementation will
reduce the cancer risk though the trials have found some association between the deficiency and
cancer.

125. A 55 Year old man comes to your office with complaints of progressively shortness of
breath on exertion for the past 3 months. Physical examination reveals scattered wheezes .
Pulmonary function tests reveal irreversible obstructive pattern. The patient is started on
tiotropium inhaler. His other medications include enalapril for hypertension and over-the-counter
vitamin supplements that he routinely uses on a daily basis which include Vitamin A, Vitamin B-
complex, Beta-carotene, Selenium and Vitamin E. Chest X-ray is consistent with chronic
obstructive airway disease but did not reveal any lung nodules or masses. The patient is
counseled about lung cancer risk and strongly counseled on cessation of smoking. In addition to
counseling regarding smoking cessation, which of the following is the most important measure to
reduce his lung cancer risk?
A) CT scan of the chest every year
B) Discontinue Vitamin E
C) Discontinue Selenium
D) Discontinue Beta Carotene
E) Start Vitamin D supplements for chemoprevention

Ans. D

Pharmacologic or nutritional prevention of lung cancers is needed, especially for 60 million


Americans who are former smokers. A portfolio of large-scale trials of beta-carotene, beta-
carotene with and without vitamin E, and beta-carotene plus vitamin A demonstrated no benefit
whatsoever from beta-carotene. The alpha-Tocopherol / beta-Carotene Trial and the beta-
Carotene and Retinol Efficacy Trial found significant increases in lung cancer risk and total
mortality. Laboratory research soon identified multiple adverse molecular effects. Nevertheless,
chemoprevention remains an active, promising strategy, with new hypotheses and new candidate
agents, including many already approved as therapies. The most active area currently is focused
on selective inhibition of arachidonic metabolism, both Cox-2 and Lox pathways.
Beta carotene has been shown to increase the risk of lung cancer among smokers. People who
took dietary supplements of the nutrient beta-carotene while enrolled in a large cancer prevention
trial continued to have increased rates of lung cancer even six years after the trial was stopped
early and the supplements discontinued. Contrary to earlier expectations, not only do beta-
carotene supplements not prevent lung cancer in people at high risk for the lung cancer, they
appear to increase rates of the lung cancer, particularly among smokers.

A is incorrect. Screening will not reduce the lung cancer risk. Annual screening with CT Scan of
the chest has not yet been shown to reduce the mortality of the lung cancer. NIH is currently is
conducting a large study that should conclusively answer if either spiral CT or chest X-ray can
reduce lung cancer deaths.
B is incorrect. There is no conclusive association between Vitamin E and Lung Cancer. Smokers
given vitamin E supplementation during a six-year clinical trial showed no reduction in their
lung cancer risk.
C is incorrect. Selenium has not been shown to increase the lung cancer risk nor has it been
proved to protect against lung cancer.
E is incorrect. Though Vitamin D has been shown to prolong survival in lung cancer patients in
some small trials, there is no evidence that it prevents lung cancer.

HERBS and their interactions with drugs are high yield on USMLE Step3.

Adverse Effects and Drug Interactions of Herbal Remedies : -


Approximately 25 percent of Americans who consult their physician about a serious health
problem are employing unconventional therapy, but only 70 percent of these patients inform
their physician of such use.

Unlike conventional drugs, herbal products are not regulated for purity and potency. Thus, some
of the adverse effects and drug interactions reported for herbal products could be caused by
impurities (e.g., allergens, pollen and spores) or batch-to-batch variability. In addition, the
potency of an herbal product may increase the possibility of adverse effects.

Ginkgo Biloba : The active ingredients in Ginkgo biloba extract account for its antioxidant
properties and its ability to inhibit platelet aggregation. Consequently, this herbal product is
promoted for use in improving cognitive function and blood flow. To date, however, at least four
reports of spontaneous bleeding in association with use of Ginkgo biloba.

herb.......................>side effects

Ginkgo biloba...........> Bleeding

St. John's wort..........> Gastrointestinal disturbances, allergic reactions, fatigue, dizziness,


confusion, dry mouth, photosensitivity , Interaction with Red Wine.
Ephedra (ma huang).......> Hypertension, insomnia, arrhythmia, nervousness, tremor, headache,
seizure, cerebrovascular event, myocardial infarction, kidney stones.

Kava..............> Sedation, oral and lingual dyskinesia, torticollis, oculogyric crisis, exacerbation
of Parkinson's disease, painful twisting movements of the trunk, rash

HERB..............>INTERACTING DRUGS

Ginkgo biloba........> Aspirin, warfarin (Coumadin), ticlopidine (Ticlid), clopidogrel (Plavix),


dipyridamole (Persantine)

St. John's wort........> Antidepressants

Ephedra..................> Caffeine, decongestants, stimulants

Ginseng....................> Warfarin

Kava........................> Sedatives, sleeping pills, antipsychotics, alcohol

Ginkgo biloba may also interact with warfarin (Coumadin). A 78-year-old woman who had been
taking warfarin for five years after coronary bypass surgery suffered a left parietal hemorrhage
after using a ginkgo product for two months. No change was noted in her prothrombin time. The
intracerebral bleeding was attributed to the antiplatelet effects of ginkgo.

St. John's Wort : St. John's wort (Hypericum perforatum) is an herb widely promoted as a
"natural" antidepressant. In Germany, this herb is commonly prescribed for various
psychopathologic conditions involving depression and anxiety.

Ephedra : - Ephedrine and related alkaloids are the pharmacologically active moieties of the
extract of Ephedra (a genus of shrubs). Ephedrine constitutes 30 to 90 percent of the alkaloids of
Ephedra species. The extract of some species also contains pseudoephedrine. Ephedra (ma
huang) is commonly found in herbal weight-loss products referred to as "herbal fen-phen." Some
weight loss clinics and retail outlets promote herbal products as alternative to the use of
fenfluramine (Pondimin) and dexfenfluramine (Redux). Ephedrine-containing products are also
marketed as decongestants, bronchodilators and stimulants. Other promoted uses include
enhancement of athletic performance and body-building efforts. Marketed uses of ephedrine-
containing products such as "herbal ecstasy" include induction of a euphoric state.

Ephedra-containing products have also been associated with the development of kidney
stones. The risks of using ephedrine-containing supplements appear to outweigh the benefits.
Consequently, patients should be advised not to use these products if they are sensitive to the
effects of sympathomimetic agents. Such patients include those with hypertension,
hyperthyroidism, diabetes mellitus, psychiatric conditions, glaucoma, prostate enlargement,
seizure disorders and cardiovascular disease. Concomitant use of ephedrine-containing products
and caffeine or other stimulants should also be discouraged.

Ginseng : -Little scientific evidence shows that ginseng is effective for any purpose.
Nonetheless, this herb has been purported to strengthen normal body functions, increase
resistance to stress.

Kava :- Kava is an herbal sedative with purported antianxiety or calming effects. In one case
series involving four patients, kava was associated with extrapyramidal effects at dosages of 100
to 450 mg per day. The extrapyramidal side effects included oral and lingual dyskinesia,
torticollis, painful twisting movements of the trunk, oculogyric crisis and exacerbation of
Parkinson's disease.

Kava has also been shown to have additive effects with central nervous system depressants.
example : A patient taking alprazolam (Xanax), cimetidine (Tagamet) and terazosin (Hytrin)
became lethargic and disoriented after ingesting kava. Kava should not be used with
benzodiazepines.

126. A 25-year-old man presented to the dermatology clinic with a one-week history of papular
lesions on penis. He says they are very itchy. The lesions appeared about the same time on his
hands as well. The patient has multiple sexual partners. His wife has similar a problem with itchy
papules. The patient denied any penile discharge. On examination he has lesions consisted of
firm papules about 1 to 4 mm in diameter. Some papules were present in the finger webs. There
were excoriations on other parts of his body. Based on the patient's history and physical
examination, which one of the following is the most likely diagnosis?
A. Bullous pemphigoid.
B. Pemphigus vulgaris.
C. Genital herpes.
D. Scabies.
E. Syphilis.

127. The most appropriate treatment for this patient:

a. Imiquomoid
b. Permethrin
c. Liquid nitrogen
d. Acyclovir
e. Penicillin

Ans. D and B
Scabies is an itchy, highly contagious skin condition caused by an infestation by the itch mite
Sarcoptes scabiei. Direct skin-to-skin contact is the mode of transmission. Sexual contact is the
most common form of transmission among sexually active young people, and scabies has been
considered by many to be a sexually transmitted disease(STD). However, other forms of physical
contact, such as mothers hugging their children, are sufficient to spread the mites. Over time,
close friends and relatives can contract it this way, too. School settings typically do not provide
the level of prolonged personal contact necessary for transmission of the mites.

Scabies produces a skin rash composed of small red bumps and blisters and affects specific areas
of the body. Scabies may involve the webs between the fingers, the wrists and the backs of the
elbows, the knees, around the waist and umbilicus, the axillary folds, the areas around the
nipples, the sides and backs of the feet, the genital area, and the buttocks. The bumps (medically
termed papules) may contain blood crusts. It is helpful to know that not every bump is a bug. In
most cases of scabies affecting otherwise healthy adults, there are no more than 10-15 live mites
even if there are hundreds of bumps and pimples on the skin.

The scabies rash is often apparent on the head, face, neck, palms, and soles of the feet in infants
and very young children but usually not in adults and older children.

Apply a mite-killer like permethrin (Elimite). These creams are applied from the neck down, left
on overnight, then washed off. This application is usually repeated in seven days. Permethrin is
approved for use in people 2 months of age and older.

128. Parents of a 2-year-old come in to discuss their child's recent febrile seizures. The child has
experienced four seizures, each associated with fever (usually from an ear infection). Each
seizure lasted less than 2 minutes and was generalized tonic-clonic. The child was usually post-
ictal for about 60 minutes but then returned to his normal level of mental function. The parents
are concerned about the long-term significance of these seizures specifically, about any
permanent brain damage and retardation. They wonder if their child should be on medication to
prevent the seizures. Which of the following is the most appropriate statement?

A) Children with a history of febrile seizures usually go on to a more complicated seizure pattern
as they age.
B) Children with a history of febrile seizures typically perform less well on standardized school
tests.
C) Children with febrile seizures typically are growth retarded.
D) Children with febrile seizures are at greater risk for premature death than the general public.
E) Most children who experience febrile seizures develop normally.

Ans. E
There is no evidence that short febrile seizures cause brain damage. Large studies have found
that children with febrile seizures have normal school achievement and perform as well on
intellectual tests.

129. You receive a telephone call from a worried mother. She says her 8 month old son just had
a seizure lasting for 2 minutes. The description is generalized tonic-clonic. The seizure has
subsided. He is feeding well and acting normally. His temperature is 103F and RR: 28/min. The
child is not in any distress as per mother. She asks you what needs to be done. The most
appropriate response:

A) "Take him immediately to the nearest ER"

B) "This is nothing serious. You need to stay calm"

C) "Give antipyretic to the child and monitor the temperature"

D) " Does anyone in your family have epilepsy?"

130. The patient's mother in the above question also asks you what is the risk of her child
developing a recurrent febrile seizure in future. The most appropriate response :

A) There is no such risk to your child

B) Risk is increased if his family member has a history of febrile seizure

C) He has no increased risk since he is younger than one year

D) He is at increased risk of developing intellectual impairment and neurological deficits.

131 The child's mother is still very concerned and she has further questions. She asks you, "
Doctor. I am very worried. Does this episode of seizure increase my son's risk of developing
future epilepsy?" Most appropriate response:

A) Your child had a simple febrile seizure and is definitely at very high risk of developing
epilepsy
B) Your son will be at an increased risk of epilepsy if father has history of febrile seizures.
C) If another seizure occurs during this illness within 24 hours then he will be at increased risk
D) He will not have increased risk of developing future epilepsy.
Ans. C, B, C

Children with febrile seizures are at risk for developing recurrent febrile seizures. The overall
recurrence rate is approximately 30 to 35 percent. However, the values vary with age from as
high as 50 to 65 percent in children who are younger than one year of age at the time of the first
seizure to as low as 20 percent in older children. A major factor influencing the recurrence rate is
the age of the infant at the time of the first seizure.

Young age at onset. History of febrile seizures in a first-degree relative. Low degree of fever
while in the emergency department. Brief duration between the onset of fever and the initial
seizure are the main criteria.

CRITERIA FOR THE DIAGNOSIS OF SIMPLE FEBRILE SEIZURE

Between 6 months and 5 years of age


Non-focal, generalized seizure, involving all limbs
No more than a single seizure per 24-hour period
No other neurologic diagnoses or history of afebrile seizures
No severe metabolic disturbance
No evidence of intracranial infection
Seizure lasting less than 15 minutes
Fever (>38C) present prior to the seizure

Evaluation and Treatment

The laboratory, radiographic, and neurologic evaluation should be aimed at diagnosing the
etiology of fever as outlined below. Although the majority of simple febrile seizures are
associated with viral illnesses, consider serious bacterial infections when appropriate, especially
in children less than 36 months of age. Treatment should be directed at proven or presumed
causes of the fever and not directed at the etiology of the seizure activity.

LUMBAR PUNCTURE (for CSF glucose, protein, cells, culture, Gram stain, and potentially,
herpes PCR)

Perform if the patient has a history or physical exam suggestive of meningitis or intracranial
infection
Perform if the patient has fever, seizure, and meningeal signs or symptoms
Consider if the patient is between 6 and 12 months of age and has not received the
recommended doses of HIB
or Streptococcus pneumoniae vaccines OR if the immunization status cannot be accurately
determined
Consider if the patient has fever, seizure, and antibiotic pre-treatment

EEG is not indicated. NEUROIMAGING is not indicated unless focal abnormalities on physical
exam are identified.
LABORATORY EVALUATION should not be performed routinely for the sole purpose of
identifying the etiology of the simple febrile seizure, but maybe indicated if the source of fever
is uncertain or if the diagnosis is unclear.

Consider CBC, blood culture, serum glucose, electrolytes, magnesium, calcium, phosphorous,
urinalysis, and urine culture.

Parental Education

The risk of febrile seizure in the general population is between 2% and 5%. Simple febrile
seizures are benign, and the long-term prognosis is excellent.
There is no evidence that treating simple febrile seizures with anti-epileptic drugs decreases the
incidence of epilepsy later in life or results in improved cognitive outcomes.
If a patient is less than 12 months of age at the time of the first simple febrile seizure, the risk
of a second simple febrile seizure is 50%. If a patient is over 12 months of age at the time of the
first simple febrile seizure, the risk of a second simple febrile seizure is 30%. Following a second
simple febrile seizure, the risk of future simple febrile seizures is 50%, regardless of the age of
the initial simple febrile seizure.
The risk of epilepsy is minimally increased from 1% to 2.4% in patients who have a simple
febrile seizure and is highest in patients with recurrent simple febrile seizures whose initial
simple febrile seizure occurred prior to 12 months of age.
In rare situations, extreme parental anxiety about recurrent, severe simple febrile seizures
warrants prophylaxis at the beginning of each febrile illness. This is not routinely recommended.
Oral diazepam (0.33 mg/kg Q8 hours at the beginning of a febrile illness) is the only medication
recommended in such circumstances. Antipyretics and anti-epileptics have not been
recommended for prevention of simple febrile seizures.
Rectal diazepam can be prescribed for use during prolonged (over 5 minutes) febrile seizures in
selected cases.

132. While you are on rounds in psychiatry wards, a 50 yr old depressed female patient tells you
that she wants to be discharged soon. She says sometimes she feels like stabbing herself with a
dinner knife provided along with her food. Your most important next step:

A) Discharge the patient according to her request


B) Replace the knives and forks with plastic utensils
C) Arrange one-on one supervision
D) Arrange a nurse to visit her every 30 minutes
Ans. C

Answer A and D are not appropriate choices as they do not reduce suicide risk. Answer B will
replace knives with plastic but will not address other possible forms of suicide she may be
planning. Since this woman already has suicidal ideation and has informed you about it, you
must arrange one-on-one supervision as soon as possible.

133. A 60 year old woman is on clopidogrel therapy and is also taking ginkgo biloba. By altering
which of the following does Ginkgo potentiate the effect of clopidogrel?
a) Bleeding time
b) Clotting time
c) Reduction of Factor V and VII
d) Displaces clopidogrel from protein binding sites
e) Thrombin time

Ans. A

Ginkgo biloba is available as an over-the-counter drug and reported to cause haemorrhage when
co-administered with other antiplatelet agents. Ginko inhibits platelet aggregation like aspirin so
it should prolong bleeding time. Gingko Bilboa (B for bleeding time) always remember!

134) A 55 year-old woman comes to your office complaining of sweating, headache, nausea and
vomiting. This happened shortly after her lunch. She had some grape fruit juice and red wine
with her lunch. She has a long history of depression but she does not believe in conventional
antidepressants. She is not being medically treated for depression and anxiety however, she has
been using over the counter herbal medicine for her depression. Which of the following is the
most likely medication she has been using ?
a) Ginkgo biloba
b) Kava Kava
c) St.johns wort
d) Ephedra
e) Yohimbine

Ans. C

St. John's wort..........> Gastrointestinal disturbances, allergic reactions, fatigue, dizziness,


confusion, dry mouth, photosensitivity , Interaction with Red Wine.
Yohimbine and st johns wort can cause the same effect. Red wine and P450 inhibitor via
grapefruit juice. But the twist that St johns wort is most commonly used as herbal anti
depression compared to yohimbine ( most commonly used to tx male impotence).

135. A 45 year old woman was recently diagnosed with Stage II right sided breast cancer with
positive axillary lymphnodes. The tumor was positive for estrogen receptors. She received
lumpectomy followed by chemotherapy and then, radiation. She is then placed on Tamoxifen
three weeks ago. The patient has been experiencing severe hot flashes since the start of
Tamoxifen therapy. Most important next step in management:
A) Discontinue Tamoxifen
B) Start Fluoxetine
C) Start Venlafaxine
D) Start low dose estrogen pills
E) Reassure that symptoms will abate

Ans. C

Severe hot flashes, reassurance not enough. SSRI are great for hot flashes but SSRI like
fluoxetine inhibits CYP2D6 in the liver which converts Tamoxifen to its active form - endoxifen.
So fluoxetine decreases the Tamoxifen action. Venlfaxine does not inhibit CYP2D6 and is safe
in hot flashes caused by tamoxifen. Discontinuing Tamoxifen and starting estrogen pills are
inappropriate as this will increase the risk of recurrence of her breast cancer.

The possibility that SSRIs might, by inhibiting CYP2D6, slow the metabolism of tamoxifen and
reduce its potency is a concern given that as many as one-fourth of breast cancer patients
experience clinical depression and may be treated with SSRIs. In addition, SSRIs are sometimes
used to treat hot flashes caused by hormone therapy.

Researchers have found that women taking certain SSRIs together with tamoxifen have
decreased blood levels of active tamoxifen metabolites. Because of this, many experts suggest
that patients who are taking antidepressants along with tamoxifen should discuss treatment
options with their doctors. For example, doctors may recommend switching from an SSRI that is
a potent inhibitor of CYP2D6 (such as paroxetine) to one that is a weaker inhibitor (such as
sertraline) or that has no inhibitory activity (such as venlafaxine or citalopram), or they may
suggest that their postmenopausal patients take an aromatase inhibitor instead of tamoxifen.

Other medications that inhibit CYP2D6 include the following:


Quinidine, which is used to treat abnormal heart rhythms
Diphenhydramine, which is an antihistamine
Cimetidine, which is used to reduce stomach acid
136. A 55-year-old man with history of depression has recently been started on fluoxetine. The
patient has history of severe depression in the past. Fluoxetine dose has been recently increased
to obtain adequate control of his symptoms. His depression symptoms are well-controlled now
but the patient now complains of erectile dysfunction. He is deeply concerned about this since
this problem is adversely affecting his relations with his wife. Which of the following is most
appropriate next step in managing this patients symptoms?
A) Decrease the dose of Fluoxetine
B) Start Sildenafil
C) Discontinue Fluoxetine and switch to Bupropion
D) Recommend Yohimbine
E) Recommend penile vacuum pump device.

Ans. C

The effect of bupropion on the sexual functioning of male and female outpatients who developed
anorgasmia or delayed orgasm while receiving fluoxetine treatment for depression. According to
a survey of psychiatrists, Wellbutrin (Bupropion) is the drug of choice for the treatment of SSRI-
induced sexual dysfunction, although this is not an FDA-approved indication. Thirty-six percent
of psychiatrists preferred switching patients with SSRI-induced sexual dysfunction to bupropion,
and 43 percent favored the augmentation of the current medication with bupropion. A higher
dose of bupropion (300 mg) may be necessary: a randomized study employing a lower dose (150
mg) failed to find a significant difference between bupropion, sexual therapy or combined
treatment.

Bupropion may be an appropriate antidepressant for patients who develop sexual dysfunction
during fluoxetine treatment or for whom sexual dysfunction is a concern.

Nice little table to remember for exam:


1) Depression + Stop smoking + No weight gain + No sexual side effects = Buprapion
2) Depression + Neuropathic pain = Duloxetine
3) Depression + Insomnia= Trazadone (pt. is gonesleep)
4) Depression + anxiety= Prozac (fluoxetine)

137. A 45-year-old man presents to your office for regular health check up. Lately, he has
experienced reduced libido and erectile dysfunction . Further work-up reveals no organic cause
for his impotence. You offer him sildenafil however, he requests you to provide information on
any natural herbal medicine that could likely help his sexual problems. Which of the following is
likely to improve his erectile dysfunction?
A) Kava
B) Yohimbine
C) Ephedra
D) St.Johns Wort
E) Ginkgo biloba

Ans. B

Yohimbine hydrochloride is the standardized form of yohimbine that is available as a


prescription medicine in the North America, and has been shown in human studies to be effective
in the treatment of male impotence.
Yohimbine Hydrochloride is a prescription medicine that has been used to treat erectile
dysfunction. Controlled studies suggest that it is not always an effective treatment for
impotence, and evidence of increased sex drive (libido) is anecdotal only.
Yohimbine blocks the pre and post-synaptic alpha-2 adrenoceptors. Blockade of post-synaptic
alpha-2 adrenoceptors leads to minor corpora cavernosa smooth muscle relaxation. In fact the
majority of adrenoceptors in the corpora cavernosa are alpha-1. Blockade of pre-synaptic alpha-2
adrenoceptors leads to increased release of neurotransmitters in the central nervous system and in
the corpora cavernosa penis such as nitric oxide, noradrenaline, and dopamine. Whether nitric
oxide released in the corpora cavernosa has a relaxing effect, noradrenaline has a much powerful
constricting effect by stimulating the unblocked alpha-1 adrenoceptors. Concomitant use of an
alpha-1 blocking agent will prevent constriction caused by the increased adrenergic stimulation.
In addition to yohimbine, Pausinystalia yohimbe contains approximately 55 other alkaloids, of
which yohimbine accounts for 1% to 20% of total alkaloids. Among them corynanthine is an
alpha-1 adrenoceptor blocker. Hence the use of Yohimbe extract in sufficient dosages may
provide concomitant alpha-1 and alpha-2 adrenoceptors blockade and thus may better enhance
erections than yohimbine alone.
Yohimbine has been shown to be effective in the treatment of orgasmic dysfunction in men.

138. A 25-year-old woman is diagnosed with major depression. She denies any suicidal ideation.
She has no history of manic episodes. She smokes about 1 pack of cigarettes per day. The
patient has been appropriately counseled on smoking cessation and is offered antidepressant
therapy. She agrees to antidepressant therapy. The patient is very particular about her weight and
would never consider any therapy that would lead to weight gain. Which of the following is the
most appropriate next step?
A) Start Mirtazepine
B) Start Fluoxetine
C) Start Venlafaxine
D) Start Bupropion
E) Psychotherapy alone
139. After one week of beginning the therapy, the patient calls you and reports no improvement
in her symptoms. She has been using the therapy regularly. She is upset and would like to
discontinue the therapy. Which of the following is most appropriate response?
A) The Medication is not working. I can switch you to a different therapy
B) I am concerned about your compliance with the medications
C) You will need group therapy
D) You will need Electro-convulsive therapy
E) There may be a lag of two to six weeks before the therapy works

Ans. D and E

Some people have reported symptoms such as changes in behavior, hostility, agitation, depressed
mood, and suicidal thoughts (thinking about harming or killing oneself or planning or trying to
do so) while taking bupropion to stop smoking. The role of bupropion in causing these mood
changes is unclear since people who quit smoking with or without medication may experience
changes in their mental health due to nicotine withdrawal. However, some of these symptoms
occurred in people who were taking bupropion and continued to smoke. Some people had these
symptoms when they began taking bupropion, and others developed them after several weeks of
treatment or after stopping bupropion. These symptoms have occurred in people without a
history of mental illness and have worsened in people who already had a mental illness.

Bupropion takes two weeks to reach a steady state in the blood and may take as long as a month
or so to reach complete effectiveness.

140. A 15 year old boy with history of Cystic Fibrosis presents to the Emergency room with
complaints of severe right lower quadrant abdominal pain. He also gives a history of nausea and
vomiting for the past 8 hours. On examination, his temperature is 102F and he has right lower
quadrant tenderness at McBurneys Point. CT scan of the abdomen revealed calcified
appendicolith within the dilated, fluid-filled appendix and infiltration of the surrounding
retroperitoneal fat. The patient is scheduled for Appendicectomy. However, pre-operative labs
reveal an abnormal result that would put the patient at an increased risk of bleeding from
surgery. The patient is referred for medical clearance. Which of the following is most likely to be
abnormal in this patient?
A) Platelet count
B) Prothrombin time
C) Partial Thromboplastin Time
D) Both Prothrombin time and Partial Thromboplastin time
E) Bleeding time
Ans. B

Vitamin K is a fat soluble vitamin and its absorption from the gut is dependent on bile salt and pancreatic
lipase secretion stimulated by dietary fat. People with CF are at risk of developing vitamin K deficiency
due to fat malabsorption as a consequence of pancreatic insufficiency and bile salt deficiency. Other risk
factors include CF related liver disease, frequent antibiotic therapy, inadequate dietary intake and short
gut syndrome resulting from bowel resection.

Regular vitamin K supplements are not currently given unless there is chronic liver disease, a prolonged
prothrombin time or a proposed surgical operation when we would give an oral daily supplement of 5-
10mg for a week prior to the operation, or 10mg intramuscularly prior to the procedure. Vitamin K
deficiency and subclinical vitamin K deficiency are common. Deficiency is almost universal in pancreatic
insufficient children with CF, occurs in all patients with CF liver disease and is found in about one third of
pancreatic sufficient patients. There is increased attention on the role of vitamin K in bone health in both
the general population and in people with CF (Weber, 2001; Nicolaidou et al, 2006). Vitamin K is required
for the carboxylation, and thereby the optimal activity, of osteocalcin which plays an important role in
bone formation. A cause and effect relationship between vitamin K deficiency and low bone mass in CF
has not been proven, but subclinical vitamin K deficiency may be important in the development of CF
related low bone mineral density and is being researched. The need for routine vitamin K
supplementation in CF remains the subject of discussion. Having identified that vitamin K deficiency is
common in our paediatric patients we now plan to undertake further research to determine the optimal
supplemental dose of vitamin K for people with CF.

141. A 50 year old obese woman with history of uncontrolled Hypertension and congestive heart
failure is admitted with complaints of palpitations for the past one week. Her symptoms have
worsened over the past 24 hours. An electrocardiogram shows atrial fibrillation with rapid
ventricular response. After adequately controlling her heart rate with pharmacotherapy, warfarin
therapy is initiated at 10mg per day and the patient is discharged. Three days after her discharge,
patient presents to the ER with erythematous and painful lesion on her right thigh.

The most likely etiology of this condition is:


A) Cholesterol Embolism
B) Toxic Epidermal Necrolysis
C) Steven Johnson Syndrome
D) Sudden drop in Protein C levels
E) Necrotizing Fascitis
142. The next IMMEDIATE step in managing this patient is:
A) Initiate Heparin
B) Surgical Debridement
C) Intravenous Clindamycin
D) Stop Warfarin, administer Vitamin K and initiate Heparin
E) Intravenous Corticosteroids

143. Appropriate treatment has been initiated and the patient symptoms have much improved.
Patient is concerned about the potential risks from her chronic atrial fibrillation. Which of the
following is important in long term management of this patients atrial fibrillation associated
risks?
A) Clopidogrel
B) Low molecular weight heparin daily
C) Initiate Warfarin at a low dose and titrate slowly to INR of 2 to 3
D) Aspirin only
E) No anticoagulation is necessary

144. Which of the following step could have prevented patients symptoms in the above case:
A) Checking Factor VII level
B) Checking Protein C level
C) Checking Protein S level
D) Initiation of Warfarin at a low dose
E) Vitmain K Supplementation

Ans. D, D, C

Patient has Protein C Deficiency, check for protein C deficiency. The reason why we dont
check for protein S is because the incidence of protein C is 1 in 300 cases Vs protein S which is
1 in 20,000.

Generally, heparin prevents the skin necrosis if started with warfarin but that is not an option.
We will not check Factor 7 because Protein C is a protease that results in the cleavage of factor
Va and VIIIa and not VII. Initiation of warfarin at a low dose would not make a difference
because patients are at risk in anyway, if they are not treated with the blood-thinning drug
heparin before taking warfarin.

Patients generally need to be treated with the blood-thinning drug heparin before taking warfarin
in such cases. However, the first thing you need to do is Stop Warfarin, give Vitamin K so that it
reduces warfarin levels in the patient and then start anticoagulation with Heparin preferably
LMW heparin.
Later, once the symptoms subside, start warfarin at a low dose and titrate it to an INR of 2-3.

patient has atrial fibrillation with high CHADS2 score, so need coumadin long term to protect
from stroke. LMWH not studied in long term stroke prevention in afib, it is not oral and difficult
to take injection for life. Most of times, warfarin causes skin necrosis only when started in high
initial doses > 10 mg/ d. This will usually not repeat if warfarin is restarted in lower doses and go
up slowly. This is the long term management.

The CHADS2 score is a clinical prediction rule for estimating the risk of stroke in patients
with non-rheumatic atrial fibrillation (AF), a common and serious heart arrhythmia associated
with thromboembolic stroke. It is used to determine whether or not treatment is required
with anticoagulation therapy or antiplatelet therapy.

CHADS

C- Congestive heart failure


H- Hypertension: blood pressure consistently above 140/90 mmHg (or treated hypertension on
medication)
A- Age 75 years
D- Diabetes mellitus
S2- Prior Stroke or TIA or Thromboembolism

146. A 65 year old Hispanic man presents to your office for intermittent abdominal discomfort.
He is afebrile and his physical examination is benign. A plain x-ray of his abdomen is obtained ,
the film is shown below:

The most likely complication that this patient is likely to develop:


A) Inflammatory bowel disease
B) Colon cancer
C) Diabetes Mellitus
D) Chronic Renal Insufficiency
E) Hyperparthyroidism.
Ans. C

Patient shows extensive pancreatic calcifications, although not very clear. The patient has
intermittent abdominal pain, so its definitely not a colicky picture showing a renal colic or some
so renal insufficiency is ruled out. Theres no this is IBD, Colon cancer or hyperparathyroidism.
The etiology is mostly hypertriglyceridemia or alcohol. The most common cause of acute
pancreatitis in USA is gallstones and not alcohol so keep that in mind AND 9/10 times epigastric
tenderness is acute pancreatitis.

The answer is Diabetes mellitus. Chronic pancreatitis leads to B cell depletion and production of
insulin depletes leading to DM.

147. A 55 year old nurse has recently been exposed to an in-patient with active Tuberculosis
about 2 months ago. Her tuberculin skin test was negative a year ago however; the skin test
reveals an 12 mm induration at this time. A chest x-ray is normal. She denies any cough or fever
or weight loss. A comprehensive metabolic panel is within normal limits. She is started on
Isoniazid for the treatment of latent tuberculosis. Two weeks after the therapy, patient develops
edema in the face and neck, maculopapular rash, lymphadenopathy, asthenia, and a fever of
38C. Laboratory tests reveal a WBC count 20k/l with a differential showing neutrophils of
50%, eosinophils of 30% and lymphocytes 20%. The most likely diagnosis is :
A) Hypersensitivity syndrome
B) Histoplasmosis
C) Disseminated Tuberculosis
D) Parasitic infection
E) Strogyloides infection

Ans. A

Hypersensitivity reactions due to Isoniazid may manifest in the form of fever, skin rash and
hepatitis. A case of pyrexia, erythematous rash, glandular swelling in right axilla, arthralgia,
albuminuria, parotitis and paralytic ileus due to Isoniazid is reported. Eosinophilia, albuminuria,
presence of casts along with reducing substance in urine may be occasionally produced due to
Isoniazid. Isoniazid induced serum sickness has also been reported. Common clinical
manifestations of serum sickness include fever, cutaneous eruptions, arthralgia,
lymphadenopathy and albuminuria.
148. A 55 year old nurse has recently been exposed to an in-patient with active Tuberculosis
about 2 months ago. Her tuberculin skin test was negative a year ago however; the skin test
reveals an 12 mm induration at this time. A chest x-ray is normal. She denies any cough or fever
or weight loss. A comprehensive metabolic panel is within normal limits. She is started on
Isoniazid for the treatment of latent tuberculosis. Two weeks after the therapy, patient develops
edema in the face and neck, maculopapular rash, lymphadenopathy, asthenia, and a fever of
38C. Laboratory tests reveal a WBC count 20k/l with a differential showing neutrophils of
50%, eosinophils of 30% and lymphocytes 20%. The most appropriate next step in management:
A) Start antifungal therapy
B) Discontinue Isoniazid and re-administer after de-sensitization
C) Discontinue Isoniazid and administer Rifampin for four months
D) Start Metronidazole
E) Change to multi-drug therapy, Isoniazid , Pyrazinamide, Rifampin and Ethambutol

Ans. C

Daily Rifampin for 4 months is the correct answer. INH needs to be stopped. Things are
changing in 2014, there will be a combined therapy of INH and Rifampin for just 3 months for a
positive PPD and 9 months of INH will be stopped due to severe side effects and reactions based
on a meta analysis that was done this year.

This is a drug reaction, a life threatening syndrome called DRESS syndrome. (drug reaction
eosinophilia and systemic symptoms syndrome) or DIHS or simply HSS (drug induced
hypersensitivity syndrome or hypersensitivity syndrome) Five diagnostic criteria are carefully
selected in this question; Eosinophilia, Leucocytosis, Rash, Fever >38C, and Lymphadenopathy.
It occurs a few weeks after initiating the offending drug. The treatment is stopping the offending
drug (Isoniazid, in this case) and supportive care and systemic steroids.

The most appropriate next step is drug holiday for at least 3 month until the patient has fully
recovered otherwise it will become difficult to identify which drug is actually causing the
reaction in the future. Rifampin can come with its own adverse effects including rash and
hepatotoxicity. Dual drug reaction to INH and Rifampin have been documented.

Stop INH and start Rifampin. Desensitization is a protocol used when 2 or more drugs are being
used for treatment of active TB and there is a suspicion of drug reaction.

149. A 29 year old internal medicine resident physician has been exposed to a patient with
cavitary pulmonary tuberculosis 1 month ago. He denies any symptoms. His physical
examination is normal. A tuberculin skin test reaction is positive now at 6mm. His Skin test one
year ago was negative. A chest X-ray is within normal limits and chemistry panel is normal. The
most appropriate management option for this patient is :
A) Isoniazid, Pyrazinamide, Rifampin and Ethambutol for 9 months
B) Observation as 10mm is considered positive in health care workers
C) Isoniazid for 9 months
D) Rifampin for 9 months
E) Isoniazid for 6 months

Ans. C

USPSFT followers CDC Guideline for investigation of contacts of person with Infectious TB.

Source (patient) - has infectious TB (equal to cavitary lesion or positive AFB or positive gastric
aspirate)

Contact (resident doctor) - has PPD 6mm induration. CDC interprets an induration transverse
diameter of >5mm as positive in ANY contact (casual or close) of INFECTIOUS TB.

This doctor was exposed to an infectious case of TB and he is declared positive by CDC and
USPSTF standards. The most appropriate management for this patient is to start INH daily for 9
months.

If this doctor was not exposed to an INFECTIOUS case of TB, he would have been a contact
with low priority risk and an induration of >10mm would be considered positive.

Question is, who must have LTBI prophylaxis at > 5mm induration?

1. Immunosuppressed patients (by disease or by cytotoxic or even >15mg prednisone or


equivalent for >4weeks) - high risk group.

2. Contacts of Infectious TB case (all)

3. Evidence of previous TB on CXR (fibrotic changes)

Then who should have LTBI prophylaxis at >10mm induration?

1. ALL ADULTS AND CHILDREN WITH NORMAL IMMUNE SYSTEM + NO EXPOSURE


HISTORY TO ACTIVE (INFECTIOUS)TB, HOWEVER, THEY HAVE BEEN EXPOSED TO
SOME FORM OF TB.

LOWER RISK TB EXPOSURE examples include individuals exposed to confirmed NON


infectious TB or TB on treatment in which AFB is negative or no specific traceable infectious
source LIKE BEING IN HIGH RISK AREAS OR GEOGRAPHICAL ZONES. All these
individuals have good immune system but they have been exposed to some form of TB THAT
HAS NOT been proven to be active. They have no confirmed active TB contact.
Who receives LTBI prophylaxis at even zero mm induration?

This is like saying that you do not believe a negative result of PPD. When can you do that?
When the exposure occurred in a period that is less than 8 to 10 weeks. Clinical suspicion is used
to salvage high risk vulnerable patients. Who are the patients we cannot risk or afford a false
negative test.

Symptomatic children <5years old and immunocompromised Adults. Why? Studies have shown
that they are at high risk of rapid disseminated form of TB that is also lethal.

1. Children1wk, and you have clinical suspicion of TB, initiate PTBI prophylaxis with INH daily
or Rifampicin until repeat PPD in 8 to 10 weeks. If it is negative then, prophylaxis can be
stopped.
2. Immunocompromised individuals (Low CD4 count), exposed to high risk individuals (one
with active confirmed TB), and there is clinical suspicion of TB, initiate PTBI prophylaxis. If
repeat PPD is positive after window period, LTBI prophylaxis is continued.
3. Severely immunocompromised individuals exposed to active TB
Key words Clinical suspicion, PPD reads negative, risk group vulnerable to rapid disseminated
TB (immunosuppressed or children<5years), if untreated for next 10 weeks.

FOR RECALL(IN EXAM):


a, if you hear or feel the word 'confirmed ACTIVE' or 'confirmed IMMUNODEFICIENCY' it's
5mm for positive NOT found?, look at b,
b, If you hear 'confirmed' RISK ALONE it's 10mm for positive
c, If you hear SYMPTOMATIC CHILD 12 years - INH plus Rifampicin RPT once weekly dose
for 12 weeks in age >12 years and adults

From TN:

Positive PPD Test

If induration at 48-72 h > 5 mm if immunosuppressed, close contact with active TB, CXR
fibrocalcific disease, with HIV-positive or if using anti-TNF blockers

> 10 mm all others; decision to treat depends on individual risk factors

False -ve : poor technique, anergy, malignancy, infection < 1 a wks ago or remotely

False +ve : BCG after 12 months of age in a low-risk individual

Booster effect: initially false -ve result boost to a true +ve result by the testing procedure itself
(usually if patient was infected long ago so had diminished delayed type hypersensitivity
reaction or if history of BCG).
Prevention
prevention of infection: BCG vaccine
~80% effective against pediatric miliary and meningeal TB
effectiveness in adults debated (anywhere from 0-80%)
routine use rarely recommended in Canadian populations
prevention of progression of latent to active disease (defer in pregnancy unless mother is high
risk)
likely INH-sensitive: isoniazid (INH) 300 mg + pyridoxine (vit B6) 50 mg PO OD x 9 months
likely INH-resistant: rifampin 600 mg PO OD x 4 months

Treatment of Active Infection


empiric therapy: INH + rifampin + pyrazinamide + ethambutol + pyridoxine
pulmonary TB: INH + rifampin + pyrazinamide + pyridoxine x 2 months (initiation phase),
then INH + rifampin + pyridoxine x 4 months in fully susceptible TB (continuation phase), total
6 months
extra-pulmonary TB: same regimen as pulmonary TB but increase to 12 months in bone/joint,
CNS, and miliary/disseminated TB + corticosteroids for meningitis, pericarditis
empiric treatment of suspected MDR (multidrug resistant) or XDR (extensively drug-resistant)
TB requires referral to a specialist
MDR = resistance to INH and rifampin others
XDR = resistance to INH + rifampin + fluoroquinolone + 1 of injectable, second-line
agents
suspect MDR TB if previous treatment, exposure to known MDR index case, or immigration
from a high-risk area
note: TB is a reportable disease to Public Health

150. A 34 year old male nurse is brought to the Emergency Room by the EMS with altered
mental status. As per his sister, the patient has been on Lithium for his manic symptoms for the
past one year. However, his psychiatrist added Fluoxetine and Amitriptyline about 1 week ago
for history of depression symptoms.

On physical examination, he has low grade fever at 100.5F, Blood pressure of 110/82 and Heart
rate of 120/min. He is confused and the pupils are dilated. There is rigidity in extremities and
deep tendon reflexes are exaggerated. Complete blood count, creatinine and Creatine kinase level
are within normal limits. Most likely diagnosis:
A) Neuroleptic Malignant Syndrome
B) Lithium Toxicity
C) Serotonin Syndrome
D) Amphetamine abuse
E) Cocaine Intoxication
Ans. C

The problem here is lithiums interaction with Prozac and this causes serotonin syndrome. Mild
symptoms may only consist of increased heart rate, shivering, sweating, dilated pupils,
myoclonus (intermittent tremor or twitching), as well as over responsive reflexes. Moderate
intoxication includes additional abnormalities such as hyperactive bowel sounds, high blood
pressure and hyperthermia; a temperature as high as 40 C (104 F) is common in moderate
intoxication. The overactive reflexes and clonus in moderate cases may be greater in the lower
limbs than in the upper limbs.

Extremely close to NMS but in NMS theres something you dont want to forget and that is
elevated CPK which is normal in this case.

mnemonic for NMS: FALTER


F Fever
A Autonomic instability
L Leukocytosis
T Tremor
E Elevated enzymes (elevated CPK)
R Rigidity of muscles

151. A 65 year old man is evaluated in the Emergency room for Shortness of breath and mild
chest pain. On examination, he has dullness to percussion in the left lung base. The breath
sounds are bronchial in nature. Vocal and tactile fremitus is increased in this area. Most likely
lung abnormality that can explain this patients physical examination findings:
A) Consolidation
B) Pneumothorax
C) Pleural Effusion
D) Lung Collapse
E) Hydropneumothorax

Ans. A

Suspect consolidation when you hear bronchial sound, increase vocal resonance and increased
tactile vocal fremitus or egophony on chest examination.
152. A 34 year old woman with history of recently diagnosed Rheumatoid Arthritis presents to
your office for follow up. She reports much improvement in her symptoms after starting
Ibuprofen for pain. She was also started on Hydroxychloroquine 3 weeks ago. Which of the
following is most appropriate in monitoring her therapy?
A) Liver function tests every month
B) Complete Blood Count every 3 months
C) Serum Creatinine every week
D) Ophthalmologic evaluation every year
E) Hydroxychloroquine serum levels every month

Ans. D

Patient taking this medication over long periods of time or at high doses have developed
irreversible damage to the retina of the eye. Opthalmologic exam needed every year to check for
retinopathy.

153. A 68 year old man presents to the clinic for progressive right hip pain. He has a history of
hearing impairment. He reports pain in the right hip when walking more than 1 block and also,
has difficulty putting shoes on his right foot. On examination, the range of motion is significantly
limited in the right hip. An X-ray of the right hip reveals significant loss of cartilage,
subchondral sclerosis and sub-chondral cysts. X-ray also reveals pagetoid changes in the right
iliac wing and right femoral neck. Serum alkaline phosphatase level is normal. The most
appropriate initial step in managing this patient is:
A) Bisphosphanates
B) Acetaminophen, Quadriceps strengthening and Tai-Chi
C) Bisphosphanates and Acetaminoiphen
D) Calcitonin
E) Oral prednisone

Ans. B

There are a number of treatments other than medications. These can substantially improve your
arthritis symptoms, and they are usually the first treatments recommended. Drug therapy is a key
component of an arthritis treatment plan. Many types of drugs are available.

Pain relief medications Analgesics relieve pain but do not have any effect on inflammation.
These drugs are often recommended when arthritis pain does not respond to nonpharmacologic
measures. Drugs in this class include acetaminophen and opioid (narcotic) analgesics.
Acetaminophen can relieve mild to moderate arthritis pain. To avoid the serious but rare side
effects of kidney and/or liver damage due to acetaminophen, it is important to follow dosing
instructions and to avoid drinking excessive amounts of alcohol.
The pain of sudden, severe arthritis exacerbations may require treatment with narcotic analgesics
such as codeine. Narcotics should be taken for only short periods of time because they can be
addictive. They are often most effective when taken together with nonsteroidal antiinflammatory
drugs (NSAIDs). Narcotics can also be combined with acetaminophen (eg, Tylenol 3 contains
acetaminophen-codeine).
Nonsteroidal antiinflammatory drugs Nonsteroidal antiinflammatory drugs (NSAIDs)
relieve pain and reduce inflammation. Many of the nonprescription products that are available
for treating arthritis pain are NSAIDs. These drugs are often recommended before analgesics for
people who have osteoarthritis and evidence of inflammation. They are also recommended for
some people with noninflammatory OA who do not get adequate pain relief with simple
analgesics.

154. A 68 year old man presents to the clinic for progressive right hip pain. He reports pain in
the right hip when walking more than 1 block and also, has difficulty putting shoes on his right
foot. On examination, the range of motion is significantly limited in the right hip. An X-ray of
the right hip reveals significant loss of cartilage, subchondral sclerosis and sub-chondral cysts.
The patient is diagnosed with Right Hip Osteoarthritis and is started on Acetaminophen. Which
of the following exercises should not be recommended to this patient at this time:

A) Stair climbing

B) Quadriceps strengthening

C) Tai-Chi

D) Swimming

E) Bicycling

Ans. A

A high load weight bearing exercise should be avoided. There are 4 activities that seem to
provide the most relief of symptoms:

Gentle hip stretching (Tai Chi).


Hip range of motion exercises in non-weight bearing position.
Stationary bicycling.
Aquatic therapy (swimming).
**Climbing stairs can aggravate OA if do it every day.

1.Lets reduce the load on the upper body, both static (single leg stance and prolong standing) and
dynamic(jumping or leaping or running).

2. Lets increase the lower body size, not with more fat, fat is uncontrollable; but with muscles.
How will muscles help?
Redistribution of pressure -With good quadriceps, and hamstrings, this patient can divert the
pressure away from the area of pain (hip) to other lower limb joints like the knees joints.
Increased lower body structure stability and reduction in risk of falls More muscles more
coordination and movement control. Work is transmitted efficiently to functional joints and
stronger muscle tone prevents falls.
Single leg stance increases the pressure on the all the joints in the active limb and this is bad for
any injured joint on the active limb.

155. A 55 year old man presents to the clinic for progressive right hip pain. He has a history of
hearing impairment. He reports pain in the right hip when walking more than 1 block and also,
has difficulty putting shoes on his right foot. On examination, the range of motion is significantly
limited in the right hip. An X-ray of the right hip reveals significant loss of cartilage,
subchondral sclerosis and sub-chondral cysts. X-ray also reveals pagetoid changes in the right
iliac wing and right femoral neck. Serum alkaline phosphatase level is 432 IU/L (n = 20 to 140
IU/L). The most appropriate initial step in managing this patient is:
A) Bisphosphanates
B) Acetaminophen
C) Bisphosphanates and Acetaminoiphen
D) Calcitonin Intranasally
E) Oral prednisone

Ans. C

This patient is having OA and Paget disease. Acetaminophen (Tylenol) is the first drug
recommended for osteoarthritis. Nonsteroidal anti-inflammatory drugs (NSAIDs) are
commonly used for arthritis pain. These include aspirin, ibuprofen (Motrin or Advil),
naproxen(Aleve), and ketoprofen (Orudis).

Bisphosphonates is the first line treatment for Paget disease. Five bisphosphonates are currently
available. In general, the most commonly prescribed are risedronic acid (Actonel), alendronic
acid (Fosamax), and pamidronic acid (Aredia). Etidronic acid (Didronel) and other
bisphosphonates may be appropriate therapies for selected patients but are less commonly used.
As a rule, bisphosphonate tablets should be taken with 200-250 mL (68 oz) of tap water on an
empty stomach. None of these drugs should be used by people with severe kidney disease.
156. A 25 year old woman presents to your office with complaints of muscle cramps and
weakness. On examination, she is afebrile, heart rate is 88/min and blood pressure is 150/98.
Ophthalmoscopic examination shows the following :

Laboratory investigations reveal a Sodium of 144 meq/L, potassium of 3.0meq/L, Chloride


98meq/L , Bicarbonate of 34meq/L and Creatinine of 0.8mg/dl. Urinary chloride is 45meq/L.
The most likely diagnosis that explains this patients clinical features:
A) Gitelmans syndrome
B) Chronic Laxative abuse
C) Diuretic Abuse
D) Primary Hyperaldosteronism
E) Bartters syndrome

Ans. D

The opthalmoscopic picture shows A-V nicking. This shows patient is hypertensive for a long
time and she is only 25. A-v nicking usually a feature of long standing hypertension. Her blood
pressure is also currently high. Diuretic abuse should have blood pressure on lower side due to
excess use of diuretics when it is not needed...some women with anorexia nervosa and psych
disorders may do this diuretic abuse. The clues are hypotension, orthostatic hypotension along
with hypokalemia. A-V nicking and hypertension is not seen in otherwise healthy 25 year old
woman who may resort to diuretic abuse.

157. A 25 year old woman presents to your office with complaints of muscle cramps and
weakness. She smokes 1 pack cigarettes per day and chews tobacco and flavored gum. She also
has history of alcoholism and ingests about one pint of vodka every day for the past 2 years. She
also uses Ecstasy during weekend parties. She has a history of snoring in the night. On
examination, afebrile, heart rate is 88/min and blood pressure is 150/98. Laboratory
investigations reveal a Sodium of 144 meq/L, potassium of 3.0meq/L, Chloride 98meq/L ,
Bicarbonate of 34meq/L and Creatinine of 0.8mg%. Urinary chloride is 45meq/L. Which of the
following points in the patients history is most helpful in diagnosing her condition?
A) Alcohol use
B) Chewing tobacco and flavored gum
C) Snoring in the night
D) Smoking
E) Use of Ecstasy

Ans. B

This patient has 20 shots of vodka daily. (10 x CDC recommendation) he has symptomatic
hypokalemia. But alcoholics, people who snore, smoke or use ecstasy do not come to the clinic
with symptomatic hypokalemia as frequently has people who are smokers, chew tobacco and
flavoring gum.

Hx of snoring alone, ecstasy alone, smoking alone do not conclude hyper aldosteronism;
But chewing tobacco and flavored gum plus smoking history is a red flag for etiology. licorice,
its extract glycirrhizin found in 90% of tobacco products, also found in flavored gum; Two
products with this substance, licorice. Licorice has minerocorticoid activity. This patient will
have bicarbonate sparing, potassium losing effect. His urine chloride will be lower than normal
range but it will be beyond the range that usually responds to saline because of aldosterone
escape phenomenon. If this substance is removed from his diet and habits, his symptoms will
likely resolve. If they persist, I will do PAC/PRA (plasma aldosterone concentration/plasma
rennin activity) to exclude primary hyperaldosteronism first before considering other common
causes of hypertension with hypokalemia like cushings syndrome, renovascular disease,
pheochromocytoma. Obstructive sleep apnea is a rare cause presented as case report in 1994.
There is no presented feature of this disease in this case except history of snoring which might as
well be from excess alcohol (20 shots of vodka) history given in this case.

Most helpful points in history to rule out licorice abuse is asking for habit of chewing gum or
tobacco or licorice laxative abuse.

158. A 55 year old man comes to the Emergency Room complaining of left upper quadrant
discomfort. His physical examination reveals Splenomegaly. Laboratory investigations were sent
but there was a significant delay in sending the specimen to the laboratory after collection.
Laboratory investigations reveal a WBC count of 110,000/l with neutrophilia, basophilia and
eosinophilia and Serum potassium of 3.0/l. Leucocyte Alkaline Phosphatase level is low. A
bone marrow biopsy is obtained and the results are pending. The most likely explanation of the
patients hypokalemia :
A) Delay in specimen transport to lab
B) Marked Leucocytosis and Delay in specimen transport
C) Tumor Lysis
D) Splenomegaly
E) Renal loss

You might also like